You are on page 1of 159

Arab Academy for Science, Technology &

Maritime Transport
Department of Basic & Applied Sciences

Mathematics 3
Lecture & Exercise Notes

Professor Nehad N. Morsi

2018
Contents

1 First-order Di¤erential Equations 1


1.1 Separable and Homogeneous Di¤erential Equations . . . . . . 1
1.1.1 Lecture Examples . . . . . . . . . . . . . . . . . . . . . 2
1.1.2 Classroom Exercises . . . . . . . . . . . . . . . . . . . 3
1.1.3 Revision Problems . . . . . . . . . . . . . . . . . . . . 5
1.2 Exact Di¤erential Equations . . . . . . . . . . . . . . . . . . . 7
1.2.1 Their Identi…cation and Method of Solution . . . . . . 7
1.2.2 Lecture Examples . . . . . . . . . . . . . . . . . . . . . 9
1.2.3 Classroom Exercises . . . . . . . . . . . . . . . . . . . 10
1.2.4 Revision Problems . . . . . . . . . . . . . . . . . . . . 12
1.3 Linear Di¤erential Equations . . . . . . . . . . . . . . . . . . . 13
1.3.1 Lecture Examples . . . . . . . . . . . . . . . . . . . . . 14
1.3.2 Classroom Exercises . . . . . . . . . . . . . . . . . . . 15
1.3.3 Revision Problems . . . . . . . . . . . . . . . . . . . . 16
1.4 Bernoulli Di¤erential Equations . . . . . . . . . . . . . . . . . 17
1.4.1 Lecture Examples . . . . . . . . . . . . . . . . . . . . . 17
1.4.2 Classroom Exercises . . . . . . . . . . . . . . . . . . . 18
1.4.3 Revision Problems . . . . . . . . . . . . . . . . . . . . 19
1.5 Further Revision Problems on Chapter 1 . . . . . . . . . . . . 22
1.6 Some Applications of First-order Di¤erential Equations in En-
gineering . . . . . . . . . . . . . . . . . . . . . . . . . . . . . . 26

2 Second-order Linear Di¤erential Equations 27


2.1 Form of the general solution . . . . . . . . . . . . . . . . . . . 27
2.2 Linear Di¤erential Equations with Constant Coe¢ cients . . . 29
2.2.1 The Homogeneous Solution yh (x) . . . . . . . . . . . . 29
2.3 A Particular Solution - Method of Undetermined Coe¢ cients . 31
2.3.1 Lecture Examples . . . . . . . . . . . . . . . . . . . . . 32
2.3.2 Classroom Exercises . . . . . . . . . . . . . . . . . . . 34
2.3.3 Revision Problems . . . . . . . . . . . . . . . . . . . . 38
2.4 Order Reduction of Linear Di¤erential Equations . . . . . . . 42

1
CONTENTS 2

2.5 Method of Variation of Parameters . . . . . . . . . . . . . . . 44


2.5.1 Describing The Method of Variation of Parameters . . 44
2.5.2 Steps of the Method of Variation of Parameters: . . . . 46
2.5.3 Lecture Examples . . . . . . . . . . . . . . . . . . . . . 47
2.5.4 Classroom Exercises . . . . . . . . . . . . . . . . . . . 47
2.5.5 Revision Problems . . . . . . . . . . . . . . . . . . . . 50
2.6 Cauchy-Euler Di¤erential Equations . . . . . . . . . . . . . . . 54
2.6.1 Description and Method of Solution . . . . . . . . . . . 54
2.6.2 Lecture Examples . . . . . . . . . . . . . . . . . . . . . 56
2.6.3 Classroom Exercises . . . . . . . . . . . . . . . . . . . 56
2.6.4 Revision Problems . . . . . . . . . . . . . . . . . . . . 58

3 The Laplace Transform 60


3.1 De…nition and Properties . . . . . . . . . . . . . . . . . . . . . 60
3.2 Problems on Direct Laplace Transform . . . . . . . . . . . . . 64
3.2.1 Lecture Examples . . . . . . . . . . . . . . . . . . . . . 64
3.2.2 Classroom Exercises . . . . . . . . . . . . . . . . . . . 68
3.2.3 Revision Problems . . . . . . . . . . . . . . . . . . . . 71
3.3 Inverse Laplace Transform . . . . . . . . . . . . . . . . . . . . 75
3.3.1 Lecture Examples . . . . . . . . . . . . . . . . . . . . . 75
3.3.2 Classroom Exercises . . . . . . . . . . . . . . . . . . . 76
3.3.3 Revision Problems . . . . . . . . . . . . . . . . . . . . 79
3.4 Fredholm Integral Equations . . . . . . . . . . . . . . . . . . . 83
3.4.1 Lecture Examples . . . . . . . . . . . . . . . . . . . . . 83
3.4.2 Classroom Exercises . . . . . . . . . . . . . . . . . . . 84
3.4.3 Revision Problems . . . . . . . . . . . . . . . . . . . . 85
3.5 Laplace Transform and Initial-value Problems: . . . . . . . . . 87
3.5.1 Lecture Examples . . . . . . . . . . . . . . . . . . . . . 87
3.5.2 Classroom Exercises . . . . . . . . . . . . . . . . . . . 88
3.5.3 Revision Problems . . . . . . . . . . . . . . . . . . . . 90
3.6 Laplace Transform and Systems of Di¤erential Equations . . . 92
3.6.1 Lecture Examples . . . . . . . . . . . . . . . . . . . . . 92
3.6.2 Classroom Exercises . . . . . . . . . . . . . . . . . . . 93
3.6.3 Revision Problems . . . . . . . . . . . . . . . . . . . . 95

A Table of Famous Integrals 98

B Assignments 99
CONTENTS 3

B.1 Assignment 1 on Separable and Homogeneous Di¤erential Equa-


tions . . . . . . . . . . . . . . . . . . . . . . . . . . . . . . . . 100
B.2 Assignment 2 on Exact Di¤erential Equations . . . . . . . . . 105
B.3 Assignment 3 on Linear Di¤erential Equations . . . . . . . . . 110
B.4 Assignment 4 on Bernoulli Di¤erential Equations . . . . . . . 115
B.5 Assignment 5 on Method of Undetermined Coe¢ cients . . . . 120
B.6 Assignment 6 on Method of Variation of Parameters . . . . . . 126
B.7 Assignment 7 on Cauchy-Euler Di¤erential Equations . . . . . 131
B.8 Assignment 8 on Direct Laplace Transform . . . . . . . . . . . 135
B.9 Assignment 9 on Inverse Laplace Transform . . . . . . . . . . 137
B.10 Assignment 10 on Solution of Fredholm Integral Equations . . 141
B.11 Assignment 11 on Laplace Transform and Initial-value Prob-
lems (1) . . . . . . . . . . . . . . . . . . . . . . . . . . . . . . 146
B.12 Assignment 12 on Laplace Transform and Initial-value Prob-
lems (2) . . . . . . . . . . . . . . . . . . . . . . . . . . . . . . 151
Chapter 1

First-order Di¤erential
Equations

1.1 Separable and Homogeneous Di¤erential


Equations
A …rst-order di¤erential equation (in an unknown function y (x)) takes the
form
dy
= f (x; y (x)) : (1.1)
dx
It is said to be Separable if it takes the form :
dy A (x)
= , (1.2)
dx B (y (x))
dy
which is rewritten as B (y (x)) dx = A (x). To solve this separable D.E., we
R dy
R
integrate both sides, and get B (y (x)) dx dx = A (x) dx+C. When we use
R dy
R
the substitution y = y (x), we …nd that B (y (x)) dx dx = B (y) dy y=y(x) .
Therefore, the general solution of (1.2) is implicit in the equation
Z Z
B (y) dy = A (x) dx + C. (1.3)
y=y(x)

A …rst-order di¤erential equation is said to be Homogeneous if it can


be put in the following form :
dy y (x)
=f .
dx x

1
CHAPTER 1. FIRST-ORDER DIFFERENTIAL EQUATIONS 2

The substitution u = xy (and so, dx dy


= u + x du
dx
) turns a homogeneous D.E.
into the following separable one (in the unknown function u (x)):

du du
u+x = f (u) ; that is x = f (u) u
dx dx

1.1.1 Lecture Examples


Solve the following …rst-order di¤erential equations:
2
Problem 1.1.1 (4y + yx2 ) y 0 = (y 2 + 25) :

dy 2 R ydy
R dx
y (4 + x2 ) dx = (y 2 + 25) Separable D.E. (y 2 +25)2
= 4+x2
R R dx
) 2y (y 2 + 25) dy = 2 4+x
2
2

) ) y 2 + 25 =
1
(y 2 + 25) = 22 tan 1 x
C 1
.
2 C tan 1
( x2 )

dy
Problem 1.1.2 (4xy 3 + 2x3 y) dx = 3y 4 + x2 y 2 + 5x4 :

y 3 y 4 y 2
Divide by x4 : ) 4 x
+2 y
x
dy
dx
=3 x
+ x
+5
Homogeneous D.E., Put u = y
x
.) y = xu ) dy
dx
= u + x du
dx
) (4u3 + 2u) u + x du
dx
= 3u4 + u2 + 5
) 4u4 + 2u2 + (4u3 + 2u) x du
dx
= 3u4 + u2 + 5
) (4u3 + 2u) x du = (u4 + u2 5) Separable D.E. in u (x).
R 4u3 +2u dx R dx
) u4 +u2 5 du = x
) ln (u + u2 5) = ln (C1 x) = ln
4 C
x
y4 2
) u4 + u2 5= C
x
) x4
+ xy 2 5= C
x
) y 4 + x2 y 2 5x4 = Cx3 .
CHAPTER 1. FIRST-ORDER DIFFERENTIAL EQUATIONS 3

1.1.2 Classroom Exercises


dy
p
Problem 1.1.3 cos2 x dx =x 1 y2:

Separable
R D.E. R R
) pdy = xdx
cos2 x
= x sec2 x dx
1 y2
R
) sin y = x tan x
1
tan xdx = x tan x + ln (cos x) + C
) y = sin (x tan x + ln (cos x) + C) .

dy
Problem 1.1.4 dx
5x4 = 5x4 y 2 ; y (0) = 1:
R dy R 4
dy
dx
= 5x4 (1 + y 2 ) Separable D.E. ) 1+y 2 = 5x dx
) tan y = C + x ; i.e. y = tan (C + x ) :
1 5 5

At x = 0; y = 1: ) tan 1 (1) = C + 0 ) 4 =C
) y = tan 4
+ x5 .

2y dy 2y 2y
Problem 1.1.5 2x3 cos x
+ 15xy 2 dx
= 2x2 y cos x
+x3 sin x
+20y 3 :

y 2
Divide by x3 : ) 2 cos 2y
x
+ 15 x
dy
dx
= 2 xy cos 2y
x
+ sin 2y
x
+
y 3
20 x
)Homogeneous D.E. Put u = y
x
. ) y = xu ) dy
dx
= u + x du
dx
) (2 cos (2u) + 15u2 ) u + x du
dx
= 2u cos (2u) + sin (2u) + 20u3
) (2 cos (2u) + 15u2 ) x du = sin (2u) + 5u3 Separable D.E. in u (x)
R 2 cos(2u)+15u2 Rdxdx
) sin(2u)+5u3
du = x ) ln (sin (2u) + 5u3 ) = ln (Cx)
y 3
) sin (2u)+5u3 = Cx ) sin 2y
x
+5 x
= Cx ) x3 sin 2y
x
+ 5y 3 = Cx4 .

dy
Problem 1.1.6 (x7 8x4 y 3 + 7xy 6 ) dx = 4y 7 2x3 y 4 2x6 y; y (1) =
1:

y 3 y 6 dy y 7 y 4 y
1 8 x
+7 x dx
=4 x
2 x
2 x
: Homogeneous
D.E.
Put u = y
x
. ) y = xu ) dy
dx
= u + x du
dx
) (1 8u3 + 7u6 ) u + x du
dx
= 4u7 2u4 2u
CHAPTER 1. FIRST-ORDER DIFFERENTIAL EQUATIONS 4

) (1 8u3 + 7u6 ) x du
dx
+u 8u4 + 7u7 = 4u7 2u4 2u
) (1 8u3 + 7u6 ) x dudx
= 3u7 + 6u4 3u: Separable D.E. in u (x).
R R
) 1u7 8u2u+7u
3 6
4 +u du = 3 dx x

) ln (u7 2u4 + u) = 3 ln (Cx) = ln (C 3


x 3)
y7 4
) u7 2u4 + u = C 3
x 3
) x7
2 xy 4 + y
x
=C 3
x 3

) y7 2x3 y 4 + x6 y = Kx4 :
At x = 1, y = 1, )K= 1 2 1= 4:
) y7 2x3 y 4 + x6 y = 4x4 :

p q
dy y+ x2 y 2 y y 2
Problem 1.1.7 dx
= x
= x
+ 1 x
; y (1) = 0

Homogeneous D.E. Put u = xy ) y = xu ) dx


dy
= u + x du
dx
p p
) Ru + x dx = Ru + 1 u
du 2 ) x dx = 1 u Separable D.E.
du 2

) p1duu2 = dx x
) sin 1
u = ln Cx
) x = u = sin (ln Cx) :
y
At x = 1, y = 0. ) 0 = sin (ln C)
) 0 = ln C )C=1 ) y = x sin (ln x) :

dy y
Problem 1.1.8 x dx = y + x cot x
:

) dy
dx
= y
x
+ cot y
x
Homogeneous D.E. Put u = y
x
.
) y = xu ) =u+ dy
dx
)u+ x du
= u + cot u
dx
x du
dx
)x du
Separable D.E. in u (x).
R dx = cot u R dx
) tan u du = x ) ln (sec u) = ln (Cx)
) sec u = Cx ) y
x
= u = sec 1
(Cx) ) y = x sec 1
(Cx) .
CHAPTER 1. FIRST-ORDER DIFFERENTIAL EQUATIONS 5

1.1.3 Revision Problems


dy
Problem 1.1.9 3x2 dx = 9x2 + 3xy + y 2 :
y
Homogeneous D.E. Put u = x
) y = xu ) dx
dy
= u + x du
dx
) 3x u + x dx = 9x2 + 3x2 u + x2 u2
2 du

)u = 3 + u + u3 ) x du
2 2 2
x du
R +3du dx R dx
= 3 + u3 = 9+u
3
Separable D.E. in u (x)
) 9+u2 = dx x
) tan 1 u3 = ln (Cx)
) y
3x
= u
3
= tan (ln (Cx)) : ) y = 3x tan (ln (Cx)) .

dy 2y 2 +7xy+5x2
Problem 1.1.10 dx
= xy+x2
; y (1) = 3

Homogeneous D.E. Put u = y


x
. ) y = xu ) dy
dx
= u + x du
dx
y 2
2( x ) +7 xy +5
)
2 +7u+5
u + x dudx
= y
+1
= 2u u+1
x

)
2 +7u+5 2
x du = 2u u+1 u = u +6u+5 = (u+1)(u+5) = u+5
R dxdu R dx u+1 u+1
) u+5
= x ) ln (u + 5) = ln (Cx)
) y
x
+ 5 = u + 5 = Cx: At x = 1; y = 3: )C=8
) y
x
+ 5 = 8x; that is, y = 8x2 5x .

dy
Problem 1.1.11 (x2 2x + 1) dx 2y = 0:

Final Answer: y = Ce2=1 x

Problem 1.1.12 (cos2 x cos y) y 0 + x sin y = 0:


x tan x
Final Answer: sin y = C sec xe

Problem 1.1.13 (x2 + 9) y 0 + xy = 0:

Final Answer: y= pC
x2 +9

Problem 1.1.14 (x2 + x2 y) y 0 + (1 x) y 2 = 0:


1 1
Final Answer: ln y y x
ln x = C
CHAPTER 1. FIRST-ORDER DIFFERENTIAL EQUATIONS 6

Problem 1.1.15 y 0 3x2 (1 + y 2 ) = 0:

Final Answer: y = tan (x3 + C)

6y
Problem 1.1.16 2 sec 2xdy e dx = 0:

Final Answer: 2e6y = 3 sin 2x + C

y dy ex
Problem 1.1.17 x dx
= ln y
:

Final Answer: y 2 (1 2 ln y) = 4ex (1 x) + C

y
Problem 1.1.18 xy 0 = y + x cot x
:
1
Final Answer: y = x sec (Cx)

dy
Problem 1.1.19 x2 + 3y 2 2xy dx = 0:
p
Final Answer: y= x Cx 1

p dy
p
Problem 1.1.20 xy x2 + y 2 dx = x3 + y 2 x2 + y 2 :

2=3
Final Answer: y 2 = x2 [ln (Cx3 )] x2
CHAPTER 1. FIRST-ORDER DIFFERENTIAL EQUATIONS 7

1.2 Exact Di¤erential Equations


1.2.1 Their Identi…cation and Method of Solution
All …rst-order di¤erential equations, in an unknown function y (x) ; can be
written as follows:
dy
M (x; y (x)) + N (x; y (x)) = 0; (1.4)
dx
In this section, we often simplify this writing as follows:

M (x; y) dx + N (x; y) dy = 0, (1.5)

Equation (1.4) is said to be Exact if it satis…es the following exactness


condition:
@M (x; y) @N (x; y)
= . (1.6)
@y @x
The solution of an exact di¤erential equation (1.4) consists of executing
the following three steps:
Step 1: Test the exactness condition (1.6). If it is valid for the given
equation, proceed to the next two steps.
Step 2: We recall the following theorem of Di¤erential Calculus:
Theorem 1 Two continuously di¤erentiable functions M (x; y) and N (x; y)
satisfy the exactness condition (1.6), in some region of the xy-plane, if and
only if a function (x; y) exists in that region, such that:

@ (x; y) @ (x; y)
= M (x; y) and = N (x; y) : (1.7)
@x @y
That function (x; y) is one and only one (up to additive constants).

Proof. First, suppose that M and N satisfy the exactness condition (1.6).
Choose a point (x ; y ) in their common region of de…nition. De…ne a function
(x; y) as follows:
Z x Z y
(x; y) = M (t; y) dt + N (x ; u) du: (1.8)
x y

Then we have @ @x (x;y)


= M (x; y) + 0; and
@ (x;y) R x @M (t;y) R x @N (t;y)
@y
= x @y
dt + N (x ; y) = x @t
dt + N (x ; y) (by (1.6)),
= N (x; y) N (x ; y) + N (x ; y) = N (x; y) ; as required.
CHAPTER 1. FIRST-ORDER DIFFERENTIAL EQUATIONS 8

Conversely, we suppose the existence of a function (x; y) that satis…es


the two identities in (1.7). Then we …nd that
@M (x;y) @ (x;y) 2 (x;y) 2 (x;y)

@y
@
= @y @x
= @ @y@x = @ @x@y @
= @x @ (x;y)
@y
= @N@x
(x;y)
;
that is, M and N must satisfy the exactness condition (1.6). This com-
pletes the proof.
Our task in Step 2 is to determine that function (x; y). We can do that
through computing the following two integrals, then comparing them with
each other:
Z
(x; y) = M (x; y) dx + A (y) ;
Z
(x; y) = N (x; y) dy + B (x) :

(Alternatively, you can compute from formula (1.8). But this could be a
bit more laborious!)
Step 3: We recall the following basic theorem of Di¤erential Calculus:.
Theorem 2 (Chain Rule in Several Variables) Let z (x), y (x) and (z; y)
be continuously di¤erentiable functions around points x and (z (x) ; y (x)), re-
spectively. Then at that point x we have:
d (z (x) ; y (x)) @ (z; y) dz @ (z; y) dy
= + :
dx @z jat (z(x);y(x)) dx @y jat (z(x);y(x))
dx
dz
In the special case that z (x) = x; we have dx
= 1. Therefore, the Chain
Rule becomes:
d (x; y (x)) @ (x; y) @ (x; y) dy
= + : (1.9)
dx @x jat (x;y(x)) @y jat (x;y(x))
dx

When we apply formula (1.9) to the particular function (x; y) (obtained


in Step 2) we can bene…t from the two identities in (1.7), and we come out
with:
d (x; y (x)) dy
= M (x; y (x)) + N (x; y (x)) :
dx dx
We combine this with our di¤erential equation (1.4), to …nd that
d (x; y (x))
= 0:
dx
CHAPTER 1. FIRST-ORDER DIFFERENTIAL EQUATIONS 9

But, the only functions of one variable whose derivatives are identically 0 are
the constant functions. We conclude that the general solution y (x) of the
di¤erential equation (1.4) is implicit in the equation:
(x; y (x)) = C; (1.10)
for each value of the arbitrary constant C.

1.2.2 Lecture Examples


4
Problem 1.2.1 (y 3 y 2 sin x + 3e3x ) dx + 3xy 2 + 2y cos x + y
dy = 0:

@M (x;y) @ (y 3 y 2 sin x+3e3x )


M dx + N dy = 0 @y
= @y
= 3y 2 2y sin x;
@N (x;y) @ (3xy 2 +2y cos x+ y4 )
@x
= @x
= 3y 2 2y sin x:
) @M@y (x;y)
= @N@x (x;y)
: ) Exact D.E.
R R
(x; y) = M (x; y) dx + A (y) = (y 3 y 2 sin x + 3e3x ) dx + A (y)
= xy 3 + y 2 cos x + e3x + A (y) ;
R R
(x; y) = N (x; y) dy + B (x) = 3xy 2 + 2y cos x + y4 dy + B (x)
= xy 3 + y 2 cos x + 4 ln y + B (x) :
) (x; y (x)) = xy 3 + y 2 cos x + e3x + 4 ln y = C :

Problem 1.2.2 (9 cos x + y 2 cos x) dx + 2y sin x dy = 0; y 2


= 4:

M (x; y) dx + N (x; y) dy = 0
@M (x;y) @ (9 cos x+y 2 cos x)
@y
= @y
= 2y cos x;
@N (x;y)
= @2y@xsin x = 2y cos x. ) @M@y(x;y)
= @N@x
(x;y)
: ) Exact D.E.
@x R R
(x; y) = M (x; y) dx+A (y) = (9 + y ) cos x dx+A (y) = (9 + y 2 ) sin x+
2

A (y) ; R R
(x; y) = N (x; y) dy + B (x) = 2y sin x dy + B (x) = y 2 sin x + B (x) :
p
(x; y (x)) = (9 + y 2 ) sin x = C: So by initial condition y = 25 csc x 9 .
2
Another
R cos Solution:
R 2ySeparable D.E. (9 + y ) cos x dx = 2y sin x dy
) x
sin x
dx = 9+y2 dy
) ln (C sin x) = ln (9 + y 2 ) ) 9 + y 2 = C sin
1
x
:
p
At x = 2 , y = 4. ) C = 1
25
and y = 25 csc x 9 .
CHAPTER 1. FIRST-ORDER DIFFERENTIAL EQUATIONS 10

1.2.3 Classroom Exercises


Problem 1.2.3 (cos x y sin x) dx + (2y + cos x) dy = 0; y (0) = 2

M (x; y) dx + N (x; y) dy = 0
@M (x;y)
@y
= @(cos x@yy sin x) = sin x; @N (x;y)
@x
= @(2y+cos x)
@x
= sin x:
) @M (x;y)
@y
= @N@x
(x;y)
: ) Exact D.E.
R R
(x; y) = M (x; y) dx + A (y) = (cos x y sin x) dx + A (y)
= sin x + y cos x + A (y) ;
R R
(x; y) = N (x; y) dy + B (x) = (2y + cos x) dy + B (x)
= y 2 + y cos x + B (x) :
) (x; y (x)) = sin x + y cos x + y 2 = C:
At x = 0; y = 2: ) sin 0 + 2 cos 0 + 4 = C. ) C = 6
) sin x + y cos x + y = 6 .
2

3
Problem 1.2.4 (2xey + cos x) dx + x2 ey y
dy = 0:

M (x; y) dx + N (x; y) dy = 0
3
@M (x;y) y @ (x2 ey )
@y
= @(2xe @y+cos x) = 2xey ; @N (x;y)
@x
= @x
y
= 2xey :
) @M@y(x;y)
= @N@x
(x;y)
: ) Exact D.E.
R R
(x; y) = M (x; y) dx + A (y) = (2xey + cos x) dx + A (y)
= x2 ey + sin x + A (y) ;
R R
(x; y) = N (x; y) dy + B (x) = x2 ey y3 dy + B (x)
= x2 ey 3 ln y + B (x) :
) (x; y (x)) = x2 ey + sin x 3 ln y = C:

x4 dy
Problem 1.2.5 2e2x sin y + 4x3 tan 1
y + e2x cos y + 1+y 2
+ sec2 y dx
= 0:

M (x; y) dx + N (x; y) dy = 0
@M (x;y) @ (2e2x sin y+4x3 tan 1 y) 4x3
@y
= @y
= 2e2x cos y + 1+y 2
;
4
@ e2x cos y+ x 2 +sec2 y
@N (x;y) 4x3
@x
= @x
1+y
= 2e2x cos y + 1+y 2
:
) @M@y (x;y)
= @N@x(x;y)
: ) Exact D.E.
R R
(x; y) = M (x; y) dx + A (y) = (2e sin y + 4x3 tan
2x 1
y) dx + A (y)
= e2x sin y + x4 tan 1 y + A (y) ;
CHAPTER 1. FIRST-ORDER DIFFERENTIAL EQUATIONS 11

R R x4
(x; y) = N (x; y) dy + B (x) = e2x cos y + 1+y 2
+ sec2 y dy + B (x)
= e2x sin y + x4 tan 1 y + tan y + B (x) :
) (x; y (x)) = e2x sin y + x4 tan 1 y + tan y = C .

Problem 1.2.6 (5x4 tan y + sec2 x) dx + (x5 sec2 y + 5y 4 ) dy = 0:

M (x; y) dx + N (x; y) dy = 0
@M (x;y) @ (5x4 tan y+sec2 x)
@y
= @y
= 5x4 sec2 y;
@N (x;y) @ (x5 sec2 y+5y 4 )
@x
= @x
= 5x4 sec2 y:
) @M@y (x;y)
= @N@x (x;y)
: ) Exact D.E.
R R
(x; y) = M (x; y) dx + A (y) = (5x4 tan y + sec2 x) dx + A (y)
= x5 tanR y + tan x + A (y) ; R 5 2
(x; y) = N (x; y) dy + B (x) = (x sec y + 5y 4 ) dy + B (x)
= x5 tan y + y 5 + B (x) :
) (x; y (x)) = x5 tan y + tan x + y 5 = C:

Problem 1.2.7 x3 ln y cos x sin4 y dx:


x4
+ 4y
4 sin x sin3 y cos y + sec y tan y dy = 0:

M (x; y) dx + N (x; y) dy = 0
@M (x;y) @ (x3 ln y cos x sin4 y ) x3
@y
= @y
= y
4 cos x sin3 y cos y;
4
@N (x;y) @ x4y 4 sin x sin3 y cos y+sec y tan y 3
@x
= @x
= xy 4 cos x sin3 y cos y + 0
) @M@y (x;y)
= @N@x(x;y)
: ) Exact D.E.
R R 3
(x; y) = M (x; y) dx + A (y) = x ln y cos x sin4 y dx + A (y)
4
= x4 ln y sin x sin4 y + A (y) ;
R R x4
(x; y) = N (x; y) dy+B (x) = 4y
4 sin x sin3 y cos y + sec y tan y dy+
B (x)
x4
= 4
ln y sin x sin4 y + sec y + B (x) :

) (x; y) = x4
4
ln y sin x sin4 y + sec y = C :
CHAPTER 1. FIRST-ORDER DIFFERENTIAL EQUATIONS 12

1.2.4 Revision Problems


Problem 1.2.8 (3x2 tan y + cos x + sec y) dx+(x3 sec2 y + x sec y tan y 2e2y ) dy =
0:

M (x; y) dx + N (x; y) dy = 0
@M (x;y) @ (3x2 tan y+cos x+sec y )
@y
= @y
= 3x2 sec2 y + 0 + sec y tan y
@N (x;y) @ (x3 sec2 y+x sec y tan y 2e2y )
@x
= @x
= 3x2 sec2 y + sec y tan y 0:
) @M@y (x;y)
= @N@x (x;y)
: ) Exact D.E.
R R
(x; y) = M (x; y) dx = (3x2 tan y + cos x + sec y) dx + A (y)
= x3 tan y + sin x + x sec y + A (y)
R R
(x; y) = N (x; y) dy = (x3 sec2 y + x sec y tan y 2e2y ) dy + B (x)
= x3 tan y + x sec y e2y + B (x)
(x; y (x)) = x3 tan y + sin x + x sec y e2y = C .
(by taking A (y) = e2y and B (x) = sin x)

1
Problem 1.2.9 (2y 3 e2x + cos x) dx + 3y 2 e2x + y
dy = 0; y (0) = 1:
Final Answer: y 3 e2x + sin x + ln y = 1

Problem 1.2.10 Exact(7y + 3x 2) y 0 + (3y + 12x2 ) = 0:


7 2
Final Answer: 2
y + 3xy 2y + 4x3 = C

Problem 1.2.11 (y sin x + cos x) dx cos x dy = 0:


Final Answer: y = tan x + C sec x

Problem 1.2.12 (2yx2 + 4) y 0 + (2y 2 x 6) = 0:


Final Answer: y 2 x2 + 4y 6x = C

Problem 1.2.13 y + x2 + x1 dx + (x sec2 y) dy = 0:


3
Final Answer: xy + x3 + ln x tan y = C:
CHAPTER 1. FIRST-ORDER DIFFERENTIAL EQUATIONS 13

1.3 Linear Di¤erential Equations


A …rst-order di¤erential equation is said to be Linear if it takes the form :
dy
+ p (x) y = q (x) : (1.11)
dx
The integrating factor of this equation is the function (x) given by
R
p(x) dx
(x) = e . (1.12)
When we multiply both sides of (1.11) by , we obtain the following di¤er-
ential equation:
dy
(x) + (x) p (x) y (x) q (x) = 0:
dx
In the form N (x; y (x)) dy + M (x; y (x)) dx = 0; this d.e. becomes
(x) dy + ( (x) p (x) y (x) q (x)) dx = 0: (1.13)
We show that this d.e. is exact, with M (x; y) = (x) p (x) y (x) q (x)
and N (x; y) = (x) :
d d
R
p(x) dx
R
p(x) dx d
R
First, we have dx
= dx
e . =e . dx p (x) dx
d
) = p:
dx
Therefore,
@M (x;y) @
@y
= @y ( (x) p (x) y (x) q (x)) = p and
@N (x;y)
@x
= ddx = p:
So, @M@y (x;y)
= @M@y
(x;y)
, ensuring that the d.e. (1.13) is exact.
We obtain the unique function (x; y) such that @ @x (x;y)
= M (x; y) and
@ (x;y)
= N (x; y):
@y R R
R (x;
d
y) =R M (x; y) dx = R ( (x) p (x) y (x) q (x)) dx
= y dx dx (x)R q (x) dx = y R (x) q (x) dx + A (y)
Also, (x; y) = N (x; y) dy = (x) dy = y + B (x) :
By comparing
R these two descriptions of the function , Rwe have to take
B (x) = (x) q (x) dx, A (y) = 0 and (x; y) = y (x) q (x) dx.
The solution of the exact d.e.R (1.13) is given by
(x; y (x)) = (x) y (x) q (x) dx = C.
This leads immediately to the following explicit solution of (1.11):
Z
1
y (x) = C+ (x) q (x) dx . (1.14)
(x)
CHAPTER 1. FIRST-ORDER DIFFERENTIAL EQUATIONS 14

1.3.1 Lecture Examples


dy
Problem 1.3.1 dx
+ (tan x) y = 10e5x cos x; y (0) = 8:

dy
Linear D.E., dx
+ p (x) y = q (x) :
R R
p = tan x; q = 10e5x cos x; p dx = tan x dx = ln (sec x) :
R
p dx
Integrating factor: =e = eln(sec x) = sec x:
R R
)y= 1 C+ q dx = cos x C + 10e5x dx
= cos x (C + 2e5x ) :
At x = 0, y = 8, )8=C +2 ) C = 6:
5x
) y = cos x (6 + 2e ) :

dy
Problem 1.3.2 dx
+ 3 (tan x) y = ex cos3 x; y (0) = 5:

dy
Linear D.E., dx
+ p (x) y = q (x) :
p = 3 tan x q = ex cos3 x:
R R
p dx 3 tan x dx
Integrating factor: =e =e = e3 ln(sec x) = sec3 x:
R R
)y= 1 C+ q dx = sec13 x C + sec3 x ex cos3 xdx
R x 3
) y = cos3 x C + ex dx ) y = (C + e ) cos x:
At x = 0, y = 5, ) 5 = 1 (C + 1) ) C = 4:
x 3
)y = (4 + e ) cos x.
CHAPTER 1. FIRST-ORDER DIFFERENTIAL EQUATIONS 15

1.3.2 Classroom Exercises


dy
Problem 1.3.3 x2 dx + 3xy = 3ex :
dy x x
Linear D.E. dx + x3 y = x3e2 R :, p= 3
R 3x
q = x3e2
3
Integrating factor: R = e p dx = e x dx = e3 ln x = eln x = x3 :
C+
R
q dx C 3 x ex dx R x
y= = x3
= x13 C 3 x ex e dx
)y= 1
x3
(C 3 (x ex ex )) : )y= 1
x3
(C 3x ex + 3ex ) :

dy 2x 2
Problem 1.3.4 dx
+ x2 16
y = x 4
:

dy
Linear D.E., dx
+ p (x) y = q (x) :
p = x22x16 q = x2 4 :
R 2x
R dx
= eln(x 16)
p dx x2 16 2
Integrating factor: =e =e = x2 16:
R R 2(x2 16) R
y= 1 C+ q dx = x2 1 16 C+ x 4
dx = 1
x2 16
C+ 2x + 8dx

) y= x2 +8x+C
x2 16
:

dy
Problem 1.3.5 dx
(tan x) y = 4 sin x:

dy
Linear D.E., dx
+ p (x) y = q (x) :
p= tan x q = 4 sin x:
R R
p dx tan x dx
Integrating factor: =e =e = eln(cos x) = cos x:
R R
)y= 1 C+ q dx = cos1 x C + 4 sin x cos xdx
2
) y = sec x C + 2 sin x
Also, y = sec x (C1 2 cos2 x) ) y = C1 sec x 2 cos x:
CHAPTER 1. FIRST-ORDER DIFFERENTIAL EQUATIONS 16

1.3.3 Revision Problems


dy
Problem 1.3.6 dx
+ (tan x) y = sec x:
dy
Linear D.E., dx + p (x) y =R q (x) ; R p = tan x q = sec x:
p dx tan x dx ln(sec x)
Integrating factor:
R =e =R e =e = sec x:
y= 1 C+ q dx = cos x C + sec2 x dx = cos x (C + tan x)
) y = C cos x + sin x:

dy 2
Problem 1.3.7 dx x
y = 4x2 sec2 x
dy
Linear D.E., dx
+ p (x) y = q (x) :
2
R R 2
p= x
q = 4x2 sec2 x: p dx = x
dx = 2 ln x
R
p dx ln(x 2 )
Integrating factor: =e =e = x12 :
R R
)y= 1 C+ q dx = x2 C + 4 sec2 xdx

) y = x2 (C + 4 tan x) :

Problem 1.3.8 y 0 + 2xy = 2x:


x2
Final Answer: y = 1 + Ce

Problem 1.3.9 y 0 + y cos x = cos x:


sin x
Final Answer: y = 1 + Ce
CHAPTER 1. FIRST-ORDER DIFFERENTIAL EQUATIONS 17

1.4 Bernoulli Di¤erential Equations


A …rst-order di¤erential equation is said to be a Bernoulli’s D.E. if it takes
the following form (n is a constant 6= 0; 1):

dy
+ g (x) y = h (x) y n : (1.15)
dx
We multiply both sides by
n
(1 n) y (1.16)
and get
n dy
(1 n) y + (1 n) g (x) y 1 n
= (1 n) h (x) :
dx
The substitution
z = y1 n
(1.17)
dz dy
(and so, dx = (1 n) y n dx ) turns this Bernoulli D.E. into the following
linear di¤erential equation in z (x) :

dz
+ (1 n) g (x) z = (1 n) h (x) :
dx
We obtain its solution z (x), then we get y (x) from (1.17).

1.4.1 Lecture Examples


dy
Problem 1.4.1 x dx + 2y x6 y 3 = 0:
dy
Bernoulli D.E., n = 3. dx
+ x2 y = x5 y 3
Multiply by (1 n) y n = 2y 3 . ) 2y 3 dx dy 4
x
y 2 = 2x5
Put z = y 2: ) dx
dz
= 2y 3 dx dy
:
) D.E. becomes dz
dx
4
x
z = 2x :5

Linear D.E. in z, p = Rx4 q =R 2x5 :


4 4
Integrating Rfactor: = e p dx R= e x dx = e 4 ln x
= eln x = x 4:
z= 1 C+ q dx = x4 C 2 x dx
1
y2
= z = x4 (C x2 )
) y= x2
p1
C x2
:
CHAPTER 1. FIRST-ORDER DIFFERENTIAL EQUATIONS 18

1.4.2 Classroom Exercises


dy
Problem 1.4.2 x dx + 2y = (x5 cos x) y 3 :
dy
+ x2 y = (x4 cos x) y 3 : Bernoulli D.E, n = 3.
dx
Multiply by (1 n) y n = 2y 3 . ) 2y 3 dx dy 4
x
y 2 = 2x4 cos x
Put z=y 2. ) dx
dz
= 2y 3 dxdy
:
) D.E. becomes R dx x zR= 2x cos x
dz 4 4

Linear D.E. in z, p dx = R x4 dx = 4 ln x = ln x14


p dx
Integrating factor:
R =e = x14R:
1
y2
=z= 1 C+ q dx = x4 C 2 cos x dx = x4 (C 2 sin x)
) y= p 1
x2 C 2 sin x
:

Problem 1.4.3 4y 0 + y cos x = y 3


cos x; y (0) = 2:

y 0 + 41 y cos x = 14 y 3 cos x Bernoulli D.E, n = 3 Multiply by


n 3
(1 n) y = 4y
) 4y 3 y 0 + y 4 cos x = cos x
Put z = y4 . ) dx
dz dy
= 4y 3 dx :
) D.E. becomes dz
dx
+ (cos x) z = cos x
R
p dx
R
Linear D.E. in z,R Integrating factor: R = e = e cos x dx = esin x :
4 1 sin x sin x
y =z= C+ q dx = e C+ e cos x dx = e sin x C + esin x
) y 4 = 1 + Ce sin x At x = 0; y = 2: ) 16 = 1 + C
p
) C = 15: Thus, y= 4
1 + 15e sin x :

dy
Problem 1.4.4 dx
(cos x) y = x3 e4 sin x y 3 : Bernoulli D.E., n = 3:
n
Multiply by (1 n) y = 4y 3 .
) 4y 3 dx
dy
4 (cos x) y 4 = 4x3 e4 sin x : Put z = y 4 . ) dz
dx
dy
= 4y 3 dx :
dz
) D.E. becomes Linear in z : dx
4 (cos x) z = 4x3 e4 sin x p =
4 cos x q = 4x3 e4 sin x :
R R
p dx 4 cos x dx 4 sin x
Integrating factor: =e =e =e :
R R
) y4 = z = 1 C + q dx = e4 sin x C + 4x3 dx = e4 sin x (C + x4 )
p p
) y = 4 e4 sin x (C + x4 ): ) y = esin x 4 C + x4 : y = ln(Kx)
x
:
CHAPTER 1. FIRST-ORDER DIFFERENTIAL EQUATIONS 19

dy p
Problem 1.4.5 dx
+ 2y = 6e2x y:
n 1
Bernoulli D.E, n = 1=2 Multiply by (1 n) y = p
2 y
.
p p
) 2p1 y dx
dy
+ y = 3e2x Put z = y: ) dx
dz
= 1
p
dy
2 y dx
:
) D.E. becomes dz
dx
+ z = 3e2x
Linear D.E. in z, p = 1, q =R 3e2x
p dx
Integrating factor: =e = ex :
p R R
y=z= 1 C+ q dx = e x C + 3e3x dx
p
) y = e x (C + e3x ) = Ce x + e2x ) y = (Ce x 2
+ e2x ) :

1.4.3 Revision Problems


dy 3
Problem 1.4.6 4 cos x dx + (sin x) y = y
n
Bernoulli D.E., n = 3. Divide by 4 cos x and multiply by (1 n) y =
3
4y .
) 4y 3 dx
dy
+ (tan x) y 4 = sec x
Put z = y4: ) dx
dz dy
= 4y 3 dx :
) D.E. becomes dz
dx
+ (tan x) z = sec x
Linear D.E. in z, p = tan
R x qR = sec x:
p dx
Integrating Rfactor: =e =R e tan x dx = eln(sec x) = sec x:
1
z= C+ q dx = cos x C + sec2 x dx = cos x (C + tan x)
y 4 = z = C cos x + sin x
p
) y = 4 C cos x + sin x:

dy 1 ex 4
Problem 1.4.7 dx x
y = x2
y
n
Bernoulli D.E., n = 4. Multiply by (1 n) y = 3y 4 .
) dy x
3y 4 dx + 3 x1 y 3 = x3e2
Put z = y 3: ) dx
dz
= 3y 4 dxdy
:
) D.E. becomes dz
dx
3 3ex
+ x z = x2 :
x
Linear D.E. in z, p = x3 R q = Rx3e2 :
3 3
Integrating factor: R = e p dx = e x dx = e3 ln x = eln x = x3 :
C+
R
q dx C 3 x ex dx R x
z= = x3
= x13 C 3 x ex e dx
) 1
y3
=z= 1
x3
(C 3x ex + 3ex ) ) y= p
3
x
C 3x ex +3ex
:
CHAPTER 1. FIRST-ORDER DIFFERENTIAL EQUATIONS 20

dy
Problem 1.4.8 x2 dx xy = y 2 :

dy y 2
dx x
= xy Bernoulli D.E., n = 2
Multiply by (1 n) y n = y 2 . ) y 2 dx
dy
+ x1 y 1
= 1
x2
Put z = y 1: ) dx
dz
= y 2 dxdy
:
) D.E. becomes dz
dx
1
+ x z = x2 1
R
p dx
Linear D.E. in z,
R Integrating factor:
R = e = eln x = x:
1 1 1 1 1
y
=z= C+ q dx = x C x
dx = x (C ln x)
) y= x
C ln x
= x
ln K+ln x
= x
ln(Kx)
:

Another Solution: Homogeneous D.E.


2
dy
dx
y
x
= xy Put u = xy . ) y = xu ) dy
dx
= u + x du
dx
)u du
R + x2 dx uR =dxu
2
) x du
dx
= u2
) u du = x ) u = yx = ln (Kx)
1
)y= x
ln(Kx)
:

Problem 1.4.9 y 0 y = y 3 e2x :


p x
Final Answer: y= p 2 e
C e4x

dy
Problem 1.4.10 x2 dx 2xy = 3y 4 :

x2
Final Answer: y= p
3
C (9x5 =5)

dy
Problem 1.4.11 dx
+ x3 y = 2x9 y 3 ; y (1) = 1:

dy
Problem 1.4.12 (2x3 9xy 2 ) dx = 2x3 + 6x2 y 15y 3 ; y (1) = 1:

dy
Problem 1.4.13 (6xy 2 3x2 y) dx = 10y 3 6xy 2 + 2x3 ; y (1) = 2:

dy
Problem 1.4.14 dx
(cot x) y = 2 (cos x) y 2 :

dy
Problem 1.4.15 x dx + 3y = (x7 tan x) y 3 :

dy 4
Problem 1.4.16 5 cos x dx + (sin x) y = y :
CHAPTER 1. FIRST-ORDER DIFFERENTIAL EQUATIONS 21

dy 1
Problem 1.4.17 dx 3
(sec x) y = (cos x) y 4 ; y (0) = 1:

dy
Problem 1.4.18 dx
+ 34 y = (e 3x
cos 2x) y 3 :

dy
Problem 1.4.19 dx
+ 14 (tan x) y = 3 (e3x cos x) y 3 :

dy
Problem 1.4.20 dx
+ x2 y = 2x4 y 3 :

dy
Problem 1.4.21 dx
+ x1 y = 1 2x
x3
e y 2:

dy
Problem 1.4.22 3 dx (tan x) y = 2 (sin x) y 2 :

dy 1
Problem 1.4.23 dx x
y = x2 y 2 :

dy
Problem 1.4.24 x2 dx xy = y 2 :
CHAPTER 1. FIRST-ORDER DIFFERENTIAL EQUATIONS 22

1.5 Further Revision Problems on Chapter 1


dy y 2 5xy
Problem 1.5.1 dx
= y 2 xy
; y (1) = 2

Homogeneous D.E. Put u = y


x
. ) y = xu ) dy
dx
= u + x du
dx
) u + x du
2 2 2
dx
= xx2uu2 5x u
x2 u
= uu 51
) x du
2 2
dx
= uu 15 u = u 5u u1 +u = 2u u 5 1 u
R R dx R R
) u(u2 2u+5
1)du
= x
) u22u2u+5
2
du = 2 dx
x
) ln (u2 2u + 5) = 2 ln (Cx) = ln xK2
) u2 2u + 5 = xK2 : ) K = u2 x2 2ux2 + 5x2 = y 2 2yx + 5x2
p
At x = 1; y = 2: ) K = 5: ) y 2 2yx + 5x2 = 5 . ) y = x + 5 4x2 .

1
Problem 1.5.2 (sec2 x + yexy ) dx + y
+ xexy dy = 0; y (0) = 1

M (x; y) dx + N (x; y) dy = 0
@M (x;y) @ (sec2 x+yexy )
@y
= @y
= exy + xyexy ;
@N (x;y) @ ( y1 +xexy )
@x
= @x
= exy + xyexy :
) @y = @x :
@M (x;y) @N (x;y)
) Exact D.E.
R R
(x; y) = M (x; y) dx + A (y) = (sec2 x + yexy ) dx + A (y)
= tan x + exy + A (y) ;
R R 1
(x; y) = N (x; y) dy + B (x) = y
+ xexy dy + B (x)
= ln y + exy + B (x) :
) (x; y (x)) = tan x + exy + ln y = C:
At x = 0; y = 1: ) 0 + 1 + 0 = C: ) tan x + exy + ln y = 1 .

Problem 1.5.3 x2 y 0 = y xy, y (1) = 2e :

Final Answer: Separable di¤erential equation, y = x2 e 1=x

dy
p
Problem 1.5.4 y dx = 2x 9 + y2, y (0) = 4:

Final Answer:
q Separable di¤erential equation,
p
y= (x2 + 5)2 9 = x4 + 10x2 + 16
CHAPTER 1. FIRST-ORDER DIFFERENTIAL EQUATIONS 23

dy y y
Problem 1.5.5 dx
= x
+ sin2 x
, y (1) = 4 :
1 e
Final Answer: Homogeneous d.e., y = x cot ln x

+ (x + y)2 = 0,
2 dy
Problem 1.5.6 2xy + x2 2yey dx
y (3) = 0:

2
Final Answer: Exact d.e. x3 + 3x2 y + 3xy 2 3ey = 24

dy
Problem 1.5.7 y 2 + 2x sin y ex + (2y + 2xy + x2 cos y) dx = 0; y (0) = 0:

Final Answer: Exact d.e. xy 2 + x2 sin y ex + y 2 = 1

dy
Problem 1.5.8 dx
+ 3y = 7e4x , y (0) = 3:

Final Answer: Linear d.e. y = e4x + 2e 3x

Problem 1.5.9 Solve the following …rst-order di¤erential equations:


dy
1. (xy 2 + x3 ) dx = 2y 3 + 4x2 y:

dy
2. (xy + 3x2 ) dx = 2y 2 + 9xy 7x2 ; y (1) = 2:

dy
3. (3xy 2 + 6x2 y) dx = 2y 3 + 3xy 2 2x3 :

dy
4. (4xy 3 + 2x3 y) dx = 3y 4 + x2 y 2 + 5x4 :

dy
5. (4xy 3 15x2 y 2 ) dx = 2y 4 5xy 3 + 2x4 :

dy y
6. x dx = 3y ln x
+ y; y (1) = 1:

dy
7. (3xy 2 4x2 y + 3x3 ) dx = 5y 3 8xy 2 + 9x2 y:

dy
8. (3xy 2 + 3x3 ) dx = 2y 3 + 4x3 ; y (1) = 0:
CHAPTER 1. FIRST-ORDER DIFFERENTIAL EQUATIONS 24

dy
9. (9x2 y 2 + 4xy 3 ) dx = 2y 4 + 3xy 3 2x4 :

dy
10. (12x2 y 3 10xy 4 ) dx = 3x5 + 3xy 4 4y 5 :

dy
11. (3xy + x2 ) dx = 6y 2 + 3xy; y (1) = 2:

dy
12. x2 dx xy = y 2 :

1 dy
13. (3e2y cos 3x sin x) + 2e2y sin 3x + 1+y 2 dx
=0; y 3
= 1:

y2 1
14. 3x2 sin y x
+ 1+x2
dx + (x3 cos y 2y ln x) dy = 0; y (1) = :

3
15. (2e2x + 3x2 tan y) dx + y
+ x3 sec2 y dy = 0; y (0) = 1:

1
16. (3x2 y 2 + ey sec x tan x cos x) dx + 2x3 y + ey sec x + y
dy = 0 :

4
17. (2e2x sin 3y + sec2 x) dx + 3e2x cos 3y + y
dy = 0 :

y2 dy
18. x
sin ( x) + 2x tan y + (3y 2 + 2y ln x + x2 sec2 y) dx =0;
y (1) = 0:

19. 2xy 3 + ey + p 1 dx + (3x2 y 2 + xey + sec2 y) dy = 0; y (0) = 4 :


1 x2

20. (2x + 2xy + y 2 cos x) dx + (x2 + 2y sin x ey ) dy = 0; y (0) = 0:

2y
21. (2x cos y + sec2 x) dx + 1+y 2
x2 sin y dy = 0; y (0) = 0:

dy
22. (2 cos 2x tan y + 3e3x ) + (sin 2x sec2 y + 5y 4 ) dx =0; y (0) = 1:

3
23. y 2 ex + y x
dx + (2yex + x + cos y) dy = 0:

24. (y 3 sec2 x + 4x3 cos y) dx + (3y 2 tan x x4 sin y + ey ) dy = 0:

dy
25. dx
+ (sec x) y = 3 cos x:
CHAPTER 1. FIRST-ORDER DIFFERENTIAL EQUATIONS 25

dy
26. dx
+ y tan x = 5 cos2 x; y (0) = 7:

dy
27. dx
+ x5 y = 6 sin x
x5
; y (1) = 4

dy 2x 2
28. dx
+ x2 16
y = x 4
; y (0) = 1:

dy
29. e3x dx + 3e3x y = 4 cos 2x:

dy
30. (csc x) dx (sec x) y = 4:

dy x
31. dx
+y =e cos x; y (1) = 2:

dy 4
32. dx
4x3 y = 3ex cos 3x; y (0) = 2:

dy
33. x dx 2y = x3 sec2 x; y( ) = 2
:
CHAPTER 1. FIRST-ORDER DIFFERENTIAL EQUATIONS 26

1.6 Some Applications of First-order Di¤er-


ential Equations in Engineering
Problem 1.6.1 A body is projected vertically from the surface of earth. At
distance x from the centre of earth, the acceleration is given by
d2 x dv gR2
= = ; x R; (1.18)
dt2 dt x2
where v (t) is the velocity, g is the earth gravitational constant and R is the
earth’s radius. By using the substitution dv dt
= dv(x(t))
dt
dv dx
= dx dt
dv
= v dx , the
di¤erential equation (1.18) becomes the following separable D.E., describing
the velocity v in terms of the distance x:
dv gR2
v = ; x R:
dx x2
Given that the initial velocity is v (at x = R), show that
r
2gR2
v (x) = (v 2 2gR) + :
x
Hence, show that the escape velocity from the earth’s surface (ignoring air
resistance) equals p
v escape = 2gR.

Problem 1.6.2 RIn a series RC-circuit, the current I (t) is governed by the
equation RI + C1 I dt = E (t), where E (t) is the electric potential applied to
the circuit. When we di¤erentiate both sides, we get the di¤erential equation
dI 1 dE
R + I= :
dt C dt
Given that E (t) = E e kt and the initial current is 0, …nd I (t).
8
kE C
< kRC 1
e kt e t=RC ; if kRC 6= 1;
Final Answer: I (t) =
: kER t e t=RC ; if kRC = 1:

Problem 1.6.3 The di¤erential equation for a series RL-circuit is


dI
L + RI = E (t) :
dt
kt
Given that E (t) = E e and the initial current is 0, …nd I (t).
( E
R kL
e kt e Rt=L ; if R 6= kL;
Final Answer: I (t) = E
:
kt
L
te ; if R = kL:
Chapter 2

Second-order Linear
Di¤erential Equations

2.1 Form of the general solution


A second-order linear di¤erential equation (in an unknown function y (x))
takes the following form:

d2 y dy
a (x) 2
+ b (x) + c (x) y = Q (x) : (2.1)
dx dx
All solutions of this D.E. are provided within its general solution yG:S: (x).
The corresponding homogeneous equation is

d2 y dy
a (x) 2
+ b (x) + c (x) y = 0: (2.2)
dx dx
Its general solution is called the homogeneous solution of (2.1) (also called
the cpmplementary function) of (2.1), and is denoted by yh (x).
We say that two functions y1 (x) and y2 (x) are independent if their ratio
y1 (x)
y2 (x)
is not a constant function.

Theorem 3 The homogeneous equation (2.2) has solutions of dimension 2.


Let y1 and y2 be any two independent solutions of (2.2).Then the general
solution yh (x) of (2.2) is given by

yh (x) = C1 y1 (x) + C2 y2 (x) ; (2.3)

in which C1 and C2 are arbitrary constants.

27
CHAPTER 2. SECOND-ORDER LINEAR DIFFERENTIAL EQUATIONS28

Let let yp (x) be any solution of (2.1). Then the general solution yG:S: (x)
of (2.1) is given by

yG:S: (x) = yp (x) + yh (x) (2.4)


= yp (x) + C1 y1 (x) + C2 y2 (x) :

Proof. Let y1 be any nonzero solution of (2.2). We use the substitution:


y (x)
V (x) = : (2.5)
y1 (x)
2 2
dy d2 V
So, y = V y1 . Hence, dx = V dydx
1
+ dV y and dx
dx 1
d y d y1 dV dy1
2 = V dx2 + 2 dx dx + dx2
y1 :
Then the di¤erential equation (2.1) becomes
2 2
a V ddxy21 + 2 dV dy1
dx dx
+ ddxV2 y1 + b V dy dx
1
+ dV y + cV y1 = q
dx 1
2
) V a ddxy21 + b dy
2
dx
1
+ cy1 + ay1 ddxV2 + 2a dy
dx
1
+ by1 dV
dx
= q:
d2 y
Since a dx21 + b dydx
1
+ cy1 = 0, we are left with the following di¤erential
equation:
2
ay1 ddxV2 + 2a dy
dx
1
+ by1 dVdx
= q: Then we use a second substitution:
dV
z (x) = (2.6)
dx
dz 2
Hence, dx = ddxV2 . And we come out with the following …rst-order, linear
di¤erential equation in z (x) : dz
ay1 dx + 2a dy
dx
1
+ by1 z = q; that is

dz 2a dy1 b q
+ + z= :
dx y1 dx a ay1
Z
1 q
Its solution is z (x) = ay1
dx + C2 = z (x) + C2 z2 (x) ; say, where
R 2a dy1
+b dx
= e y1 dx a is the integrating factor.
But, we conclude,
R from (2.6),R that R
)
R V (x) = z (x) dx + C 1 = (z (x) + C 2 z2 (x)) dx + C 1 = z (x) dx +
C2 z2 (x) dx + C1
We apply (2.5)R and get yG:S: (x) = V (x) R y1 (x) :
) yG:S: (x) = z (x) dx y1 (x) + C2 z2 (x) dx y1 (x) + C1 y1 (x)
= yp (x) + C1 y1 (x) + C2 y2 (x) ; say.
On one hand, when we repeat this derivation for the general solution yh
of the homogeneous equation (2.2) (i.e. with q = 0), we …nd that yp = 0 and
that yh = C1 y + C2 y2 . On the other hand, when we choose C1 = C2 = 0,
we …nd that yp is one of the solutions of the di¤erential equation (2.1) itself.
CHAPTER 2. SECOND-ORDER LINEAR DIFFERENTIAL EQUATIONS29

Finally, elementary Linear Algebra allows us to replace this speci…c yp (x)


by any other solution of (2.1), and to replace this speci…c y2 (x) by any other
solution of (2.2), as long as it is independent of y1 .

The function yp (x) in (2.4) is usually said to be a particular solution of


the di¤erential equation (2.1).

Remark 1 (Important) In di¤erential equations, the form of the general


solution yG:S: = yp +C1 y1 +C2 y2 (in which C1 and C2 are arbitrary constants)
is valid for second-order linear di¤erential equations only.

2.2 Linear Di¤erential Equations with Con-


stant Coe¢ cients
In this section and the next one, we consider only second-order linear di¤er-
ential equations with constant coe¢ cients; that is di¤erential equations

d2 y dy
a 2 + b + cy = Q (x) : (2.7)
dx dx
whose three coe¢ cients a; b; c are constants.

2.2.1 The Homogeneous Solution yh (x)


The homogeneous solution (the cpmplementary function) of (2.7) yh (x) =
C1 y + C2 y2 comprises all solutions of the corresponding homogeneous equa-
tion
d2 y dy
a 2 + b + cy = 0: (2.8)
dx dx
Two independent solutions y1 and y2 of (2.8) are obtained by examining the
following auxiliary equation:

am2 + bm + c = 0: (2.9)

Its two roots are p


b b2 4ac
m1;2 = : (2.10)
2a
CHAPTER 2. SECOND-ORDER LINEAR DIFFERENTIAL EQUATIONS30

Theorem 4 We take yh as follows (C1 ; C2 are arbitrary constants):


Case 1: When m1 ; m2 are distinct and real, take
yh (x) = C1 em1 x + C2 em2 x :
Case 2: When m1 = m2 (hence, real), take
yh (x) = C1 em1 x + C2 xem1 x = (C1 + C2 x) em1 x :
Case 3: When m1;2 = i and 6= 0 (complex conjugates), take
x x x
yh (x) = C1 e sin x + C2 e cos x = e (C1 sin x + C2 cos x) :
Proof. As m1 ; m2 are the two roots of (2.9), then
0 = m2 + ab m + ac = (m m1 ) (m m2 ) = m2 (m1 + m2 ) m + m1 m2 :
So, (2.8) becomes the following d.e. in the unknown function z (x) :
d2 y b dy c d2 y dy
0 = dx 2 + a dx + a y = dx2 (m1 + m2 ) dx + m1 m2 y:
In all three cases, we perform the following substitution in the given
homogeneous d.e. (2.8): y (x) = e x z (x) , where = m1 +m
2
2
.
dy x dz x dz m1 +m2
Then dx
=e dx
+ z =e dx
+ 2
z ; and
d2 y d2 z 2
dx2
=e x
dx2
+2 dz
dx
+ 2
z =e x d z
dx2
+ (m1 dz
+ m2 ) dx + 14 (m1 + m2 )2 z :
)0=e x d2 z
dx2
dz
+ 0 dx + 1
4
(m1 + m2 )2 1
2
(m1 + m2 )2 + m1 m2 z :
d2 z (m1 +m2 )2
We divide by e x ; to get: dx2
+ m1 m2 4
z = 0; that is,

d2 z (m1 m2 )2
= z:: (2.11)
dx2 4
Case 1: As m1 ; m2 are distinct and real, then m1 m2 is a nonzero real
number. It is direct to verify that the solution of d.e. (2.11) becomes:
zh = C1 e(m1 m2 )x=2 + C21 e (m1 m2 )x=2 ; (using (2.3) of Theorem 3).
) yh (x) = e x zh = e(m1 +m2 )x=2 C1 e(m1 m2 )x=2 + C21 e (m1 m2 )x=2 :
Hence, yh (x) = e x zh = C1 em1 x + C2 em2 x :
d z 2
Case 2: As m1 = m2 , then equation (2.11) becomes here: dx 2 = 0:

Whose solution is: zh = C1 + C2 x: Also, e becomes e = em1 x :


x x

Therefore, yh (x) = e x zh = em1 x (C1 + C2 x) :


Case 3: Since b2 4ac is negative then
m1;2 = i , where 2 = m1 + m2 ; as before,
2
and 2i = m1 m2 ; that is, (m1 4m2 ) = 2
:
d z 2 2
Consequently, (2.11) becomes in this case: dx 2 + z = 0:
It is direct to verify that the general solution of this last d.e. is
zh = C1 sin x+C2 cos x: ) yh (x) = e x zh = e x (C1 sin x + C2 cos x) :
CHAPTER 2. SECOND-ORDER LINEAR DIFFERENTIAL EQUATIONS31

2.3 A Particular Solution - Method of Unde-


termined Coe¢ cients
The particular solution yp (x) is one (any one) solution of the equation (2.7)
itself. One easy choice of yp (x) can be carried out through the method of
undetermined coe¢ cients. This consists of the following three steps:
Step 1: Put Q (x) (the right-hand side of (2.7)) in a form W (x) that
is stable under di¤erentiation (cannot be altered by di¤erentiation). Then
take yp (x) in that same form W (x), but with undetermined coe¢ cients.
Examples of those forms are:
W (x) = A (constants),
W (x) = Aekx (exponential functions)
W (x) = Ax + B (polynomials of degree 1),
W (x) = Ax2 + Bx + C (polynomials of degree 2),
W (x) = Ax3 + Bx2 + Cx + D (polynomials of degree 3),
W (x) = A sin !x + B cos !x;
as well as multiples of the forms above, such as W (x) = (Ax + B) ekx .
Step 2: If the form W (x), suggested in Step 1, has terms in common
with yh (x), then modify it by multiplying the whole form by the variable x.
Repeat this step, if necessary, until the …nal form of yp (x) has no terms in
common with yh (x).
Step 3: Substitute the above suggested yp (x) for the unknown func-
tion y (x), in the D.E. (2.7). Use the resulting equation to compute all the
undetermined coe¢ cients A; B; in that suggestion.

We can see why the modi…cation carried out in Step 2 works, in tfollowing
simple case. Suppose the form W (x), suggested in Step 1, is contained within
yh (x), but xW (x) is not. Take yp (x) = xW (x) and substitute it for y in
(2.7). Then
2
q = a ddxy2p + b dy
dx
p
+ cyp = a (xW00 + 2W0 ) + b (xW0 + W) + cxW
= x (aW00 + bW0 + cW) + (2aW0 + bW) = 0 + (2aW0 + bW)
(because W (x) is a solution of (2.8)).
) 2aW0 + bW = q:
As the three terms 2aW0 , bW and q lie in one form, then we can use
this equation to compute all undetermined coe¢ cients in yp (x) = xW (x).
CHAPTER 2. SECOND-ORDER LINEAR DIFFERENTIAL EQUATIONS32

2.3.1 Lecture Examples


Solve the following second-order linear di¤erential equations, us-
ing the Method of Undetermined Coe¢ cients:
Problem 2.3.1 y 00 + 4y 0 12y = 18e3x :

Auxiliary Equation: m2 + 4m 12 = 0
) (m 2) (m + 6) = 0 ) m1 = 2; m2 = 6:
) Homogeneous solution is: yh = C1 e2x + C2 e 6x
:
) Particular solution takes the form: yp = Ae3x
) yp0 = 3Ae3x ; ) yp00 = 9Ae3x :
) yp + 4yp 12yp = 9Ae3x = 18e3x
00 0
)A= 2
General Solution is yG:S: = yp + yh = 2e3x + C1 e2x + C2 e 6x
.

Problem 2.3.2 y 00 + 5y 0 14y = 27e2x :

Auxiliary Equation: m2 + 5m 14 = 0
) (m + 7) (m 2) = 0 ) m1 = 7; m2 = 2:
) Homogeneous solution is: yh = C1 e 7x
+ C2 e2x :

) Particular solution takes the form yp = Axe2x :


) yp = 2Axe + Ae ;
0 2x 2x

) yp00 = 4Axe2x + 2Ae2x + 2Ae2x = 4Axe2x + 4Ae2x


) yp00 + 5yp0 14yp = Ae2x (4x + 4 + 10x + 5 14x) = 9Ae2x = 27e2x :
) A = 3: ) yp = 3xe2x :
General Solution is yG:S: = yp + yh = 3xe2x + C1 e 7x + C2 e2x :

Problem 2.3.3 y 00 + 4y 0 + 3y = 10 cos x

Auxiliary Equation: m2 + 4m + 3 = 0
) (m + 3) (m + 1) = 0 ) m1 = 3; m2 = 1:
) Homogeneous solution is: yh = C1 e 3x
+ C2 e x :

) Particular solution takes the form: yp = A sin x + B cos x


) yp = A cos x B sin x
0

) yp00 = A sin x B cos x


) yp00 + 4yp0 + 3yp = A sin x B cos x + 4A cos x 4B sin x + 3A sin x +
3B cos x
CHAPTER 2. SECOND-ORDER LINEAR DIFFERENTIAL EQUATIONS33

= (2A 4B) sin x + (4A + 2B) cos x = 0 sin x + 10 cos x


) 2A 4B = 0; 4A + 2B = 10: ) A = 2B
) 10B = 10 )B=1 ) A = 2:
General Solution is yG:S: = yp + yh = 2 sin x + cos x + C1 e 3x + C2 e x
.

Problem 2.3.4 y 00 + 4y = 8x2 :

Auxiliary Equation:
p mp2 + 4 = 0 p
) m1;2 = b 2ab 4ac = 0 20 16 = 6 2 144 = 24i = 2i =
2
i
) =0 and =2
) yh = e x (C1 sin ( x) + C2 cos ( x)) = C1 sin (2x) + C2 cos (2x) :
) Particular solution takes the form yp = Ax2 + Bx + C:
) yp0 = 2Ax + B; yp00 = 2A:
) yp + 4yp = 4Ax + 4Bx + (2A + 4C) = 8x2 + 0x + 0
00 2

) 4A = 8; 4B = 0; 2A + 4C = 0
) A = 2; B = 0; C= 1 ) yp = 2x2 1:
) yG:S: = yp + yh = 2x2 1 + C1 sin (2x) + C2 cos (2x) :

Problem 2.3.5 y 00 + y 0 6y = (14x 5) e4x

Auxiliary Equation: m2 + m 6 = 0
) (m + 3) (m 2) = 0 ) m1 = 3; m2 = 2:
) Homogeneous solution is: yh = C1 e 3x
+ C2 e2x :
) Particular solution takes the form yp = (Ax + B) e4x :
) yp0 = (4Ax + A + 4B) e4x ; yp00 = (16Ax + 4A + 16B + 4A) e4x :
) yp + yp 6yp = (16Ax + 8A + 16B + 4Ax + A + 4B 6Ax 6B) e4x
00 0

= 14Axe4x + (9A + 14B) e4x = 14xe4x 5e4x


) 14A = 14; ) A = 1. Also, 9A + 14B = 5 )B= 1
) yG:S: = yp + yh = (x 1) e4x + C1 e 3x
+ C2 e2x .
CHAPTER 2. SECOND-ORDER LINEAR DIFFERENTIAL EQUATIONS34

2.3.2 Classroom Exercises


Problem 2.3.6 y 00 + 4y = 8 sin (2x) + 12 cos (2x)

Homogeneous solution yh = C1 sin (2x) + C2 cos (2x) :(Problem (2.3.4))


) Particular solution takes the form yp = Ax sin (2x) + Bx cos (2x) :
) yp0 = 2Ax cos (2x) 2Bx sin (2x) + A sin (2x) + B cos (2x) ;
yp00 = 4Ax sin (2x) 4Bx cos (2x)+2A cos (2x) 2B sin (2x)+2A cos (2x)
2B sin (2x)
= ( 4Ax 4B) sin (2x) + ( 4Bx + 4A) cos (2x)
) yp +4yp = ( 4Ax 4B) sin (2x)+( 4Bx + 4A) cos (2x)+4Ax sin (2x)+
00

4Bx cos (2x)


= 4B sin (2x) + 4A cos (2x) = 8 sin (2x) + 12 cos (2x)
) A = 3; B=2 ) yp = 3x sin (2x) + 2x cos (2x) :
General Solution is yG:S: = yp + yh = (3x + C1 ) sin (2x) + (2x + C2 ) cos (2x) :

Problem 2.3.7 y 00 3y 0 + 2y = 2e2x

Auxiliary Equation: m2 3m + 2 = 0
) (m 1) (m 2) = 0 ) m1 = 1; m2 = 2:
) Homogeneous solution is: x
yh = C1 e + C2 e2x :

) Particular solution takes the form yp = Axe2x :


) yp0 = 2Axe2x + Ae2x ;
) yp00 = 4Axe2x + 2Ae2x + 2Ae2x = 4Axe2x + 4Ae2x
) yp00 3yp0 + 2yp = Ae2x (4x + 4 6x 3 + 2x) = Ae2x = 2e2x :
) A = 2: ) yp = 2xe2x :
General Solution is yG:S: = yp + yh = 2xe2x + C1 ex + C2 e2x :

Problem 2.3.8 y 00 + 4y 0 + 4y = 6e 2x
:

Auxiliary Equation: m2 + 4m + 4 = 0
) (m + 2)2 = 0 ) m1 = m2 = 2:
) Homogeneous solution is: yh = C1 e 2x
+ C2 xe 2x
:

) Particular solution takes the form yp = Ax2 e 2x :


) yp0 = 2Ax2 e 2x + 2Axe 2x = Ae 2x ( 2x2 + 2x) ;
) yp00 = 4Ax2 e 2x 4Axe 2x 4Axe 2x + 2Ae 2x = Ae 2x
(4x2 8x + 2)
CHAPTER 2. SECOND-ORDER LINEAR DIFFERENTIAL EQUATIONS35

) yp00 + 4yp0 + 4yp = Ae 2x


(4x2 8x + 2 8x2 + 8x + 4x2 ) = 2Ae 2x
=
2x
6e
) A = 3: ) yp = 3x2 e 2x
:
General Solution is yG:S: = yp + yh = 3x2 e 2x
+ (C1 + C2 x) e 2x
:

Problem 2.3.9 y 00 + 6y 0 + 45y = 81 sin (6x) + 18 cos (6x)

Auxiliary Equation:
p m2p+ 6m + 45 =p0
) m1;2 = b b2 4ac
2a
= 6 236 180 = 6 2 144 = 6 2 12i
) m1;2 = 3 6i = i ) = 3 and =6
) Homogeneous solution is:
yh = e x (C1 sin ( x) + C2 cos ( x)) = e 3x (C1 sin (6x) + C2 cos (6x)) :
) Particular solution takes the form yp = A sin (6x) + B cos (6x) :
) yp = 6A cos (6x) 6B sin (6x) ;
0 00
yp = 36A sin (6x) 36B cos (6x) :
) yp +6yp +45yp = ( 36A 36B + 45A) sin (6x)+( 36B + 36A + 45B) cos (6x)
00 0

= (9A 36B) sin (6x)+(36A + 9B) cos (6x) = 81 sin (6x)+18 cos (6x)
) 9A 36B = 81; 36A + 9B = 18 ) A = 1. and B = 2
) yp = sin (6x) 2 cos (6x) : General Solution is
yG:S: = yp + yh = sin (6x) 2 cos (6x) + e 3x (C1 sin (6x) + C2 cos (6x)) :

Problem 2.3.10 y 00 6y 0 + 9y = (4 + 4x) ex ; y (0) = 1; y 0 (0) = 1

Auxiliary Equation: m2 6m + 9 = 0
) (m 3)2 = 0 ) m1 = m2 = 3:
) Homogeneous solution is: yh = (C1 + C2 x) e3x :

) Particular solution takes the form: yp = (A + Bx) ex


) yp0 = (A + B + Bx) ex
) yp00 = (A + 2B + Bx) ex
) yp00 6yp0 + 9yp = (A + 2B + Bx 6A 6B 6Bx + 9A + 9Bx) ex
= (4A 4B) ex + 4Bxex = 4ex + 4xex
) 4A 4B = 4; 4B = 4: )B=1 ) A = 2:
General Solution is x
yG:S: = yp + yh = (2 + x) e + (C1 + C2 x) e3x .
y (0) = 2 + C1 = 1 ) C1 = 3
y 0 (x) = (3 + x) ex + (2C1 + C2 + 2C2 x) e3x
y 0 (0) = 3 + 2C1 + C2 = 1 ) C4 = 4
) Solution is y (x) = (2 + x) ex + (4x 3) e3x
CHAPTER 2. SECOND-ORDER LINEAR DIFFERENTIAL EQUATIONS36

Problem 2.3.11 y 00 4y = 16e2x

Auxiliary Equation: m2 4 = 0
) (m 2) (m + 2) = 0 ) m1 = 2; m2 = 2:
) Homogeneous solution is: yh = C1 e + C2 e 2x :
2x

) Particular solution takes the form: yp = Axe2x


) yp0 = (2x + 1) Ae2x ) yp00 = (4x + 4) Ae2x
) yp 4yp = (4x + 4 4x) Ae2x = 4Ae2x = 16e2x :
00

) A = 4: ) yp = 4xe2x :
General Solution is yG:S: = yp + yh = (4x + C1 ) e2x + C2 e 2x
.

Problem 2.3.12 y 00 + 5y 0 + 6y = 2e 3x

Auxiliary Equation: m2 + 5m + 6 = 0
) (m + 3) (m + 2) = 0 ) m1 = 3; m2 = 2:
) Homogeneous solution is: yh = C1 e 3x
+ C2 e 2x :

) Particular solution takes the form yp = Axe 3x :


) yp = 3Axe
0 3x
+ Ae ;3x

) yp = 9Axe
00 3x
3Ae 3x 3Ae 3x = 9Axe 3x 6Ae 3x
) yp + 5yp + 6yp = Ae 3x (9x 6 15x + 5 + 6x) = Ae
00 0 3x
= 2e 3x
:
) A = 2: ) yp = 2xe 3x :
3x 3x 2x
General Solution is yG:S: = yp + yh = 2xe + C1 e + C2 e :

Problem 2.3.13 y 00 4y 0 + 13y = 50xe3x

Auxiliary Equation:
p mp2 4m + 13 p = 0
) m1;2 = b 2
b 4ac 4 16 52 4 36
2a
= 2
= 2
= 4 26i
) m1;2 = 2 3i = i ) Homogeneous solution is:
) yh = e (C1 sin ( x) + C2 cos ( x)) = e2x (C1 sin (3x) + C2 cos (3x)) :
x

) Particular solution takes the form yp = (Ax + B) e3x :


) yp0 = (3Ax + A + 3B) e3x ; yp00 = (9Ax + 6A + 9B) e3x :
) yp 4yp +13yp = (9Ax + 6A + 9B 12Ax 4A 12B + 13Ax + 13B) e3x
00 0

= 10Axe3x + (2A + 10B) e3x = 50xe3x + 0e3x


) 10A = 50; ) A = 5. Also, 2A + 10B = 0 )B= 1
) yp = (5x 1) e3x )General Solution is
yG:S: = yp + yh = (5x 1) e3x + e2x (C1 sin (3x) + C2 cos (3x)) :
CHAPTER 2. SECOND-ORDER LINEAR DIFFERENTIAL EQUATIONS37

Problem 2.3.14 y 00 8y 0 + 25y = 75x2 23x 52

Auxiliary Equation:
p mp2 8m + 25p= 0
) m1;2 = b b2 4ac
2a
= 8 64 100
2
= 8 2 36 = 8 26i
) m1;2 = 4 3i = i ) Homogeneous solution is:
) yh = e (C1 sin ( x) + C2 cos ( x)) = e 4x (C1 sin (3x) + C2 cos (3x)) :
x

) Particular solution takes the form yp = Ax2 + Bx + C:


) yp0 = 2Ax + B; yp00 = 2A:
) yp 8yp + 25yp = 2A 16Ax 8B + 25Ax2 + 25Bx + 25C
00 0

= 25Ax2 + ( 16A + 25B) x + (2A 8B + 25C) = 75x2 23x 52


) 25A = 75; ) A = 3;
16A + 25B = 23 ) 25B = 16A 23 = 48 23 = 25
) B = 1;
2A 8B + 25C = 52 ) 25C = 2A + 8B 52 = 6 + 8 52 = 50
)C= 2 General Solution is
yG:S: = yp + yh = 3x2 + x 2+e 4x
(C1 sin (3x) + C2 cos (3x)) :

Problem 2.3.15 y 00 2y 0 = 6x2 6x 8

Auxiliary Equation: m2 2m = 0
) m (m 2) = 0 ) m1 = 0; m2 = 2:
) Homogeneous solution is: yh = C1 + C2 e2x :
) Particular solution takes the form: yp = x (Ax2 + Bx + C) = Ax3 +
Bx2 + Cx
) yp0 = 3Ax2 + 2Bx + C
) yp00 = 6Ax + 2B
) yp00 2yp0 = 6Ax + 2B 2 (3Ax2 + 2Bx + C)
= 6Ax2 + (6A 4B) x + (2B 2C) = 6x2 6x 8
) A = 1; 6A 4B = 6; ) B = 3;
2B 2C = 8 ) C = 7:
General Solution is yG:S: = yp + yh = x3 + 3x2 + 7x + C1 + C2 e2x .
CHAPTER 2. SECOND-ORDER LINEAR DIFFERENTIAL EQUATIONS38

2.3.3 Revision Problems


Problem 2.3.16 y 00 3y 0 + 2y = 0

Auxiliary Equation: m2 3m + 2 = 0
) (m 1) (m 2) = 0 ) m1 = 1; m2 = 2:
) General solution is: x
yh = C1 e + C2 e2x :

Problem 2.3.17 y 00 6y 0 + 9y = 0

Auxiliary Equation: m2 6m + 9 = 0
) (m 3) = 0
2
) m1 = m2 = 3:
) General solution is: yh = (C1 + C2 x) e3x :

Problem 2.3.18 y 00 + 8y 0 + 25y = 0

Auxiliary Equation:
p m2p+ 8m + 25 =p0
) m1;2 = b b2 4ac
2a
= 8 264 100 = 8 2 36 = 82 6i
) m1;2 = 4 3i = i ) = 4 and =3
x
and yh = e (C1 sin ( x) + C2 cos ( x)) :
) General solution is: yh = e 4x (C1 sin (3x) + C2 cos (3x)) :

Problem 2.3.19 y 00 + y = 6 sin (x) 2 cos (x)

Auxiliary Equation:
p m2 + 1 = 0
) m1;2 = 1= i= i ) Homogeneous solution is:
) yh = e (C1 sin ( x) + C2 cos ( x)) = C1 sin (x) + C2 cos (x) :
x

) Particular solution takes the form yp = Ax sin (x) + Bx cos (x) :


) yp = Ax cos (x) Bx sin (x) + A sin (x) + B cos (x) ;
0

yp00 = Ax sin (x) Bx cos (x)+A cos (x) B sin (x)+A cos (x) B sin (x)
= ( Ax 2B) sin (x) + ( Bx + 2A) cos (x)
) yp00 + yp = 2B sin (x) + 2A cos (x) = 6 sin (x) 2 cos x
) A = 1; B= 3 ) yp = x sin (x) 3x cos (x) :
General Solution is yG:S: = yp +yh = ( x + C1 ) sin x + ( 3x + C2 ) cos x:
CHAPTER 2. SECOND-ORDER LINEAR DIFFERENTIAL EQUATIONS39

Problem 2.3.20 y 00 + 2y 0 15y = 8e3x

Auxiliary Equation: m2 + 2m 15 = 0
) (m 3) (m + 5) = 0 ) m1 = 3; m2 = 5:
) Homogeneous solution is: yh = C1 e3x + C2 e 5x :
) Particular solution takes the form: yp = Axe3x
) yp = Ae (1 + 3x)
0 3x
) yp = Ae (6 + 9x)
00 3x

) yp + 2yp 15yp = Ae (6 + 9x + 2 + 6x 15x) = 8Ae3x = 8e3x


00 0 3x

) A = 1: ) yp = xe3x :
General Solution is yG:S: = yp + yh = xe3x + C1 e3x + C2 e 5x .

Problem 2.3.21 y 00 + 2y 0 15y = 99e6x

yh = C1 e3x + C2 e 5x
(Problem(2.3.20)):

) Particular solution takes the form: yp = Ae6x


) yp0 = 6Ae6x ) yp00 = 36Ae6x
) yp + 2yp 15yp = Ae6x (36 + 12 15) = 33Ae6x = 99e6x
00 0

) A = 3: ) yp = 3e6x :
General Solution is yG:S: = yp + yh = 3e6x + C1 e3x + C2 e 5x
.

Problem 2.3.22 y 00 7y 0 + 12y = 12x2 14x + 26:

Final Answer: y = x2 + 2 + C1 e4x + C2 e3x :

Problem 2.3.23 y 00 + 6y 0 27y = 81x2 63x + 27:

Final Answer: y= 1+x 3x2 + C1 e3x + C2 e 9x


:

Problem 2.3.24 y 00 4y = 16x3 + 36x2 + 180x + 80.


49
Final Answer: y= 2
39x 9x2 + 4x3 + C1 e2x + C2 e 2x
:

Problem 2.3.25 y 00 + 6y 0 16y = 30e2x :

Final Answer: y = 3xe2x + C1 e2x + C2 e 8x


:
CHAPTER 2. SECOND-ORDER LINEAR DIFFERENTIAL EQUATIONS40

Problem 2.3.26 y 00 + 4y 0 + 5y = 10ex :

Final Answer: y = e x + C1 e 2x
cos x + C2 e 2x
sin x: cos x:

Problem 2.3.27 y 00 6y 0 + 5y = 4ex :

Final Answer: y = 41 ex + xex + C1 ex + C2 e5x :

Problem 2.3.28 y 00 5y 0 + 4y = 20 sin 2x:

Final Answer: y = 2 cos 2x + C1 ex + C2 e4x :

Problem 2.3.29 y 00 + 2y 0 3y = 130 sin x + 10 cos x:

Final Answer: y = 11 cos x + 27 sin x + C1 ex + C2 e 3x


:

Problem 2.3.30 y 00 + 2y 0 + 5y = 3 sin 2x + 12 cos 2x:


x
Final Answer: y = 3 sin 2x + C1 (cos 2x) e + C2 (sin 2x) e x :

Problem 2.3.31 y 00 = 220x9 112x6 + 12x2 :

Final Answer: y = 2x11 2x8 + x4 + C1 x + C2 :

Problem R 2.3.32 In an RLC-circuit, the current I (t) is governed by L dI


dt
+
1
RI + C I dt = E (t). Di¤erentiating, we get the di¤erential equation:
2
L ddt2I + R dI
dt
+ C1 I = dE(t)
dt
:
1
Find I (t) in the case L = 1, R = 8, C = 25 , E = 48 sin 2t 63 cos 2t,
and initially I = I 0 = 0.
4t
Final Answer: I (t) = 4e sin 3t + 6 sin 2t:

Problem 2.3.33 Solve the following second-order linear di¤erential equa-


tions

1. y 00 + y = 6 cos x:
CHAPTER 2. SECOND-ORDER LINEAR DIFFERENTIAL EQUATIONS41

2. y 00 y = 6 cos x:

3. y 00 + y 0 = 6 cos x:

4. y 00 + 8y0 + 25y = 80 cos 5x:

5. y 00 2y 0 + y = 8xe3x :

6. y 00 2y 0 + y = 2 cos x :

7. y 00 14y 0 + 45y = 4e5x :

8. y 00 + 6y 0 + 13y = 10e2x + 29xe2x :

9. y 00 2y 0 + 10y = 50xe2x :

10. y 00 4y 0 + 4y = xe3x :

11. y 00 9y = 6e 3x
:

12. y 00 + 9y = 6e 3x
:

13. y 00 6y 0 + 5y = 8ex :

14. y 00 + 6y 0 + 5y = 8ex :

15. y 00 y0 6y = 15e 2x
:

16. y 00 + 2y 0 8y = 18e2x :
CHAPTER 2. SECOND-ORDER LINEAR DIFFERENTIAL EQUATIONS42

2.4 Order Reduction of Linear Di¤erential Equa-


tions
This is a general method for solving an nth-order linear di¤erential equation,
through solving another (n-1) th-order linear di¤erential equation. Although
the steps of this method are quite simple, their performance is usually rather
long, and requires the computation of many integrals, which may or may not
be amenable!
We shall, therefore, re…ne this method in the next subsection to an easier
one, called the Method of Variation of Parameters.
Nevertheless, in the rest of this subsection, we present the theoretical
basis of the Method of Order Reduction, in the case of a given second order
linear di¤erential equation:
d2 y dy
a (x) 2
+ b (x) + c (x) y = Q (x) ; (2.12)
dx dx
whose general solution yp (x) is sought after.
This method comes with the stipulation of knowing beforehand one nonzero
solution y1 (x) of the corresponding homogeneous equation
d2 y dy
a (x) 2
+ b (x) + c (x) y = 0: (2.13)
dx dx
The steps of the Method of Order Reduction are:
Step 1: We introduce into the given problem (2.12) the following new
unknown function
y (x)
U (x) = ;
y1 (x)
that is,
y (x) = y1 (x) :U (x) : (2.14)
Using this substitution, we transform equation (2.12) into a new di¤er-
ential equation in the unknown function U (x).

Step 2: That new equation will take the form U " + p (x) U 0 = q (x),
for certain functions p (x) and q (x); that is
0
(U 0 ) + p (x) U 0 = q (x) : (2.15)

In consequence, we have realized a so called reduction of order of a linear


di¤erential equation, from order 2 of equation (2.12) (in the unknown y (x))
to order 1 of equation (2.15) (in the unknown U 0 (x)).
CHAPTER 2. SECOND-ORDER LINEAR DIFFERENTIAL EQUATIONS43

Proof. We have y = y1 :U ) y 0 = y1 :U 0 + y10 :U


) y" = y1 :U " + 2y10 :U 0 + y100 :U:
When we substitute in (2.12), we obtain
ay1 :U " + (2ay10 + by1 ) :U 0 + (ay100 + by10 + cy1 ) :U = Q (x) :
But, y1 (x) is chosen such that ay100 + by10 + cy1 = 0.
Substituting in the equation above, we get
ay1 :U " + (2ay10 + by1 ) :U 0 = Q (x) ;
y0
that is, U " + 2 y11 + ab :U 0 = Q(x) ay1
; as claimed.

Step 3: We solve the …rst-order linear di¤erential equation (2.15) in


the usual manner, and get
Z
0 1
U = C2 + (x) q (x) dx ; (2.16)
(x)
R
p(x) dx
where (x) = e :

Step 4: We then obtain U by intgrating U 0;


Z
U = C1 + U 0 dx: (2.17)

Step 5: Finally, we get the required solution y (x) of the original equa-
tion (2.12), as y (x) = y1 (x) :U (x) .

Remark 2 As can be seen from the above exposition, the steps comprising
the Method Order Reduction are very easy to memorize. Which is the greatest
merit of this method.
Remark 3 The only two formulae we need to memorize, in the course of
applying this method, are the substitution y (x) = y1 :U (2.14) and the solution
(2.16) of …rst-order linear di¤erential equations. Naturally, we must know
also how to perform the …ve steps of the method. But, we need not memorize
any further formulae.
Remark 4 If we retain the two arbitrary constants C1 and C2 in (2.17) and
(2.16), respectively, we do come out with the general solution yG:S: (x) of
the original equation (2.12). But, if we are only interested in computing a
particular solution yp (x) of (2.12), then we can ignore these two constants
(i.e., take C1 = C2 = 0).
Remark 5 This method is an exact replica of our proof of Theorem 3 in
Section 2.1, above.
CHAPTER 2. SECOND-ORDER LINEAR DIFFERENTIAL EQUATIONS44

2.5 Method of Variation of Parameters


2.5.1 Describing The Method of Variation of Parame-
ters
This Method of Variation of Parameters is a general, and much easier,
method for computing a particular solution yp (x) for a second-order linear
di¤erential equation (in an unknown function y (x)):

d2 y dy
2
+ b (x) + c (x) y = Q (x) ; (2.18)
dx dx
d y 2
in which the coe¢ cient of dx2 is 1 (If otherwise, divide by its given coef-

…cient.)
This time, we need a prior knowledge of the whole of the homogeneous
solution (the complementary function) of (2.18)

yh (x) = C1 y1 (x) + C2 y2 (x) ; (2.19)

which contains all solutions of the homogeneous equation:

d2 yh dyh
2
+b + cyh = 0: (2.20)
dx dx
We put the required particular solution yp (x) of (2.18) in the form:

yp (x) = V1 (x) y1 (x) + V2 (x) y2 (x) ; (2.21)

in which V1 (x) and V2 (x) are two unknown functions, that we need to de-
termine.
We are going to substitute this yp (x) for y (x) in equation (2.18). This will
result in one di¤erential equation only in the two unknown functions V1 (x)
and V2 (x). This is consistent with our previous knowledge that equation
(2.18) has an in…nite number of solutions, any one of them can serve as
the required yp (x) : In order to come out with one solution for yp (x) ; we
impose another di¤erential equation on the two unknowns V1 and V2 . We
choose another equation, which also facilitates their determination greatly,
without falling in a contradiction with the equation derived from (2.18). That
expedient equation is:
y1 V10 + y2 V20 = 0: (2.22)
As yp = y1 V1 + y2 V2 then
yp0 = y10 V1 + y20 V2 + y1 V10 + y2 V20 = y10 V1 + y20 V2 + 0,
CHAPTER 2. SECOND-ORDER LINEAR DIFFERENTIAL EQUATIONS45

using (2.22), and


yp00 = y100 V1 + y200 V2 + y10 V10 + y20 V20 :
We next substitute yp (x) for y (x) in equation (2.18) and get:
) Q (x) = yp00 + byp0 + cyp = y100 V1 + y200 V2 + y10 V10 + y20 V20
+by10 V1 + by20 V2 + cy1 V1 + cy2 V2
= y100 V1 + by10 V1 + cy1 V1 + y200 V2 + by20 V2 + cy2 V2 + y10 V10 + y20 V20
= (y100 + by10 + cy1 ) V1 + (y200 + by20 + cy2 ) V2 + y10 V10 + y20 V20
= 0V1 + 0V2 + y10 V10 + y20 V20 because y1 ; y2 satisfy (2.20).
So, our second di¤erential equation in V1 ; V2 turns out to be:

y10 V10 + y20 V20 = Q (x) : (2.23)

We were expecting two di¤erential equations in V1 ; V2 : Instead, we got


two algebraic linear equations (2.22) and (2.23) in V10 ; V20 ; which are much
easier to solve.
CHAPTER 2. SECOND-ORDER LINEAR DIFFERENTIAL EQUATIONS46

2.5.2 Steps of the Method of Variation of Parameters:


We want to solve a second-order linear di¤erential equation
d2 y dy
2
+ b (x) + c (x) y = Q (x) ; (2.24)
dx dx
Step 1: Find the homogeneous solution yh (x) (the solution of the
homogeneous equation (2.20)):
yh (x) = C1 y1 (x) + C2 y2 (x) : (2.25)
Step 2: Write the particular solution yp (x) as:
yp (x) = V1 y1 + V2 y2 ;
whereby V1 (x) and V2 (x) are two unknown functions, to be determined.
Step 3: Write down the following two equations:
y1 V10 + y2 V20 = 0: (2.26)
y10 V10 + y20 V20 = Q (x) : (2.27)
Step 4: Compute the following determinant (the Wronskian of y1 ; y2 ):
y1 y2
W = : (2.28)
y10 y20
Note that the independence of the two solutions y1 (x) ; y2 (x) of (2.20)
ensures that their Wronskian W is a nonzero function. Which means that
the two algebraic equations (2.26) and (2.27) are mutually consistent (non-
contradictory), as we had hoped.
Step 5: Solve these two algebraic equations in the unknown functions
V1 and V20 : using Cramer’s Rule, which yields the solution as:
0

0 y2 y1 0
y2 Q (x) Q (x) y20 y1 Q (x) 0
y1 Q (x)
V10 = = ; V20 = = : (2.29)
W y1 y2 W y1 y2
y10 y20 y10 y20
Step 6: Obtain the two functions V1 (x) and V2 (x), by integrating V10 (x)
and V20 (x), respectively.
Step 7: Obtain a particular solution yp (x) as:
yp (x) = V1 y1 + V2 y2 . (2.30)
Step 8: Then write the general solution yG:S: (x) of (2.24) as:
yG:S: = yp + yh .
CHAPTER 2. SECOND-ORDER LINEAR DIFFERENTIAL EQUATIONS47

2.5.3 Lecture Examples


Problem 2.5.1 Verify that the function yh (x) = C1 sec x + C2 tan x is the
homogeneous solution of the following linear di¤erential equation:
y 00 (tan x) y 0 (sec2 x) y = 12 sin x cos x.
Then use this yh (x) to …nd the its general solution, by means of the
Method of Variation of Parameters.

The student should verify that both (independent) functions:


We take y1 = sec x and y2 = tan x
are solutions of the corresponding homogeneous d.e..
Their Wronskian is:
y1 y2 sec x tan x
W = 0 0 = = sec3 x sec x tan2 x
y1 y2 sec x tan x sec2 x
) W = sec x (sec2 x tan2 x) = sec x:
We solve the two equations, in which Q (x) = 12 sin x cos x:
y1 V10 + y2 V20 = 0;
y10 V10 + y20 V20 = Q (x)
y2 Q
V10 = W
= 12 tan x sin x cos x
sec x
= 12 sin2 x cos x
Z Z
V1 = V10 dx = 12 sin2 x cos x dx = 4 sin3 x:
y1 Q sec xQ
V20 = W
= = Q = 12 sin x cos x:
sec x
Z Z
) V2 = 0
V1 dx = 12 sin x cos x dx = 6 cos2 x
Consequently,
yp (x) = V1 e x + V2 e 2x = 4 sin3 x sec x + 6 cos2 x tan x
=4 sin2 x tan x + 4 cos2 x tan x + 2 cos2 x tan x
) yp (x) = 4 tan x + 2 sin x cos x: Finally,

) yG:S: = yp + yh = 2 sin x cos x + C1 sec x + K2 tan x:

2.5.4 Classroom Exercises


Problem 2.5.2 Verify that the function yh (x) = C1 sec2 x + C2 sec x tan x
is the homogeneous solution of the following linear di¤erential equation:
y 00 3 (tan x) y 0 2y = sec3 x.
Then use this yh (x) to …nd the its general solution, by means of the
Method of Variation of Parameters.
CHAPTER 2. SECOND-ORDER LINEAR DIFFERENTIAL EQUATIONS48

The student should verify that both (independent) functions:


y1 = sec2 x and y2 = sec x tan x
are solutions of the corresponding homogeneous d.e..
y1 y2 sec2 x sec x tan x
Their Wronskian is: W = 0 0 =
y1 y2 2 sec2 x tan x sec x tan2 x + sec3 x
= sec3 x (tan2 x + sec2 x 2 tan2 x) = sec3 x (sec2 x tan2 x)
) W = sec3 x:
We solve the two equations, in which Q (x) = sec3 x :

y1 V10 + y2 V20 = 0;
y10 V10 + y20 V20 = Q (x)
y2 Q
V10 = W
= sec x tan x
Z Z
0
V1 = V1 dx = sec x tan x dx = sec x:
y1 Q
V20 = W
sec2 x:
=
Z Z
) V2 = V1 dx =
0
sec2 x dx = tan x
Consequently,
yp (x) = V1 e x + V2 e 2x = sec3 x sec x tan2 x
) yp (x) = sec x (sec2 x tan2 x) = sec x: Finally,

) yG:S: = yp + yh = sec x + C1 sec2 x + C2 sec x tan x:

Problem 2.5.3 Find the general solution of the following second-order


linear di¤erential equation, using the Method of Variation of Parameters:
1
y 00 + 3y 0 + 2y = 1+e x

Auxiliary Equation: m2 + 3m + 2 = 0 ) (m + 1) (m + 2) = 0
) m1 = 1 ; m2 = 2 ) yh = C1 e x
+ C2 e 2x :
We take y1 (x) = e x and y2 (x) = e 2x
; and write
x
yp (x) = V1 y1 + V2 y2 = V1 e + V2 e 2x :
We solve the two equations:

y1 V10 + y2 V20 = 0:
1
y10 V10 + y20 V20 = Q (x) = :
1 + ex
y1 y2 e x e 2x 3x
The Wronskian of y1 ; y2 is = = e :
y10 y20 e x 2e 2x
CHAPTER 2. SECOND-ORDER LINEAR DIFFERENTIAL EQUATIONS49

0 y2 0 e 2x
1
Q (x) y20 1+ex
2e 2x e 2x ex
V10 = W
= e 3x = e3x 1+ex
= 1+ex
:
Z Z
ex
V1 = V10 dx = 1+ex
dx = ln (1 + ex )
y1 0
y10 Q (x) e x 0 e2x
V20 = = e3x 1 =
W e x 1+ex
1+ex
Z Z Z
e2x ex
V2 = V10 dx = 1+ex
dx = 1+ex
ex dx:
We use
Z the.substitution
Z z = ex :
z 1
V2 = 1+z dz = 1 + 1+z dz = z + ln (1 + z) = ex + ln (1 + ex ) :
Consequently,
yp (x) = V1 e x + V2 e 2x = e x ln (1 + ex ) + e 2x
( ex + ln (1 + ex ))
) yp (x) = (e x + e 2x ) ln (1 + ex ) e x : Finally,

) yG:S: = yp + yh = (e x
+e 2x
) ln (1 + ex ) + C3 e x
+ C2 e 2x
:

Check: e2x yp (x) = (ex + 1) ln (1 + ex ) ex


We di¤erentiate both sides:
ex
e2x yp0 + 2e2x yp = ex ln (1 + ex ) + (ex + 1) 1+e x ex = ex ln (1 + ex )
) e yp + 2e yp = ln (1 + e )
x 0 x x
(divide by ex )
We di¤erentiate both sides:
ex
ex yp00 + ex yp0 + 2ex yp0 + 2ex yp = 1+e x

) ex yp00 + 3ex yp0 + 2ex yp = 1+e ex


x :
When we divide by ex , we get
1
yp00 + 3yp0 + 2yp = 1+e x;

which veri…es that this yp is indeed a solution of the given di¤erential


equation.

Problem 2.5.4 Find the general solution of the following second-order lin-
ear di¤erential equation, using the Method of Variation of Parameters:
y 00 + y = sec x tan x

Auxiliary Equation:
p m2 + 1 = 0 ) m2 = 1
) m1;2 = 1= i= i
) =0 and =1
) yh = e (C1 sin x + C2 cos x) = C1 sin x + C2 cos x :
x

We take y1 (x) = sin x and y2 (x) = cos x; and write


CHAPTER 2. SECOND-ORDER LINEAR DIFFERENTIAL EQUATIONS50

yp (x) = V1 y1 + V2 y2 = V1 sin x + V2 cos x:


We solve the two equations:

y1 V10 + y2 V20 = V10 sin x + V20 cos x = 0:


y10 V10 + y20 V20 = V10 cos x V20 sin x = Q (x) = sin (ex ) :

The Wronskian of y1 ; y2 is
y1 y2 sin x cos x
W = = = sin2 x cos2 x = 1:
y10 y20 cos x sin x
y2 Q
V10 = W = y2 Q = cos x sec x tan x = tan x:
Z Z
0
V1 = V1 dx = tan x dx = ln (sec x) :
y1 Q
V20 = W
= y1 Q = sin x sec x tan x
) V20 = Z tan2 x = Z1 sec2 x
) V2 = V10 dx = 1 sec2 x dx = x tan x
Consequently,
yp (x) = V1 e x + V2 e 2x = sin x ln (sec x) + x cos x cos x tan x
) yp (x) = sin x ln (sec x) + x cos x sin x: Finally,

) yG:S: = yp + yh = sin x ln (sec x) + x cos x + C3 sin x + C2 cos x:

2.5.5 Revision Problems


Problem 2.5.5 Find the general solution of the following second-order lin-
ear di¤erential equation, using the Method of Variation of Parameters:
y 00 + 3y 0 + 2y = sin (ex ) :

Auxiliary Equation: m2 + 3m + 2 = 0 ) (m + 1) (m + 2) = 0
) m1 = 1 ; m2 = 2
) yh = C1 e + C2 e 2x :
x

We take y1 (x) = e x and y2 (x) = e 2x


; and write
x
yp (x) = V1 y1 + V2 y2 = V1 e + V2 e 2x :
We solve the two equations:

y1 V10 + y2 V20 = 0:
y10 V10 + y20 V20 = Q (x) = sin (ex ) :
CHAPTER 2. SECOND-ORDER LINEAR DIFFERENTIAL EQUATIONS51

y1 y2 e x e 2x 3x
The Wronskian of y1 ; y2 is W = = = e :
y10 y20 e x 2e 2x
0 y2 0 e 2x
Q (x) y20 sin (ex ) 2e 2x
V10 = W
= e 3x = e3x ( e 2x
sin (ex ))
) V10 =Zex sin (ex ) : Z
V1 = V10 dx = ex sin (ex ) dx = cos (ex ) (using.substitution
z = ex ).
y1 0 e x 0
V20 = 0 =W = e3x = e2x sin (ex )
y1 Q (x) e x sin (ex )
Z Z Z
0 2x x
V2 = V1 dx = e sin (e ) dx = z sin z dz (z = ex )
Z
) V2 = z cos z cos z dz = z cos z sin z = ex cos (ex ) sin (ex ) :
Consequently,
yp (x) = V1 e x + V2 e 2x = e x
cos (ex ) + e 2x
(ex cos (ex ) sin (ex ))
) yp (x) = 0 e 2x sin (ex ) : Finally,

) yG:S: = yp + yh = e 2x
sin (ex ) + C1 e x
+ C2 e 2x
:

Check: e2x yp (x) = sin (ex )


We di¤erentiate both sides:
e2x yp0 + 2e2x yp = ex cos (ex )
) ex yp0 + 2ex yp = cos (ex ) (divide by ex )
We di¤erentiate both sides w.r.t. x:
ex yp00 + ex yp0 + 2ex yp0 + 2ex yp = ex sin (ex )
) ex yp00 + 3ex yp0 + 2ex yp = ex sin (ex ) :
When we divide by ex , we get
yp00 + 3yp0 + 2yp = sin (ex ) ;
which veri…es that this yp is indeed a solution of the given di¤erential
equation.

In the next three problems, we solve a second-order linear di¤erential


equation by means of the three methods of Variation of Parameters, Unde-
termined Coe¢ cients and Order Reduction.

Problem 2.5.6 Find the general solution of the following second-order


linear di¤erential equation, using the Method of Variation of Parameters:
y 00 + 9y = 27 cos2 3x 9:
CHAPTER 2. SECOND-ORDER LINEAR DIFFERENTIAL EQUATIONS52

Auxiliary Equation:
p m2 + 9 = 0 ) m2 = 9
) m1;2 = 9 = 3i = i
) =0 and =3
) yh = e x (C1 sin x + C2 cos x) = C1 sin 3x + C2 cos 3x :
We take y1 (x) = sin 3x and y2 (x) = cos 3x; and write
yp (x) = V1 y1 + V2 y2 = V1 sin 3x + V2 cos 3x:
We solve the two equations:

y1 V10 + y2 V20 = V10 sin 3x + V20 cos 3x = 0:


y10 V10 + y20 V20 = V10 cos 3x V20 sin 3x = Q (x) = 27 cos2 3x 9:

0 y2 0 cos 3x
Q (x) y20 27 cos2 3x 9 3 sin 3x 9 cos 3x 27 cos3 3x
V10 = = = 3 sin2 3x 3 cos2 x
y1 y2 sin 3x cos 3x
y10 y20 3 cos 3x 3 sin 3x
3(9 cos3 3x 3 cos 3x)
= 3
= 9 cos3 3x 3 cos 3x = 9 cos 3x 1 sin2 3x 3 cos 3x
) V10 = Z6 cos 3x 9Zsin2 3x cos 3x
) V1 = V10 dx = 6 cos 3x 9 sin2 3x cos 3x dx = 2 sin 3x sin3 3x:
y1 0 y y sin 3x 0
V20 = 0 = 10 20 = 2 = ( 3)
y1 Q (x) y1 y2 3 cos 3x 27 cos 3x 9
) V20 = Z27 sin 3x cos 33xZ 9 sin 3x = 3 sin 3x 9 sin 3x cos2 3x:
2

) V2 = V10 dx = 3 sin 3x 9 sin 3x cos2 3x dx = cos 3x + cos3 3x


Consequently, yp (x) = V1 y1 + V2 y2 = V1 sin 3x + V2 cos 3x
2 4
= 2 sin 3x sin 3x cos2 3x + cos4 3x
2
= 2 sin2 3x sin4 3x 1 sin2 3x + 1 sin2 3x
= 2 sin2 3x sin4 3x 1 + sin2 3x + 1 2 sin2 3x + sin4 3x
) yp (x) = sin2 3x: Finally,

) yG:S: = yp + yh = sin2 3x + C1 sin 3x + C2 cos 3x:

Problem 2.5.7 Find the general solution of the following second-order


linear di¤erential equation, using the Method of Undetermined Coe¢ cients:
y 00 + 9y = 27 cos2 3x 9:
CHAPTER 2. SECOND-ORDER LINEAR DIFFERENTIAL EQUATIONS53

Problem 2.5.8 Find the general solution of the following second-order


linear di¤erential equation, using the Method of Order Reduction:
y 00 + 9y = 27 cos2 3x 9:

Auxiliary Equation:
p m2 + 9 = 0 ) m2 = 9
) m1;2 = 9 = 3i = i
) =0 and =3
) yh = e x (C1 sin x + C2 cos x) = C1 sin 3x + C2 cos 3x :
We take y1 (x) = sin 3x ) y 0 = 3 cos 3x; ) y 00 = 9 sin 3x:
y(x)
and let U (x) = y1 (x)
;
) y = U:y1 = U: sin 3x; y 0 = U:y10 + U 0 y1 = 3U cos 3x + U 0 sin 3x
) y" = U:y1 :U + 2U :y1 + U ":y1 = 9U sin 3x + 6U 0 cos 3x + U 00 sin 3x:
00 0 0

When we substitute in the given d.e., we obtain:


y 00 + 9y = 9U sin 3x + 6U 0 cos 3x + U 00 sin 3x + 9U sin 3x = 1 3 cos2 3x:
) 6U 0 cos 3x + U 00 sin 3x = 27 cos2 3x 9:
and we obtain U 00 + 6 (cot 3x) U 0 = 27 cos2 3x csc 3x 9 csc 3x:
This is a …rst-order linear di¤erential equation in the unknown function
U 0,
with p = 6 cot 3x , Q = csc 3x 3 cos2 3x csc 3x and Integrating factor:
R R

= e2 ln(sin 3x) = eln(sin 3x) = sin2 3x:


p dx 6 cot 3x dx 2
=e =e
R R
) U0 = 1
q dx + C = csc2 3x 27 cos2 3x: sin 3x 9 sin 3xdx + C
) U 0 = csc2 3x ( 3 cos3 3x + 3 cos 3x + C) = 3 csc2 3x: cos 3x ( cos2 3x + 1)+
C csc2 3x:
) U 0 = R3 csc2 3x: cos 3x: sin 2 2 2
R 3x + C csc R3x =23 cos 3x + C csc 3x
) U = U dx + K = 3 cos 3xdx + C csc 3xdx + K
0

) U = sin 3x C3 cot 3x + K:

) yG:S: = y1 U = sin2 3x + K sin 3x + C1 cos 3x:


CHAPTER 2. SECOND-ORDER LINEAR DIFFERENTIAL EQUATIONS54

2.6 Cauchy-Euler Di¤erential Equations


2.6.1 Description and Method of Solution
A Cauchy-Euler Di¤erential Equation is a linear di¤erential equation that
takes the form:

an xn y (n) + an 1 xn 1 y (n 1)
+ + a2 x2 y 00 + a1 xy 0 + a0 y = f (x) ; (2.31)
dy d2 y dk y
where y 0 = dx
; y" = dx2
and y (k) = dxk
.

Theorem 5 (Cauchy-Euler) When we use the substitution

x = et ; (2.32)

that is,
t = ln x; (2.33)
and we let be the following di¤erentail operator:
d
= ; (2.34)
dt
we …nd that, for all natural numbers r :
!
Y
r 1
xr y (r) = ( k) y = ( 1) ( 2) ( r + 1) y: (2.35)
k=0

Proof. For any continuously di¤erentiable function z (x) we have, by Chain


Rule:
z = dz(x(t))
dt
dz dx
= dx : dt = z 0 et :

) z = xz 0 : (2.36)

In particular,
r
(xr ) = x dx
dx
= xrxr 1
: that is,

(xr ) = rxr : (2.37)

Induction hypothesis: For all natural numbers r :


!
Y
r 1
xr y (r) = ( k) y: (2.38)
k=0
CHAPTER 2. SECOND-ORDER LINEAR DIFFERENTIAL EQUATIONS55

This hypothesis is true for r = 1; by (2.36).


Suppose this hypothesis is true for some natural number r . We then …nd
that
Q
r rQ1
( k) y = ( r) ( k) y
k=0 k=0
=( r) xr y (r) (by hypothesis (2.38))
r (r) r (r)
= xy r xy
= (x ) y + xr y (r)
r (r)
rxr y (r)
= rxr y (r) + xr xy (r+1) rxr y (r) (by (2.37) and (2.36))
r+1 (r+1)
=x y :
This establishes the induction step (meaning that the induction hypoth-
esis (2.38) remains true for r + 1), and completes the proof of (2.35).

So, by using the substitutions (2.32), and its consequence (2.35), the
Cauchy-Euler di¤erential equation (2.31) becomes a linear di¤erential
equation with constant coe¢ cients.

Instances of (2.35) are:

xy 0 = y;
2
x2 y 00 = ( 1) y = y; (2.39)
3 000 3 2
xy = ( 1) ( 2) y = 3 +2 y:
CHAPTER 2. SECOND-ORDER LINEAR DIFFERENTIAL EQUATIONS56

2.6.2 Lecture Examples


Find the general solutions of the following di¤erential equations:

Problem 2.6.1 x2 y 00 4xy 0 + 4y = 10 sin (2 ln x) :

This is a Cauchy-Euler di¤erential equation.


Let x = et . Then ln x = t and
xy 0 = y; x2 y 00 = ( 1) y = 2 y:
This equation becomes the follwing di¤erential equation in the unkown
function y (t) :
2
y 4 y + 4y = 2 5 + 4 y = 10 sin (2t) :
Auxiliary Equation: m2 5m + 4 = 0
) (m 1) (m 4) = 0 ) m1 = 1; m2 = 4:
) Homogeneous solution is: yh (t) = C1 et + C2 e4t :

) Particular solution takes the form yp (t) = A sin (2t) + B cos (2t) :
) yp = 2B sin (2t) + 2A cos (2t)
) 2 yp = 4A sin (2t) 4B cos (2t)
) 2 5 + 4 yp = 10B sin (2t) 10A cos (2t) = 10 sin (2t)
) A = 0; B = 1: ) yp (t) = cos (2t) :
General Solution is yG:S: (t) = yp + yh = cos (2t) + C1 et + C2 e4t .
As a function in x; this general solution becomes
yG:S: (x) = cos (2 ln x) + C1 x + C2 x4 :

2.6.3 Classroom Exercises


18
Problem 2.6.2 x2 y 00 + 9xy 0 + 25y = x4

This is a Cauchy-Euler di¤erential equation.


Let x = et . Then t = ln x and
xy 0 = y; x2 y 00 = ( 1) y = 2 y:
This equation becomes the follwing di¤erential equation in the unkown
function y (t) :
2
y + 9 y + 25y
2
= + 8 + 25 y = 18e 4t :
Auxiliary Equation:
p m2p+ 8m + 25 =p0
) m1;2 = b b2 4ac
2a
= 8 264 100 = 8 2 36 = 82 6i
) m1;2 = 4 3i = i ) = 4 and =3
CHAPTER 2. SECOND-ORDER LINEAR DIFFERENTIAL EQUATIONS57

and yh = e t (C1 sin ( t) + C2 cos ( t)) :


yh (t) = e 4t (C1 sin (3t) + C2 cos (3t)) :
) Particular solution takes the form: yp = Ae 4t
) yp = 4Ae 4t
) 2 yp = 16Ae 4t
) 2 + 8 + 25 yp = Ae 4t (16 32 + 25) = 9Ae 4t = 18e 4t :
) A = 2: ) yp (t) = 2e 4t :
General Solution is yG:S: (t) = yp + yh = 2e 4t + e 4t (C1 sin (3t) + C2 cos (3t)) .
As a function in x; this general solution becomes
yG:S: (x) = x14 (2 + C1 sin (3 ln x) + C2 cos (3 ln x)) .

Problem 2.6.3 x2 y 00 5xy 0 + 9y = 36 cos (3 ln x)

This is a Cauchy-Euler di¤erential equation.


Let x = et . Then ln x = t and
xy 0 = y; x2 y 00 = ( 1) y = 2 y:
This equation becomes the follwing di¤erential equation in the unkown
function y (t) :
2
y 5 y + 9y
2
= 6 + 9 y = 36 cos 3t:
Auxiliary Equation: m2 6m + 9 = 0
) (m 3) = 02
) m1 = m2 = 3:
) Homogeneous solution is: yh (t) = C1 e3t + C2 te3t :
) Particular solution takes the form: yp = A sin 3t + B cos 3t
) yp = 3A cos 3t 3B sin 3t
) 2 yp = 9A sin 3t 9B cos 3t
) 2 6 + 9 yp = 18A cos 3x + 18B sin 3x = 36 cos 3t
) A = 2; B = 0: ) yp (t) = 2 sin 3t:
General Solution is yG:S: (t) = yp + yh = 2 sin 3t + C1 e3t + C2 te3t .
As a function in x; this general solution becomes
yG:S: (x) = 2 sin (3 ln x) + C1 x3 + C2 x3 ln x:
CHAPTER 2. SECOND-ORDER LINEAR DIFFERENTIAL EQUATIONS58

2.6.4 Revision Problems


Find the general solutions of the following di¤erential equations:

Problem 2.6.4 x2 y 00 2xy 0 + 2y = 5x2 :

This is a Cauchy-Euler di¤erential equation.


Let x = et . Then t = ln x and
xy 0 = y; x2 y 00 = ( 1) y = 2 y:
This equation becomes the follwing di¤erential equation in the unkown
function y (t) :
2
y 2 y + 2y
2
= 3 + 2 y = 5e2t :
Auxiliary Equation: m2 3m + 2 = 0
) (m 1) (m 2) = 0 ) m1 = 1; m2 = 2:
) y h = C1 e + C2 e :
t 2t

) Particular solution takes the form: yp = Ate2t


) yp = 2Ate2t + Ae2t
) 2 yp = 4Ate2t + 4Ae2t
) 2 3 + 2 yp = Ae2t (4t + 4 6t 3 + 2t) = Ae2t = 5e2t :
) A = 5; yp (t) = 5te2t :

) General solution is:: yG:S: (t) = yp + yh = (5t + C2 ) e2t + C1 et :


As a function in x; this general solution becomes
yG:S: (x) = (C2 + 5 ln x) x2 + C1 x:

Problem 2.6.5 x2 y 00 5xy 0 + 9y = 6x3 :

This is a Cauchy-Euler di¤erential equation.


Let x = et . Then t = ln x and
xy 0 = y; x2 y 00 = ( 1) y = 2 y:
This equation becomes the follwing di¤erential equation in the unkown
function y (t) :
2
y 5 y + 9y
2
= 6 + 9 y = 6e3t :
Auxiliary Equation: m2 6m + 9 = 0
) (m 3) = 02
) m1 = m2 = 3:
) yh (t) = e (C1 + C2 t) :
3t

) Particular solution takes the form: yp = At2 e3t


CHAPTER 2. SECOND-ORDER LINEAR DIFFERENTIAL EQUATIONS59

) yp = 3At2 e3t + 2Ate3t


) 2 yp = 9At2 e3t + 12Ate3t + 2Ae3t
) 2 6 + 9 yp = Ae3t (9t2 + 12t + 2 18t2 12t + 9t2 ) = 2Ae3t =
6e3t
) A = 3; yp (t) = 3t2 e3t :
) General solution is:: yG:S: (t) = yp (t) + yh (t) = e3t (C1 + C2 t + 3t2 )
As a function in x; this general solution becomes
yG:S: (x) = x3 C1 + C2 ln x + 3 (ln x)2 :

Problem 2.6.6 x2 y 000 + xy 00 4y 0 = 24x2 :


We mulitply both sides by x; in order to get the following Cauchy-Euler
di¤erential equation:
x3 y 000 + x2 y 00 4xy 0 = 24x3 :
Let x = et . Then xy 0 = y; x2 y 00 = ( 1) y = 2 y; and
3 000 3 2
xy = ( 1) ( 2) y = 3 + 2 y:
This equation becomes the follwing di¤erential equation in the unkown
function y (t) :
3
3 2+2 y+ 2 y 4 y = 24e3t :
) 3
2 2 3t
3 y = 24e ; that is,
2
2 3 V = 24e3t ;
where V = dydt
:
Auxiliary Equation: m2 2m 3 = 0
) (m + 1) (m 3) = 0 ) m1 = 1; m2 = 3:
) Homogeneous solution is: Vh = C1 e t
+ C2 e3t :
) Particular solution takes the form Vp = Ate3t :
) Vp = 3Ate3t + Ae3t :
) 2 Vp = 9Ate3t + 3Ae3t + 3Ae3t = 9Ate3t + 6Ae3t :
) 2 2 3 Vp = Ae3t (9t + 6 6t 2 3t) = 4Ae3t = 24e3t :
) A = 6: ) Vp = 6te3t :
) dydt
G:S:
= VG:S: = Vp + Vh = 6te3t + C2 e3t + C1 e t
R dyG:S: R
) yG:S:
R (t) = dt
dt = (6te3t + C2 e3t + C1 e t ) dt
3t 3t C2 3t 2 C2
= 2te 2 e dt + 3 e C1 e = 2t 3 + 3 e3t C1 e t :
t

) yG:S: (t) = (K2 + 2t) e3t + K1 e t :


As a function in x; this general solution becomes
yG:S: (x) = x3 (K2 + 2 ln x) + Kx1 :
Chapter 3

The Laplace Transform

3.1 De…nition and Properties


De…nition. Let f (t) be a real function on the real line R, which satis…es
the following three conditions:
(i) f (t) = 0 for all negative
R t values of t,
(ii) All de…nite integrals 0 f (t) dt exist, for all (…nite) values of t ,
and
(iii) f (t) has an exponential order s . (This means that, starting from
some constant value t of the variable t, the inequality jf (t)j es t holds.)
Then the Laplace transform of f (t) is another function L ff g ; of one real
variable s, de…ned for all s > s by
Z 1
L ff g (s) = e st f (t) dt.
0

In what follows, the unit step function U (t) is a real function on R,


de…ned by
0 when t < 0
U (t) = :
1 when t 0
The Dirac function (t) is a distribution, de…ned on the real line R by
the condition that Z t
(z) dz = U (t) ;
0

for all values of the variable t. It follows that (t) = 0 for all t < 0 and all
t > 0. This (t) is an idealization of a very thin pulse that starts at t = 0
and has area 1.

60
CHAPTER 3. THE LAPLACE TRANSFORM 61

Basic Properties of the Laplace Transform:


1. L ff g (s) exists at all s > s .

2. The Laplace transform is linear; that is L ff gg = L ff g L fgg


and L fcf g = cL ff g when c is a constant.

3. The Laplace transform is injective (one-to-one); that is, distinct func-


tions have distinct Laplace transforms. Our need for this important
property is the main reason why Laplace transforms must be functions.

4. lim L ff g (s) = 0:
s!1

5. lim sL ff g (s) = f (0) :


s!1

Rules of Laplace Transform


We denote L ff g (s) by F (s), and we assume s > s :

n o
dF (s) df (t)
L ft f (t)g = ds
L dt
= sF (s) f (0)
n o
dn F d2 f (t)
L ftn f (t)g = ( 1)n dsn
L dt2
= s2 F (s) sf (0) f 0 (0)

R1 nR o
1 t
L t
f
(t) = s F (v) dv; L 0
f (z) dz = 1s F (s)
whenever one of them exists.

First Shift Theorem: (s > s + k) Second Shift Theorem: (b > 0)

L ekt f (t) = F (s k) = F (s)js !(s k)


L ff (t b) U (t b)g = e bs
F (s)
CHAPTER 3. THE LAPLACE TRANSFORM 62

Proof of Rules of Laplace Transform.


d d
R 1 st R1 @e st
Rule 1: ds (L
R1 ff g) = ds R 01
e f (t) dt = 0 @s
f (t) dt
= 0
t e st f (t) dt = 0 e st tf (t) dt = L ft f (t)g :
) d
ds
(L ff g) = L ftf g : Consequently,
n d d d
L ft f g = L ft (t ( (tf )))g = ds ds ds
(L ff g)
n dn
= ( 1) dsn (L ff g) :
n o R R 1 @e st
1 t=1
Rule 2: L dfdt(t) = 0 e st dfdt(t) dt = [e st f (t)]t=0 0 @t
f (t) dt
(integration by parts)
R1
=0 f (0) + s 0 e st f (t) dt (because s > s )
n o
) L dfdt(t) = sF (s) f (0) : Applying this rule twice, we get
n 2 o n o n o
L d dtf 2(t) = L dtd dfdt(t) = sL dfdt(t) f 0 (0)
= s (sF f (0)) f 0 (0) = s2 F sf (0) f 0 (0) :
R1
Rule 3: Since lim s F (v) dv = 0 then there is a function g (t) such that
s!1 R
1
L fgg (s) = s F (v) dv.
d
R1
Hence by Rule 1, L ftg (t)g (s) = ds s
F (v) dv = F (s) = L ff g (s) :
Consequently, tg = f and so g = 1t f: This means that
R1
L 1t f (t) = s F (v) dv.
d
Rt
Rule 4: Since f (t) = dt 0
f (z) dz , then
n Rt o
d
F (s) = L f f (t)g = L 0
f (z) dz dt
nR R o
t 0
= sL 0 f (z) dz 0
f (z) dz (by Rule 2). So,
nR o nR o
t t
F (s) = sL 0 f (z) dz 0; that is L 0 f (z) dz = 1s F (s) :
R1 R1
Rule 5: L ekt f (t) = 0
e st
ekt f (t) dt = 0
e (s k)t
f (t) dt = F (s k) :
R 1 st
Rule 6: L ff (t b) U (t b)g = e f (t b) U (t b) dt
R b st R01 st
= e f (t b) U (t b) dt + e f (t b) U (t b) dt
Rb 0R
1 R 1
b
= 0 0dt + b e st f (t b) dt = 0 + e bs b e s(t b) f (t b) dt:
Now we use the substitution z = t b, and …nd that
R1
L ff (t b) U (t b)g = e bs 0 e sz
f (z) dz = e bs
L ff g :
CHAPTER 3. THE LAPLACE TRANSFORM 63

Famous Laplace Transforms


f (t) F (s) =L ff g (s) f (t) F (s) =L ff g (s)

1 !
1 = U (t) s
(s > 0) sin (!t) s2 +! 2
(s > 0)
1
t s2
(s > 0)
2 s
t2 s3
(s > 0) cos (!t) s2 +! 2
(s > 0)
6
t3 s4
(s > 0)
24 k
t4 s5
(s > 0) sinh (kt) s2 k 2
(s > jkj)

n! s
tn (n = 0; 1; 2; ) sn+1
(s > 0) cosh (kt) s2 k 2
(s > jkj)

1
ekt s k
(s > k) (t) 1

Derivation of Famous Laplace Transforms.


R1 t=1
1. L f1g = 0 e st 1 dt = s1 e st t=0 = 0 + 1s (when s > 0):
dn dn
2. L ftn g = L ftn 1g = ( 1)n dsn
(L f1g) = ( 1)n dsn
1
s
= n!
sn+1
:
1 1
3. L ekt = L ekt 1 = L f1gjs !(s k)
= s js
= s k
(using
!(s k)
Rule 5, and taking s > k).
Z1
st
4. whenever s > 0 : L fsin (!t)g = e sin (!t) dt
0
h it=1
s sin(!t) ! cos(!t) s sin(0)+! cos(0) !
= s2 +! 2
e st =0+ s2 +! 2
= s2 +! 2
: :
t=0
h it=1
! sin(!t) s cos(!t) s
Similarly, L fcos (!t)g = s2 +! 2
e st = s2 +! 2
:
t=0

5. L fsinh (kt)g = 21 L ekt e kt


= 1
2
1
s k
1
s+k
= k
s2 k 2
;
L fcosh (kt)g = 12 L ekt + e kt
= 1
2
1
s k
+ 1
s+k
= s
s2 k 2
;
whenever s > k and s > k; that is s > jkj :
Z1 Z1 Z1
st s0
6. L f g = e (t) dt = e (t) dt = (t) dt = U (1) = 1:
0 0 0
Alternatively, the de…nition of the Dirac function (t) stipulates that
Rt
U (t) = 0 (z) n dz: We combine
o this with Rule 4, and …nd that
1
R t
s
= L fU (t)g = L 0 (z) dz = 1s L f g : Therefore, L f g = 1:
CHAPTER 3. THE LAPLACE TRANSFORM 64

3.2 Problems on Direct Laplace Transform


Find the Laplace transforms of the following functions:

3.2.1 Lecture Examples


Problem 3.2.1 t sin 3t + 8t3 4e2t

L (sin 3t) = s23+9


) L (t sin 3t + 8t3 4e2t ) = d
ds
L (sin 3t) + L (8t3 4e2t )
d 3
= ds s2 +9
+ 8 s3!4 4s 12 = 6s
(s2 +9)2
+ 48
s4
4
s 2
.

sin(3t)
Problem 3.2.2 t

sin(3t) R1 R1 3 3 1 v v=1
L t
= L (sin (3t)) = v 2 +9
dv = 3
tan 3 v=s
s s

1 1 s 1 s 1 s
= tan (1) tan 3
= 2
tan 3
= cot 3
:

d2 (t cos 5t)
Problem 3.2.3 h (t) = dt2

2
Let f (t) = t cos 5t; i.e., h (t) = ddt2f
s2 +25 s(2s)
L (f ) = L (t cos 5t) = dL(cot
ds
s5)
= d
ds
s
s2 +25
= (s2 +25)2
s(2s) (s +25)
2 2 25
= (s2 +25)2 = (ss2 +25) 2;

Also, f (0) = 0 and f 0 (t) = cos 5t 5t sin 5t ) f 0 (0) = 1


d2 f (s2 25)
) L (h) = L dt2
= s2 L (f ) sf (0) f 0 (0) = s2 (s2 +25)2 0 1
s4 25s2 (s4 +50s2 +625)
) L (h) = (s2 +25)2
= 75s2 625
(s2 +25)2

d(e3t cos 4t)


Problem 3.2.4 f (t) = dt

s s 3
L (e3t cos 4t) = s2 +16 js
= (s 3)2 +16
!s 3
d(e 3t cos 4t) s2 3s
L (f ) = L dt
= sL (e3t cos 4t) (e3t cos 4t)jt=0 = s2 6s+25
1
s2 3s (s2 6s+25)
) L (f ) = s2 6s+25
= 3s 25
s2 6s+25
.
CHAPTER 3. THE LAPLACE TRANSFORM 65

Rt
Problem 3.2.5 g (t) = z 10 e 4z
dz
0

Rt
L (t10 e 4t
)= 10!
s11 js
= 10!
(s+4)11
)L z 10 e 4z
dz = 10!
s(s+4)11
:
!s+4
0

Problem 3.2.6 f (t) = te3t cos (4t)


d s s2 16
L (t cos (4t)) = ds s2 +16
= (s2 +16)2
(s)2 16 (s 3)2 16
) L (e3t t cos (4t)) = 2 2 = 2 .
((s) +16) ((s 3)2 +16)
js !s 3

Problem 3.2.7 h (t) = (t 5)10 e 4(t 5)


U (t 5)

L (t 5)10 e 4(t 5)
U (t 5) = L (t10 e 4t
)e 5s
= 10!
(s+4)11
e 5s :

2(t 3)
Problem 3.2.8 g (t) = (t 3) e sin 4 (t 3) U (t 3)
d d 4 8s
L (t sin (4t)) = ds
L (sin (4t)) = ds s2 +16
= (s2 +16)2

) L (e 2t
t sin (4t)) = 8(s)
2 = 8(s+2)
2
((s)2 +16) js !s+2
((s+2)2 +16)
) L (g) = L e 2(t 3)
(t 3) sin 4 (t 3) U (t 3) = e 3s
L (e 2t
t sin (4t))
) L (g) = 8(s+2)
2 e 3s
.
((s+2)2 +16)

Problem 3.2.9 Draw a neat sketch for the following function:


8
< 0 1<t<0
1
gb (t) = 0 t<b ; where b is a positive constant.
: b
0 b t<1
gb (t) = 1b U (t) 1
b
U (t b) : So, by the Second Shift Theorem:
11 11 1 e bs
L (gb ) (s) = bs bs
e bs = bs
.
Note: By L’Hopital Rule,
0+se bs
lim L (gb ) (s) = lim s
= 1:
b !0 b !0

Problem 3.2.10 k (t) = 7 (t) 2 (t 4) + 9 sin 4 (t 5) U (t 5)


4s 36
L (k) = 7 2e + s2 +16
e 5s
CHAPTER 3. THE LAPLACE TRANSFORM 66

Convolution
De…nition 1 The convolution of two functions f and g is the function f ? g
de…ned as follows:
Rt
(f ? g) (t) = 0
f (z) g (t z) dz

It follows that the binary operation ? is bilinear, commutative and


associative, and it has unit .

Basic Properties of the Binary Operation Convolution:


1. Commutativity: f ?g =g?f

2. Associativity: f ? (g h) = (f ? g) h

3. Linearity: f ? (g + h) = (f ? g) + (f h) ;
f ? (cg) = c (f g) = (cf ) ? g when c is a constant
Rt
4. 1 is not a neutral element: (f ? 1) (t) = 0
f (z) dz

5. is the neutral element: f ? = f:

6. Existence of Laplace transforms: If both f and g have Laplace trans-


forms, then f ? g has Laplace transform.

Convolution Theorem. For any two functions f (t) and g (t) which
have Laplace transforms:

L ff ? gg=L ff g.L fgg

Hence, for any two Laplace transforms F (s) and G (s) :

1 1 1
L (F:G)=L (F ) ? L (G)
CHAPTER 3. THE LAPLACE TRANSFORM 67

Proof. We have 0 1 0 1
Z1 Z t Z1 Z t
L ff ? gg = @ f (z) g (t z) dz A e st dt = @ f (z) g (t z) e st
dz A dt
0 0 0 0
This is a double integral on the following region in the tz-plane:

0 t < 1; 0 z t:

We use the substitution t = z+y, and get the following region in the yz-plane:

0 y < 1; 0 z < 1:

Then by Substitution Theorem for Double Integrals, we have


0 @t @t 1
Z1 Z1 @y @z
L ff ? gg = @ f (z) g (y) e s(z+y) dz A dy
@z @z
0 0 @y @z
0 1 0 1
Z1 Z1 Z1 Z1
1 1
= @ f (z) g (y) e sz sy
dz A dy = @ f (z) g (y) 1 e sz
e sy
dz A dy
0 1
0
0 0 10 1 1 0 0
Z1 Z
= @ f (z) e sz
dz A @ g (y) e sy
dy A :
0 0

) L ff ? gg = L ff g :L fgg

Problem 3.2.11 Find the Laplace transforms of the following functions:

1. f (t) = t e3t sin 6t

2. g (t) = t ? (e3t sin 6t)

3. h (t) = t ? e3t ? sin 6t

d 12(s 3)
L (f ) = ds
L (e3t sin 6t) = 2 (Problem 3.2.19):
((s 3)2 +36)

1 6 6
L (g) = L (t) :L (e3t sin 6t) = s s2 +36 js
= s((s 3)2 +36)
:
!s 3

6
L (h) = L (t) :L (e3t ) :L (sin 6t) = s(s 3)(s2 +36)
.
CHAPTER 3. THE LAPLACE TRANSFORM 68

3.2.2 Classroom Exercises


d
Problem 3.2.12 r (t) = dt
(t sin 6t)
d d 6 12s
L (t sin 6t) = ds
(L (sin 6t)) = ds s2 +36
= (s2 +36)2
d
Also, L dt
(t sin 6t) = sL (t sin 6t) (t sin 6t)jt=0

) L (r) = L d
dt
(t sin 6t) = 12s2
(s2 +36)2
:

Problem 3.2.13 f (t) = t cos 3 t + 4

f (t) = t cos 3 t + 4 = t cos 3t cos 34 sin 3t sin 34 = p1 t cos 3t


2
p1 t sin 3t
2
) L (f (t)) = p12 ds
d d
L (cos 3z) + p12 ds L (sin 3z)
2
= p1 d s
+ p1 d 3
= p1 9 6s s2 .
2 ds s2 +9 2 ds s2 +9 2 (s2 +9)

Rt
Problem 3.2.14 f (t) = z 4 e3z dz
0

4! 4!
L (z 4 e3z ) = L (z 4 )js !s 3
= s5 js
= (s 3)5
!s 3

Rt
Also, L z 4 e3z dz = 1s L (z 4 e3z )
0
Rt
) L (f ) = L z 4 e3z dz = 4!
s(s 3)5
.
0

Rt 2z
Problem 3.2.15 e cosh (5z) dz;
0

2z (s) s+2
L (e cosh (5z)) = L (cosh (5z))js !s+2
= (s)2 25 j
= (s+2)2 25
s !s+2
Rt
)L e 2z
cosh (5z) dz = 1s L (e 2z
cosh (5z)) = s+2
s((s+2)2 25)
.
0

Rt
Problem 3.2.16 g (t) = z sin 3z dz
0

d d 3
L (z sin 3z) = ds L (sin 3z) = ds s2 +9
= (s26s
+9)2
.
Rt
L z sin 3z dz = 1s L (z sin 3z) = (s2 +9)
6
2 :
0
CHAPTER 3. THE LAPLACE TRANSFORM 69

d(e 2t cos 5t)


Problem 3.2.17 f (t) = dt

2t s s+2
L (e cos 5t) = s2 +25 js
= (s+2)2 +25
!s+2
d(e 2tcos 5t) 2t 2t s2 +2s
L (f ) = L dt
= sL (e cos 5t) (e cos 5t)jt=0 = s2 +4s+29
1

) L (f ) = s2 +2s s2 4s 29
s2 +4s+29
= 2s 29
s2 +4s+29

Rt
Problem 3.2.18 g (t) = 0
e6z sin 4z dz
4
L (e6t sin 4t) = s2 +16 js !s 6
= (s 6)42 +16
Rt
L (g) = L 0 e6z sin 4z dz = 1s L (e6t sin 4t) = 4
s(s2 12s+52)
:

Problem 3.2.19 f (t) = e3t t sin 6t


dL(sin 6t) d 6 12s
L (t sin 6t) = ds
= ds s2 +36
= (s2 +36)2
:
) L (f ) = L (e3t t sin 6t) = L (t sin 6t)js !s 3
= 12s
(s2 +36)2 js !s 3

12(s 3)
= 2
((s 3)2 +36)

) L (f ) = 12s 36
(s2 6s+45)2
.

1 cos 4t
Problem 3.2.20 f (t) = t

R1 R1 1 v 1 v=1
L (f ) = L (1 cos 4t) = v v 2 +16
dv = ln (v) 2
ln (v 2 + 16) v=s
s s
p v=1
h iv=1
= ln (v) ln v 2 + 16 = ln p v = ln 1 ln p s
v=s v 2 +16 v=s s2 +16
p
) L (f ) = ln s2 +16
s

1 cos 4(t 6)
Problem 3.2.21 k (t) = t 6
U (t 6) :

p
6s s2 +16
We use Problem 3.2.20: L (k) = e ln s
:
CHAPTER 3. THE LAPLACE TRANSFORM 70

Problem 3.2.22 f (t) = e2(t 3)


sin 4 (t 3) U (t 3)
f (t) = (e2t sin 4t)j
t !t 3
4 4
L (e2t sin 4t) = s2 +16 js
= (s 2)2 +16
!s 2

) L (f ) = e 3s
L (e2t sin 4t) = 4e 3s
(s 2)2 +16
:

Problem 3.2.23 k (t) = e2(t 6)


cos 3 (t 6) U (t 6) + 3 (t 2)
s s 2
L (e2t cos 3t) = L (cos 3t)js !s 2
= s2 +9 js
= (s 2)2 +9
!s 2

) L e2(t 6)
cos 3 (t 6) U (t 6) = e 6s
L (e2t cos 3t)
) L (k) = s 2
(s 2)2 +9
e 6s + 3e 2s
:

Rt Rt
Problem 3.2.24 g (t) = ze7z sin 4z dz + (ze7z ) sin 4z dz
0 0
7t 8s 56
As in Problem (3.2.19) above, L (te sin 4t) = 2
((s 7)2 +16)
) L (g) = 1s fL (te7t sin 4t) + L (te7t ) :L (sin 4t)g
1 8s 56 4
= s 2 + (s 7)2 (s2 +16)
:
((s 7)2 +16)

Problem 3.2.25 sin2 t


L sin2 t = L 1
2
1
2
cos 2x = 1
2s
s
2(s2 +4)
= 2
s(s2 +4)
.

Problem 3.2.26 q (t) = sin 5t cos 2t + sin 5t cos 2t


L (sin 5t cos 2t) = 21 L (sin 7t + sin 3t) = 1
2
7
s2 +49
3
s2 +9
5
L (sin 5t cos 2t) = L (sin 5t) :L (cos 2t) = : s
s2 +25 s2 +4

) L (q) = 1
2
7
s2 +49
3
s2 +9
+ 5s
(s2 +25)(s2 +4)

Problem 3.2.27 k (t) = (t 7) e3(t 7)


cos 2 (t 7) U (t 7)
d d s 2s2 (s2 +4) s2 4
L (t cos (2t)) = ds
L (cos (2t)) = ds s2 +4
= (s2 +4)2
= (s2 +4)2
(s)2 4 (s 3)2 4
) L (e3t t cos (2t)) = 2 = 2
((s)2 +4) js !s 3 ((s 3)2 +4)

) L (k) = L e3(t 7)
(t 7) cos 2 (t 7) U (t 7) = e 7s
L (e3t t cos (2t))
(s 3)2 4
) L (k) = 2 e 7s
= s2 6s+5
(s2 6s+13)2
e 7s .
((s 3)2 +4)
CHAPTER 3. THE LAPLACE TRANSFORM 71

Problem 3.2.28 Draw a neat sketch for the following function, and …nd its
Laplace transform:

f (t) = cos t + sin t 2


U t 2

s+e s=2
L (f ) = s2 +1
.

3.2.3 Revision Problems


Find the Laplace Transforms of the following functions:

Problem 3.2.29 h (t) = 4e3t sinh 5t

20 20 20
L (h) = (s)2 25
= (s+3)2 25
= s2 +6s 16
js !s+3

Another Solution
2 2
L (h) = L (2e3t (e5t e 5t
)) = L (2e8t 2e 2t
)= s 8 s+2
) L (h) = 20
(s 8)(s+2)
.

Rt
Problem 3.2.30 g (t) = z 6 e3z dz + (t 2)6 e3(t 2)
U (t 2)
0

6! 6!
L (t6 e3t ) = L (t6 )js !s 3
= s7 j s
= (s 3)7
:
!s 3

) L (g) = 1s L (t6 e3t ) + e 2s


L (t6 e3t ) = 1
s
+e 2s 6!
(s 3)7
:

Problem 3.2.31 g (t) = (t 4) sin 6 (t 4) U (t 4)

4s 12s e 4s
L (g) = L ((t 4) sin 6 (t 4) U (t 4)) = e L (t sin 6t) = (s2 +36)2
:

Problem 3.2.32 k (t) = e6(t 5)


sin 4 (t 5) U (t 5)

L (e6t sin 4t) = L (sin 4t)js !s 6

4 4 4
= s2 +16 js
= (s 6)2 +16
= s2 12s+52
!s 6

) L (k) = L e6(t 5)
sin 4 (t 5) U (t 5) = e 5s
L (e6t sin 4t) = 4e 5s
s2 12s+52
.
CHAPTER 3. THE LAPLACE TRANSFORM 72

2(t 6)
Problem 3.2.33 k (t) = e 3et 6
U (t 6)

6s 2t 1 3 ( 2s 7)
L (k) = e L (e 3et ) = s+2 s 1
e 6s
= (s 1)(s+2)
e 6s

Problem 3.2.34 (t 2) e9(t 2)


U (t 2)
1 1
L (t e9t ) = s 2 js !s 9
= (s 9)2

) L (t 2) e9(t 2)
U (t 2) = e 2s
L (t e9t ) = e 2s
(s 9)2
:

1 e4t
Problem 3.2.35 t

Z1 Z1
1 e4t
L t
= L (1 4t
e )= 1
v
1
v 4
dv = [ln (v) ln (v 4)]v=1
v=s
s s
v v=1 s s 4
= ln v 4 v=s
= ln 1 ln s 4
= ln s
:

Rt Rt
Problem 3.2.36 g (t) = e3z cos (4z) dz + z cos (4z) dz:
0 0

We use our results in Problems (3.2.4) and (3.2.6):


L (g) = 1s L (e3z cos (4z)) + 1s L (t cos (4t))
s 3 s2 16
= s((s 3)2 +16)
+ s(s2 +16)2
.

Problem 3.2.37 e7t cos 2t + t cos 2t + t cosh 2t + te7t cosh 2t:


s 7 s2 4 s2 +4 (s 7)2 +4
Final Answer: (s 7)2 +4
+ (s2 +4)2
+ (s2 4)2
+ 2
((s 7)2 4)

Problem 3.2.38 sin 2t cos 2t + t sin 2t cos 2t


2 4s
Final Answer: s2 +16
+ (s2 +16)2

Problem 3.2.39 (t sin 2t) cos 2t + t (sin 2t cos 2t) :


4s2 2
Final Answer: (s2 +4)3
+ s2 (s2 +16)
CHAPTER 3. THE LAPLACE TRANSFORM 73

Problem 3.2.40 te2t sin 4t + t (e2t sin 4t) + e2t (t sin 4t) :
8(s 2) 4 8s
Final Answer: 2 + s2 ((s 2)2 +16)
+ (s 2)(s2 +16)2
((s 2)2 +16)

Problem 3.2.41 3e4t cosh 5t + 4e3t + 2 cos 3t e 4t


:
3(s 4) 4 2(s+4)
Final Answer: (s 4)2 25
+ s 3
+ (s+4)2 +9

2t
Problem 3.2.42 1 3e + 2e2t cos 3t + e2t sin 3t:
1 3 2s 1
Final Answer: s s+2
+ (s 2)2 +9

Problem 3.2.43 (t 3)6 e 4(t 3)


U (t 3) :
6!
Final Answer: (s+4)7
e 3s

d
Rt
Problem 3.2.44 dt
(t9 e 5t
)+ v9 e 5v
dv + (t9 e 5t
) cos 7t:
0

9!(s) 9! 9!(s)
Final Answer: (s+5)10
+ s(s+5)10
+ (s+5)10 (s2 +49)

Problem 3.2.45 t9 (e 5t
cos 7t) + t9 e 5t
cos 7t:
9!(s+5) 9!(s)
Final Answer: s10 ((s+5)2 +49)
+ s10 (s+5)(s2 +49)

Problem 3.2.46 U (t 5) + (t 8) (Draw a sketch).


e 5s
8s
Final Answer: s
+e

Problem 3.2.47 cos 2 t 2


U t 2
cos 2 t 2
(Draw a sketch):

s(1+e s=2 )
Final Answer: s2 +4

Problem 3.2.48 e2t e5t

Problem 3.2.49 e2t e5t e4t

Problem 3.2.50 e2t e5t cos 3t

Problem 3.2.51 U (t 3) U (t 7)
CHAPTER 3. THE LAPLACE TRANSFORM 74

Problem 3.2.52 h (t) = (t 4)2 U (t 6)

h (t) = ((t 6) + 2)2 U (t 6)


= (t 6)2 + 4 (t 6) + 4 U (t 6)
= (t2 + 4t + 4)jt !t 6

) L (h) = e 6s
L (t2 + 4t + 4) = e 6s 2
s3
+ 4
s2
+ 4
s
= 2+4s+4s2
s3
e 6s .

Problem 3.2.53 g (t) = sin 2 t 2


U t 6

2
g (t) = sin 2 t 6 3
U t 6
= sin 2 t 6 3
U t 6

= sin 2 t 6
cos 23 cos 2 t 6
sin 2
3
U t 6
p
1 3
= 2
sin 2 t 6 2
cos 2 t 6
U t 6
p
1 3
= 2
sin 2t + 2
cos 2t :
jt !t =6

p p
) L (g) = e s=6
L 1
2
sin 2t + 2
3
cos 2t = -e s=6 2+ 3s
2(s2 +4)
.

Problem 3.2.54 f (t) = cos t U t 6

f (t) = cos t+ 6 6
U t 6
= cos t + 6 jt !t
:
p 6
3 1
But cos t + 6
= cos (t)
p
cos 6
sin t sin 6
= 2
cos (t) 2
sin (t)
) L cos t + 6
1+ 3s
= 2(s2 +1)

) L (f ) = L cos t + 6 jt
=e s=6
L cos t + 6
!t 6
p
) L (f ) = 1+ 3s
2
2(s +1)
e s=6
CHAPTER 3. THE LAPLACE TRANSFORM 75

3.3 Inverse Laplace Transform


1
f (t) = L (F (s)) if f L (f (t)) = F (s) :
Shortly.
1
f =L (F ) if f L (f ) = F:

3.3.1 Lecture Examples


Find the inverse Laplace transforms of the following functions:
s+8
Problem 3.3.1 H (s) = s2 +6s+34
e 4s

By completing square in the denominator


s+8 (s+3)+5 s 5
L 1 s2 +6s+34 = L 1 (s+3) 2
+25
=L 1 s2 +25
+ s2 +25 js !s+3
3t 1 s 5 3t
=e L s2 +25
+ s2 +25
=e (cos 5t + sin 5t) .
)L 1
(H) = e 3t
cos 5tjt !t 4 =e 3(t 4)
(cos 5 (t 4) + sin 5 (t 4)) U (t 4) .

Problem 3.3.2 M (s) = 4 + 3e 2s


7e 6s
+ 5s e 9s

1
L (M ) = 4 (t) + 3 (t 2) 7 (t 6) + 5U (t 9) .

s 3
Problem 3.3.3 K (s) = s(s2 6s+25)

C(s 3)
By partial fractions K (s) = s(s2 s 6s+25)
3
= As + B
s2 6s+25
+ s2 6s+25
) s 3 = A (s2 6s + 25) + Bs + C (s 3) s
at s = 0 : 3 = 25A ) A = 3=25;
at s = 3 : 0 = 16A + 3B ) B = 16=25;
coe¢ cients of s2 : 0=A+C = ) C = 3=25:
) K (s) = 1
25
3 1s + 4 (s 4
3)2 +16
+ 3 (s s 3
3)2 +16
1
= 25
3 1s + 4 s2 +16
4 s
+ 3 s2 +16 js !t 3

)L 1
(K) = 1
25
( 3 + 4e3t sin 4t + 3e3t cos 4t) .
(Compare with Problem 3.2.36.)
CHAPTER 3. THE LAPLACE TRANSFORM 76

3s+41
Problem 3.3.4 H (s) = (s 3)(s+7)
e 9s

We use partial fractions: (s 3s+41


3)(s+7)
= A
s 3
+ B
s+7
) 3s + 41 = A (s + 7) + B (s 3)
at s = 3 : 50 = 10A ) A = 5;
at s = 7 : 20 = 10B )B= 2:
) 3s+41
(s 3)(s+7)
= 5
s 3
2
s+7
)L 1 3s+41
(s 3)(s+7)
= 5e3t 2e 7t

)L 1
(H) = (5e3t 2e 7t
)jt !t 9
= 5e3(t 9)
2e 7(t 9)
U (t 9)

16+s2
Problem 3.3.5 G (s) = ln s2

Let L (g (t)) = G (s) = ln (16 + s2 ) ln (s2 )


) L (tg (t)) = dG(s)
ds
= ds d
(ln (16 + s2 ) 2 ln (s)) = 2s
16+s2
+ 2
s
) tg (t) = L 1 2s
16+s2
+ 2s = 2 2 cos 4t:

)L 1
(G) = g = 2 2 cos 4t
t
. (Compare with Problem 3.2.20.)

3.3.2 Classroom Exercises


20
Problem 3.3.6 G (s) = (s 8)(s+2)

20 A B
By partial fractions G (s) = (s 8)(s+2)
= s 8
+ s+2
) 20 = A (s + 2) + B (s 8)
at s = 2 : 20 = 10B )B= 2;
at s = 8 : 20 = 10A ) A = 2:
() ) G (s) = 2 s 1 8 1
2 s+2 :

)L 1
(G) = 2e8t 2e 2t
(Compare with Problem 3.2.29.)

3s 25
Problem 3.3.7 F (s) = s2 6s+25

By completing square in the denominator


F (s) = 3(s 3) 16
(s 3)2 +16
s
= 3 s2 +16 4
4 s2 +16 j s !t 3

)L 1
(F ) = (3 cos 4t 4 sin 4t) e3t . (Compare with Problem 3.2.4.)
CHAPTER 3. THE LAPLACE TRANSFORM 77

s 3 s
Problem 3.3.8 F (s) = s2 6s+13
+ s2 +4
e 7s
s 3 s 3 s
s2 6s+13
= (s 3)2 +4
= s2 +4 js !s 3

)L 1 s 3
s2 6s+13
=L 1 s
s2 +4 js !s 3
= e3t L 1 s
s2 +4
= e3t cos 2t
Also, L 1 s2s+4 e 7s = L 1 s2s+4 j = cos 2tjt !t 7
t !t 7

)L 1
(F ) = e3t cos 2t + cos 2 (t 7) U (t 7) .

s 1
Problem 3.3.9 F (s) = s2 2s+10
e 4s :

1 s 1 1 s 1 1 s
L s2 2s+10
=L (s 1)2 +9
=L s2 +9 js !s 1
t 1 s t
=eL s2 +9
= e cot 3t
)L 1
(F ) = et 4
cos 3 (t 4) U (t 4) .

4
Problem 3.3.10 Q (s) = (s 2)2 +16
e 3s

1 4 1 4
L (s 2)2 +16
=L s2 +16 js
= e2t sin 4t
!s 2

)L 1
(Q) = e2(t 3)
sin 4 (t 3) U (t 3) .

4s2 24s+36
Problem 3.3.11 G (s) = (s 2)2 (s 4)

4s2 24s+36 A B C
(s 2)2 (s 4)
= s 2
+ (s 2)2
+ s 4
(partial fractions)
) 4s 2
24s + 36 = A (s 2) (s 4) + B (s 4) + C (s 2)2
at s = 2 : 4 = 2B ) B = 2;
at s = 4 : 4 = 4C ) C = 1;
coe¢ cients of s2 : 4 = A + C; ) A = 4 C = 3:
) G (s) = s 2 (s 2)2 + s 4
3 2 1
) L 1 (G) = (3 2t) e2t + e4t .

s+4
Problem 3.3.12 F (s) = s2 +8s+41
e 2s

1 s+4 s+4 s
L s2 +8s+41
= L 1 (s+4) 2
+25
= L 1 s2 +25 js !s+4
s
= e 4t L 1 s2 +25 = e 4t cot 5t
)L 1 s+4
(F ) = L 1 s2 +8s+41 jt !t 2
= e 4(t 2) cos 5 (t 2) U (t 2) .
CHAPTER 3. THE LAPLACE TRANSFORM 78

3
Problem 3.3.13 F (s) = s2 4s+13
e 7s

1 3 1 3 1 3
L s2 4s+13
=L (s 2)2 +9
=L s2 +9 js !s 2
2t 1 3 2t
=e L s2 +9
=e sin 3t
)L 1
(F ) = (e2t sin 3t)jt!t 7
= e2(t 7)
sin 3 (t 7) U (t 7) .

6!
Problem 3.3.14 F (s) = s(s 3)7

1 6! 1 6!
L (s 3)7
=L s 7 js !s 3
= t6 e3t
Rt
)L 1
(F ) = L 1 1 6!
s (s 3)7
= z 6 e3z dz
0

s2 +4s+1
Problem 3.3.15 H (s) = (s+3)2 (s+2)
e 7s :

We use partial fractions:


s2 +4s+1 A B C
(s+3)2 (s+2)
= (s+3)2 + s+3 + s+2
) s2 + 4s + 1 = A (s + 2) + B (s + 3) (s + 2) + C (s + 3)2
at s = 3 : 2= A ) A = 2;
at s = 2 : 3 = C;
coe¢ cients of s2 : 1 = B + C = B 3; ) B = 4:
) s2 +4s+1
(s+3)2 (s+2)
= 2
(s+3)2
+ 4
s+3
3
s+2

)L 1 s2 +4s+1
(s+3)2 (s+2)
= 2te 3t
+ 4e 3t
3e 2t

)L 1
(H) = L 1 s2 +4s+1
(s+3)2 (s+2)
= (2te 3t
+ 4e 3t
3e 2t
)jt !t 7
jt !t 7

)L 1
(H) = 2 (t 7) e 3(t 7)
+ 4e 3(t 7)
3e 2(t 7)
U (t 7)

1 s
Problem 3.3.16 F (s) = cot 7

Let L (f ) = F (s) = cot 1 7s


) L (tf (t)) = dFds(s) = ds d
cot 1 7s
( 1) 1
= s 2 7
= s21 7
49
7
= s2 +49
(7) +1 49
+1
) tf (t) = L 1 s2 +49
7
= sin 7t
)L 1
(F ) = f (t) = sin 7t
t
.
CHAPTER 3. THE LAPLACE TRANSFORM 79

3.3.3 Revision Problems


Find the inverse Laplace Transforms of the following functions:
2s 29
Problem 3.3.17 F (s) = s2 +4s+29

By completing square in the denominator,


2(s+2) 25 s+2
F (s) = (s+2)2
+25
= 2 (s+2)2
+25
5 (s+2)52 +25 = s
2 s2 +25 5
5 s2 +25 js !s+2

)L 1
(F ) = ( 2 cos 5t 5 sin 5t) e 2t
(Compare with Problem 3.2.17.)

2s 7
Problem 3.3.18 W (s) = (s 1)(s+2)
e 6s

2s 7 A B
We use partial fractions: (s 1)(s+2) = s 1
+ s+2
) 2s 7 = A (s + 2) + B (s 1)
at s = 1 : 9 = 3A ) A = 3;
at s = 2 : 3 = 3B ) B = 1;
) 2s 7
(s 1)(s+2)
= 3s 11 + 1
s+2
)L 1 2s 7
(s 1)(s+2)
= 3et + e 2t

)L 1
(W ) = L 1 2s 7
(s 1)(s+2)
= (e 2t
3et )jt !t 6
jt !t 6

)L 1
(W ) = e 2(t 6)
3et 6
U (t 6)

s+2
Problem 3.3.19 F (s) = (s+2)2 9
e 4s

1 s+2 1 s
L (s+2)2 9
=L s 2 9 js !s+2
2t 1 s 2t
=e L s2 9
=e cosh 3t
)L 1
(F ) = (e 2t
cosh 3t)jt!t 4
=e 2(t 4)
cosh 3 (t 4) U (t 4) .

X
1
1 3ns
Problem 3.3.20 1 e 3s = e (in…nite geometric series)
n=0

3Ks
X1
K
1 e 3ns
1 e 3s = e (K is a positive integer, …nite geometric series),
n=0

X
1 X1
K
)L 1 1 e 3Ks
1 1
1 e 3s = (t 3n) , L 1 e 3s = (t 3n) .
n=0 n=0
CHAPTER 3. THE LAPLACE TRANSFORM 80

Problem 3.3.21 Find the inverse Laplace Transforms of the following


functions:
2 1
1. A (s) = s(s2 +4)
: Answer: 2
(1 cos 2t)

1 1 5t 5t
2. B (s) = s(s+5)2
: Answer: 25
(1 e 5te )

10
3. C (s) = (s 9)(s+1)
e 7s : Answer: e9(t 7)
e (t 7)
U (t 7)

s2 6s+2 t
4. D (s) = (s+1)(s 2)2
: Answer: e 2te2t

4
5. E (s) = s2 4s+20
e 3s :
Answer: sin 4 (t 3) e2(t 3)
U (t 3)

3s 12 s 7
6. F (s) = s2 8s 9
+ (s 7)2 +9
e 2s :
Answer: 3e4t cosh 5t + e7(t 2)
cos 3 (t 2) U (t 2)

2 s
7. H (s) = (s+4)3
e 7s + s2 +9
e s
:

Answer: (t 7)2 e 4(t 7)


U (t 7) + cos 3 (t ) U (t )

s2 +7s+6
8. J (s) = (s 3)(s+3)2
e 4s +e 2s
:
s 5
9. K (s) = s2 10s+29
e 3s

s+2 s 4
10. N (s) = s2 +4s+5
e 6s + s2 8s+80
e 3s :

2s
R1 4
11. P (s) = 3 + e + s v 2 +16
dv

d 4
R1 4 3s
12. R (s) = ds s2 +16
+ s v 2 +16
dv e :
sin 4(t 3)
Answer: (t 3) sin 4 (t 3) + t 3
U (t 3)

13. T (s) = 7s e 9s
+ 5 + 4e 2s
:

s2 +5s 6
14. V (s) = (s 2)2 (s+6)
:
CHAPTER 3. THE LAPLACE TRANSFORM 81

6s2 +7s 2
15. W (s) = (s 2)(s+1)2
e 4s :

s 4
16. X (s) = s2 8s+25
e 7s :
13
17. Y (s) = (s+2)(s2 +9)
e 6s :

7!
18. Z (s) = (s 3)8
e 4s :

s+2
19. A (s) = s2 2s+2
:

6s2 +34s+76
20. C (s) = (s 3)(s2 +8s+25)

8+8s2
21. D (s) = (s+1)2 (s 3)
:

s 7
22. E (s) = s2 +s 2
e 5s +e 2s
:

4s2 24s+36
23. G (s) = (s 2)2 (s 4)
:

s 2
24. H (s) = s2 4s 5
e 6s +e 4s
:
5s+5
25. J (s) = (s 1)(s2 +4s+5)
:

6s s 4
26. K (s) = 1 2e + s2 8s+20
e 3s :
8!
27. N (s) = 1 + (s+4)9
e 2s :

1
28. P (s) = 2
tan (s)
s+7 1 e 7t
29. R (s) = ln s
: Answer: t

s
30. Q (s) = ln s+7
:
s s+7
31. Y (s) = ln s+7
+ ln s
:

s2 9
32. T (s) = ln s2
:

33. V (s) = ln (s2 + 9) 2 ln s:

s2 1 3s
34. W (s) = ln s2 +1
e
CHAPTER 3. THE LAPLACE TRANSFORM 82

4s
Problem 3.3.22 F (s) = (s2 +4)2

Since, F (s) = 2 s22+4 s2s+4 (multiplication), then


1 1 2 1 s
L (F ) = 2L s2 +4
?L s2 +4
(convolution)
Zt
)L 1
(F ) = 2 sin 2t ? cos 2t = 2 sin (2z) cos 2 (t z) dz
0
Zt
1 z=t
= sin (2t) + sin (4z 2t) dz = z sin (2t) 4
cos (4z 2t) z=0
0
1
= t sin (2t) 0 4
cos (2t) + 41 cos ( 2t)

)L 1
(F ) = t sin (2t) , because cos ( 2t) = cos (2t) :

2s2
Problem 3.3.23 F (s) = (s2 +4)2

Since, F (s) = 2 s2s+4 s2s+4 (multiplication), then


1 1 s 1 s
L (F ) = 2L s2 +4
?L s2 +4
(convolution)
Zt
) L 1 (F ) = 2 cos 2t ? cos 2t = 2 cos (2z) cos 2 (t z) dz
0
Zt
2 z=t
= 2
cos (2t) + cos (4z 2t) dz = z cos (2t) + 41 sin (4z 2t) z=0
0
= t cos (2t) 0 + 14 sin (2t) 1
4
sin ( 2t)

)L 1
(F ) = t cos (2t) + 12 sin (2t) , because sin ( 2t) = sin (2t) :
CHAPTER 3. THE LAPLACE TRANSFORM 83

3.4 Fredholm Integral Equations


Use the Convolution Theorem to solve the following
Fredholm integral equations:

3.4.1 Lecture Examples


Rt
Problem 3.4.1 f (t) = e3t 0
f (z) e4(t z)
dz

f (t) = e3t (f ? e4t ) :


We denote L (f ) by F (s), and apply the Laplace transform to both sides.
We get by Convolution Theorem:
F = s 1 3 F: s 1 4
) F 1 + s 14 = s 13
) F ss 43 = s 1 3
) F = (ss 3)4 2 = (s(s 3)3)21 = s 1 3 (s 13)2 = 1s s12 js !s 3
) f (t) = L 1
(F ) = (1 t) e3t .

Rt
Problem 3.4.2 f (t) = e3t cos t + 3e3t sin t 3 0
f (z) e3(t z)
dz

f (t) = e3t cos t + 3e3t sin t 3 (f ? e3t ) :


We denote L (f ) by F (s), and apply the Laplace transform to both sides.
We get by Convolution Theorem:
F = (s s3)32 +1 + 3 (s 3)1 2 +1 3F: s 1 3
)F 1+ 3
s 3
= (s 3)s 2 +1 )F s
s 3
= s
(s 3)2 +1
)F = s 3
(s 3)2 +1
= s2s+1 j
s !s 3

) f (t) = L 1 3t
(F ) = e cos t .
CHAPTER 3. THE LAPLACE TRANSFORM 84

3.4.2 Classroom Exercises


Rt
Problem 3.4.3 f (t) = 2 + 3 sinh t 0
f (z) sinh (t z) dz

f (t) = 2 + 3 sinh t f ? sinh t: We denote L (f ) by F (s), and apply


the Laplace transform to both sides. We get by Convolution Theorem:
) F = 2s + s23 1 F: s21 1
) F 1 + s21 1 = 2s + s23 1 = 2ss(s22+3s
2
1)

)F s2
s2 1
= 2s2 +3s 2
s(s2 1)
)F = 2s2 +3s 2
s3
= 2 1s + 3 s12 2
s3

) f (t) = L 1
(F ) = 2 + 3t t2 :

Rt
Problem 3.4.4 f (t) = e3t e2t + 0
f (z) e2(t z)
dz

f (t) = e3t e2t + f ? e2t :


Denote L (f ) by F (s).
) F = s 1 3 s 1 2 + F: s 1 2
) F 1 s 12 = s 13 s 12 = (s 2) (s 3)
(s 3)(s 2)
) F ss 32 = (s 3)(s
1
2)

)F = s 2
s 3
1
(s 3)(s 2)
= 1
(s 3)2
= 1
s 2 js !s 3

) f (t) = L 1
(F ) = te . 3t

Rt
Problem 3.4.5 f (t) = 8te4t 4 0
f (z) e4(t z)
dz

f (t) = 8te4t 4 (f ? e4t ) :


Denote L (f ) by F (s).
) F = (s 84)2 4F: s 1 4
)F 1+ 4
s 4
= 8
(s 4)2
)F s
s 4
= 8
(s 4)2
)F = 8
s(s 4)
= As + sB4
(partial fractions)
) 8 = A (s 4) + Bs
at s = 0 : 8 = 4A ) A = 2;
at s = 4 : 8 = 4B ) B = 2:
)F = s+s 42 2

) f (t) = L 1
(F ) = 2 + 2e4t .
CHAPTER 3. THE LAPLACE TRANSFORM 85

3.4.3 Revision Problems


Rt
Problem 3.4.6 f (t) = 3t2 + et 0
f (z) et z
dz

f (t) = 3t2 + et (f ? et ) :
Denote L (f ) by F (s), and apply the Laplace transform to both sides of
the equation. Then we get by the Convolution Theorem:
F = s63 + s 1 1 F: s 1 1
) F 1 + s 1 1 = s63 + s 1 1 i.e., F s s 1 = s63 + s 1 1
) F = s s 1 s63 + s 1 1 = 6(ss4 1) + s(ss 11) = s63 s64 + 1s
) f (t) = L 1
(F ) = 3t2 t3 + 1 .

Rt
Problem 3.4.7 3f (t) = e7t e4t 9 0
f (z) e7(t z)
dz

3f (t) = e7t e4t 9 (f ? e7t ) :


Denote L (f ) by F (s).
) 3F = s 1 7 s 1 4 9F: s 1 7
) F 3 + s 9 7 = s 1 7 s 1 4 = s(s 47)(s
(s 7)
4)

)F 3(s 4)
s 7
= 3
(s 7)(s 4)

)F = s 7
s 4
1
(s 7)(s 4)
= 1
(s 4)2
)F = 1
s2 js !s 4

) f (t) = L 1 4t
(F ) = te .

Rt
Problem 3.4.8 f (t) = e2t + 0
f (z) et z
dz:

Rt
Problem 3.4.9 f (t) = cos t + 10 0
f (z) sin (t z) dz:

Rt
Problem 3.4.10 f (t) = e5t sinh 3t 6 0
f (z) e8(t z)
dz:

Rt
Problem 3.4.11 f (t) = et cos t + 3 0
f (z) e 2(t z)
dz:
CHAPTER 3. THE LAPLACE TRANSFORM 86

Rt
Problem 3.4.12 f (t) = 4 sin 5t 6 f (z) cos 5 (t z) dz:
0

Rt
Problem 3.4.13 f (t) = e3t e2t + 0
f (z) e2(t z)
dz:

Rt
Problem 3.4.14 f (t) = 8e3t 2 0
f (z) cos (t z) dz:

Rt
Problem 3.4.15 f (t) = et 9 0
f (z) e7(t z)
dz:

Rt
Problem 3.4.16 f (t) = e2t + sin 2t 4 0
f (z) e2(t z)
dz:

Rt
Problem 3.4.17 f (t) = 4tet + 2 0
f (z) et z
dz:

Rt
Problem 3.4.18 f (t) = sinh 2t 4 0
f (z) e2(t z)
dz:

Rt
Problem 3.4.19 f (t) = sin 3t 6 0
f (z) cos 3 (t z) dz:

Rt
Problem 3.4.20 f (t) = cos 2t + sin 2t 4 0
f (z) e2(t z)
dz:
CHAPTER 3. THE LAPLACE TRANSFORM 87

3.5 Laplace Transform and Initial-value Prob-


lems:
Use the Laplace transform to solve the following initial-
value problems:

3.5.1 Lecture Examples


dy dy dy
Problem 3.5.1 dx
8 dx + 16y = 6te4t ; y (0) = 1; dx
(0) = 4

Let L (y) = Y (s) : ) L dxdy


= sY y (0) = sY 1:
) L (y ) = s Y sy (0) y (0) = s2 Y s 4:
00 2 0

Apply the Laplace transform to both sides:


) s2 Y s 4 8 (sY 1) + 16Y = (s 64)2 :
) (s2 8s + 16) Y = (s 64)2 + s + 4 8
) (s 4)2 Y = 6
(s 4)2
+ (s 4)
)Y = 6
(s 4)2 (s 4)2
+ (s 4)
(s 4)2
= 3!
(s 4)4
+ 1
s 4
:
) y (t) = L 1
(Y ) = L 1 3!
s 4 js !s 4
+ 1
s 4
= t3 e4t + e4t = (t3 + 1) e4t :

Problem 3.5.2 y 00 + 4y 0 + 3y = 6e 3t
; y (0) = 1; y 0 (0) = 2

Let L (y) = Y (s) : ) L (y 0 ) = sY y (0) = sY 1;


L (y ) = s Y sy (0) y 0 (0) = s2 Y s + 2:
00 2

We apply the Laplace transform to both sides:


s2 Y s + 2 + 4sY 4 + 3Y = s+3 6
: ) (s2 + 4s + 3) Y = 6
s+3
+s+2
) (s + 3) (s + 1) Y = 6+(s+3)(s+2)
2
s+3
= s +5s+12
s+3
) Y = (s+3)
s2 +5s+12
2
(s+1)
= A
s+3
+ B
(s+3)2
+ C
s+1
) s2 + 5s + 12 = A (s + 3) (s + 1) + B (s + 1) + C (s + 3)2
at s = 3 : 6 = 2B ) B = 3;
at s = 1 : 8 = 4C ) C = 2;
coe¢ cients of s2 : 1 = A + C; ) A = 1 C = 1:
) Y (s) = s+3 1 3
(s+3)2
+ 2
s+1

) y (t) = L 1
(Y ) = e 3t
3te 3t
+ 2e t
= (1 + 3t) e 3t
+ 2e t
.
CHAPTER 3. THE LAPLACE TRANSFORM 88

Problem 3.5.3 y 00 2y 0 + y = 6e2t cos t; y (0) = 0; y 0 (0) = 3

Let L (y) = Y (s) : ) L (y 0 ) = sY y (0) = sY;


L (y 00 ) = s2 Y sy (0) y 0 (0) = s2 Y 3:
We apply the Laplace transform to both sides:
s 6(s 2)
s2 Y 3 2sY + Y = 6 s2 +1 js
= (s 2)2 +1
:
!s 2
) (s2 2s + 1) Y = 6s 12
s2 4s+5
+3
3(s 1)2
) (s 1)2 Y = 6s 12+3s2 12s+15
s2 4s+5
= 3s2 6s+3
2
s 4s+5
= s2 4s+5

) Y (s) = s2
3
4s+5
= (s 2)3 2 +1 = 3 1
s2 +1 js !s 2

) y (t) = L 1
(Y ) = 3e2t sin t .

3.5.2 Classroom Exercises


Problem 3.5.4 y 00 6y 0 + 9y = 6t2 e3t ; y (0) = 1; y 0 (0) = 3

Let L (y) = Y (s) : ) L (y 0 ) = sY y (0) = sY 1;


L (y ) = s Y sy (0) y 0 (0) = s2 Y s 3:
00 2

We apply the Laplace transform to both sides:


s2 Y s 3 6sY +6+9Y = (s 123)3 : ) (s2 6s + 9) Y = s 3+ (s 123)3
(s 3)2 Y = s 3+ 12
(s 3)3
) Y = s 1 3 + (s 123)5 = 1
s 3
+ 12 4!
4! s5 js !s 3

) y (t) = L 1
(Y ) = e3t + 12 t4 e3t = 1 + 21 t4 e3t .

Problem 3.5.5 y 00 6y 0 + 13y = 13; y (0) = 2; y 0 (0) = 3

Let L (y) = Y (s) : ) L (y 0 ) = sY y (0) = sY 2;


L (y 00 ) = s2 Y sy (0) y 0 (0) = s2 Y 2s 3:
Apply the Laplace transform to both sides:
) s2 Y 2s 3 6 (sY 2) + 13Y = 13 s
:
) (s 2 13
6s + 13) Y = s + 2s 9 = 2s2 9s+13
s
) Y = s(s2 6s+13) = s + s2 6s+13 + s2 6s+13
2s2 9s+13 A B C(2s 6)

) 2s2 9s + 13 = A (s2 6s + 13) + Bs + 2Cs (s 3)


at s = 0 : 13 = 13A ) A = 1;
at s = 3 : 4 = 4A + 3B ) B = 0;
coe¢ cients of s : 2
2 = A + 2C; ) C = 1 12 A = 12 :
Y (s) = 1
s
+ s 3
s2 6s+13
= 1
s
+ s 3
(s 3)2 +4
) y (t) = L 1
(Y ) = 1 + e3t cos 2t .
CHAPTER 3. THE LAPLACE TRANSFORM 89

Problem 3.5.6 y 00 4y = 16e2t ; y (0) = 3; y 0 (0) = 6

Let L (y) = Y (s) : ) L (y 0 ) = sY y (0) = sY 3;


L (y 00 ) = s2 Y sy (0) y 0 (0) = s2 Y 3s 6:
We apply the Laplace transform to both sides:
) s2 Y 3s 6 4Y = s162 :
) (s2 4) Y = s162 + 3s + 6 = 3ss +4
2
2
) Y = (s 2)(s2 4) = (s 2)2 (s+2) = sA2 + (s B2)2 + s+2
3s2 +4 3s2 +4 C

) 3s2 + 4 = A (s 2) (s + 2) + B (s + 2) + C (s 2)2
at s = 2 : 16 = 4B ) B = 4;
at s = 2 : 16 = 16C ) C = 1;
coe¢ cients of s2 : 3 = A + C; ) A = 2:
) Y (s) = s 2 + (s 2)2 + s+2
2 4 1

) y (t) = L 1
(Y ) = 2e2t + 4te2t + e 2t
.

Problem 3.5.7 y 00 6y 0 + 8y = 4e2t ; y (0) = 4; y 0 (0) = 8 :

Let L (y) = Y (s) : ) L (y 0 ) = sY y (0) = sY 4;


L (y 00 ) = s2 Y sy (0) y 0 (0) = s2 Y 4s 8:
We apply the Laplace transform to both sides:
s2 Y 4s 8 6sY +24+8Y = 4
s 2
: ) (s2 6s + 8) Y = 4
s 2
+4s 16
) (s 2) (s 4) Y = 4+(s 2)(4s 16)
s 2
= 4s2 24s+36
s 2

) Y (s) = 4s2 24s+36


(s 2)2 (s 4)
= A
s 2
+ B
(s 2)2
+ C
s 4
(partial fractions)

) 4s2 24s + 36 = A (s 2) (s 4) + B (s 4) + C (s 2)2


at s = 2 : 4 = 2B ) B = 2;
at s = 4 : 4 = 4C ) C = 1;
2
coe¢ cients of s : 4 = A + C; ) A = 4 C = 3:
) Y (s) = s 3 2 (s 22)2 + s 1 4
and so
) y (t) = L 1 (Y ) = (3 2t) e2t + e4t .
CHAPTER 3. THE LAPLACE TRANSFORM 90

3.5.3 Revision Problems


Problem 3.5.8 y 00 + 5y 0 + 6y = 2e 3t
; y (0) = 1; y 0 (0) = 4

Let L (y) = Y (s) : ) L (y 0 ) = sY y (0) = sY 1;


L (y 00 ) = s2 Y sy (0) y 0 (0) = s2 Y s + 4:
We apply the Laplace transform to both sides:
2
s2 Y s + 4 + 5sY 5 + 6Y = s+3 :
) (s + 5s + 6) Y = s + 1 s+3
2 2

) (s + 3) (s + 2) Y = (s+3)(s+1) 2
s+3
= s2 +4s+1
s+3

)Y = s2 +4s+1
(s+3)2 (s+2)
= A
(s+3)2
+ B
s+3
+ C
s+2
(partial fractions)
) s + 4s + 1 = A (s + 2) + B (s + 3) (s + 2) + C (s + 3)2
2

at s = 3 : 2= A ) A = 2;
at s = 2 : 3 = C;
coe¢ cients of s2 : 1 = B + C = B 3; ) B = 4:
)Y = s2 +4s+1
(s+3)2 (s+2)
= 2
(s+3)2
4
+ s+3 3
s+2

) y (t) = L 1
(Y ) = 2te 3t
+ 4e3t
3e 2t
. (See also Problem (2.3.12).)

Problem 3.5.9 y 00 + ! 2 y = 0, y (0) = 1 , y 0 (0) = 0:

Final Answer y = cos !t

Problem 3.5.10 y 00 ! 2 y = 0, y (0) = 0 , y 0 (0) = !:

Final Answer y = sinh !t

Problem 3.5.11 y 00 ! 2 y = 0, y (0) = 1 , y 0 (0) = 0:

Final Answer y = cosh !t:

Problem 3.5.12 y 00 + y 0 6y = 30 , y (0) = 0 , y 0 (0) = 30:

Final Answer y = 9e2t 4e 3t


5:
CHAPTER 3. THE LAPLACE TRANSFORM 91

Problem 3.5.13 y 00 8y 0 + 16y = e3t , y (0) = 1 , y 0 (0) = 4:

Problem 3.5.14 y 00 y0 6y = 15e 2t


; y (0) = 0; y 0 (0) = 3:

Problem 3.5.15 y 00 + 2y 0 + y = 80e3t ; y (0) = 8; y 0 (0) = 8:

Problem 3.5.16 y 00 + 5y 0 + 6y = 2e 3t
; y (0) = 1; y 0 (0) = 4:

Problem 3.5.17 y 00 2y 0 + y = 2et cos t; y (0) = 1; y 0 (0) = 1 .

Problem 3.5.18 y 00 y 0 6y = 13 cos 3t 13 sin 3t; y (0) = 2; y 0 (0) = 4.

Problem 3.5.19 y 00 y0 2y = 3e t ; y (0) = 6; y 0 (0) = 7:

Problem 3.5.20 y 00 + 4y 0 12y = 8e2t ; y (0) = 1; y 0 (0) = 3:

Problem 3.5.21 y 00 4y 0 + 4y = te3t ; y (0) = 0; y 0 (0) = 1:

Problem 3.5.22 y 00 6y 0 + 5y = 8et ; y (0) = 3; y 0 (0) = 1:

Problem 3.5.23 y 00 + y 0 2y = 9e 2t
; y (0) = 0; y 0 (0) = 3.

Problem 3.5.24 y 00 2y 0 + 10y = 10e2t ; y (0) = 0; y 0 (0) = 7 .

Problem 3.5.25 y 00 9y = 6e 3t
; y (0) = 1; y 0 (0) = 4 .

Problem 3.5.26 y 00 + y 0 6y = 6e t ; y (0) = 1; y 0 (0) = 1:


CHAPTER 3. THE LAPLACE TRANSFORM 92

3.6 Laplace Transform and Systems of Dif-


ferential Equations
Use the Laplace transform to solve the following initial-
value problems (systems of di¤erential equations):

3.6.1 Lecture Examples


dx dy t
Problem 3.6.1 dt
+x y = 0; 6x + dt
= 6e ; x (0) = 0; y (0) = 3 :

Let L (x) = X (s) : ) L dx


dt
= sX x (0) = sX;
let L (y) = Y (s) : ) L dt = sY y (0) = sY 3:
dy

) (s + 1) X Y = 0; When we multiply by s,

s2 + s X sY = 0; (3.1)

6
6X + sY :3= (3.2)
s+1
We add the two equations (3.1) and (3.2), and get:
6
(s2 + s 6) X = 3 + s+1 = 3s+3+6
s+1
= 3s+9
s+1
(s + 3) (s 2) X = 3(s+3)
s+1
) X = 3
(s 2)(s+1)
= sA2 + s+1
B
(partial fractions)
) 3 = A (s + 1) + B (s 2)
at s = 1 : 3 = 3B ) B = 1;
at s = 2 : 3 = 3A ) A = 1;
) X = s 2 s+1
1 1
) x (t) = L 1 (X) = e2t e t .
dx dx
As dt
+x y = 0; then y = dt
+ x = 2e2t + e t
+ e2t e t
= 3e2t .

dy dx
Problem 3.6.2 x dt
= 1; dt
2x + y = t 2,
x (0) = 2; y (0) = 0 .

Let L (x) = X (s) : )L dx


dt
= sX x (0) = sX 2:
Let L (y) = Y (s) : )L dy
dt
= sY y (0) = sY:

1
)X sY = ; (3.3)
s
1 2
sX 2 2X + Y = s2 s
:
CHAPTER 3. THE LAPLACE TRANSFORM 93

1 2 1 2s+2s2
This gives (s 2) X + Y = s2 s
+2= s2
, and so,
1 2s + 2s2
s2 2s X + sY = : (3.4)
s
2s+2s2 2s(s 1)
We add (3.3) and (3.4): (s2 2s + 1) X = s
= s
) (s 1)2 X = 2 (s 1) ) X = s 2 1 ) x (t) = L (X) = 2e . 1 t

As dx
dt
2x + y = t 2; then y = t 2 dx dt
+ 2x
) y = t 2 2e + 4e
t t
) y (t) = t 2 + 2et .

3.6.2 Classroom Exercises


dx dy
Problem 3.6.3 dt
+y = 2t; x 2y + dt
=t;
x (0) = 0; y (0) = 1 :

Let L (x) = X (s) : ) L dxdt


= sX x (0) = sX:
Let L (y) = Y (s) : ) L dt = sY
dy
y (0) = sY 1:
Apply the Laplace transform to all sides:
2
) sX + Y = ; (3.5)
s2
Also, X 2Y + sY 1 = s12 ;
s2 +1
that is, X + (s 2) Y = s12 + 1 = s2
We multiply by s :
s3 + s
) sX + s2 2s Y = : (3.6)
s2
We add equations (3.5) and (3.6), and get:
3
(s2 2s + 1) Y = s +s s2
2
:
) Y = s2 (s 1)2 = s + s2 + sC 1 + (s D1)2 (partial fractions)
s3 +s 2 A B

) s3 + s 2 = As (s 1)2 + B (s 1)2 + Cs2 (s 1) + Ds2 :


at s = 0 : 2 = B;
at s = 1 : 0 = D;
coe¢ cients of s : 1 = A 2B; ) A = 1 + 2B = 3;
3
coe¢ cients of s : 1 = A + C; ) C = 1 A = 4:
) Y (s) = 3 1s 2 s12 + 4 s 1 1
) y (t) = 3 2t + 4et :
) x = t 2y + dy dt
= t 2( 3 2t + 4et ) + ( 2 + 4et ) :
) x (t) = 4 + 3t 4et :
CHAPTER 3. THE LAPLACE TRANSFORM 94

dx dy
Problem 3.6.4 dt
6x 3y = 0; 3x + dt
= 2te3t ;
x (0) = 1; y (0) = 1:

L dx
dt
= sX x (0) = sX 1; L dy
dt
= sY y (0) = sY + 1:
Apply the Laplace transform:
) sX 1 6X 3Y = 0; 3X + sY + 1 = 2
(s 3)2
: This gives

s2 X 6sX 3sY = s; (3.7)


6
9X + 3sY = 3: (3.8)
(s 3)2
We add the two equations (3.7) and (3.8), and get:
(s2 6s + 9) X = s 3 + (s 63)2 ) (s 3)2 X = s 3+ 6
(s 3)2
)X= 1
s 3
+ 6
(s 3)4
) x (t) = L 1
(X) = (1 + t3 ) e3t .
As dx
dt
6x 3y = 0; then y = 13 dx dt
2x
) y = 3 (3 + 3t + 3t ) e
1 2 3 3t 3
2 (1 + t ) e 3t

) y (t) = ( 1 + t2 t3 ) e3t .

Problem 3.6.5 4x 2 dy
dt
= 6e3t 8te3t ; dx
dt
3x + 2y = 8 ;
x (0) = 0; y (0) = 3 :

dy
L dx
dt
= sX x (0) = sX and L dt
= sY y (0) = sY 3:
We apply the Laplace transform: ) 4X 2 (sY 3) = s 6 3 8
(s 3)2
;

6s 26 42s 6s2 80
) 4X 2sY = 6+ = : (3.9)
(s 3)2 (s 3)2
Also, (s 3) X + 2Y = 8s . We multiply by s, and get
s2 3s X + 2sY = 8: (3.10)
We add the two equations (3.9) and (3.10), and get:
6s 26 2(s 3)2 +6s 26 2s2 6s 8 2(s2 3s 4)
(s2 3s 4) X = 2 + (s 3)2
= (s 3)2
= (s 3)2
= (s 3)2

)X= 2
(s 3)2
) x (t) = L 1
(X) = 2te3t .
dx dx
As dt
3x + 2y = 8; then 2y = 8 dt
+ 3x
) 2y = 8 2e3t 6te3t + 6te3t ) y (t) = 4 e3t .
CHAPTER 3. THE LAPLACE TRANSFORM 95

3.6.3 Revision Problems


Use the Laplace transform to solve the following initial-
value problems (systems of di¤erential equations):
dy dx
Problem 3.6.6 x + dt
= 2et ; dt
y=0;
x (0) = 0; y (0) = 1 :

Let L (x) = X (s) : ) L dx


dt
= sX x (0) = sX:
Let L (y) = Y (s) : ) L dt = sY y (0) = sY 1:
dy

We apply the Laplace transform to all sides: ) X + sY 1 = 2


s 1
;
s 32
) X + sY = 1 : = (3.11)
s 1 s 1
Also, sX Y = 0. We multiply by s, and get
s2 X sY = 0: (3.12)
We add the two equations (3.11) and (3.12), and get:
(s2 + 1) X = ss 13 ) X = (s 1)(s
s 3 A B Cs
2 +1) = s 1 + s2 +1 + s2 +1 (partial
fractions)
) s 3 = A (s2 + 1) + B (s 1) + Cs (s 1)
at s = 1 : 2 = 2A ) A = 1;
at s = 0 : 3=A B ) B = A + 3 = 2;
coe¢ cients of s2 : 0 = A + C; ) C = A = 1:
X = s 1 + 2 s2 +1 + s2 +1 ) x (t) = L 1 (X) = et + 2 sin t + cos t .
1 1 s

As dx
dt
y = 0; then y = dx
dt
) y (t) = et + 2 cos t sin t .

Problem 3.6.7 x (0) = 1 , y (0) = 0 ,


dx dy
+ y = 0; x+ = 2 cos t:
dt dt
(Answer: x = cos t, y = sin t.)

Problem 3.6.8 x (0) = 0 , y (0) = 1 ,


dx dx dy
2y = 6e3t sin t; 10x 3 +2 = 0:
dt dt dt
(Answer: x = 2e3t sin t, y = e3t cot s.)
CHAPTER 3. THE LAPLACE TRANSFORM 96

Problem 3.6.9 x (0) = 1 , y (0) = 0 ,


dx
3x + 2y = 0;
dt
dy
x+y = 0:
dt
(Answer: x = e2t sin t + e2t cos t, y = e2t sin t.)

Problem 3.6.10 x (0) = 0 , y (0) = 1 ,


dx
2x 3y = 0;
dt
dy
x+ = 3te2t + 2e2t :
dt
(Answer: x = 3te2t , y = e2t .)

Problem 3.6.11 x (0) = 1 , y (0) = 0 ,


dx dy
= 2e2t ;
dt dt
dx
3x + y = 0:
dt
(Answer: x = (1 + t) e2t , y = te2t .)

Problem 3.6.12 x (0) = 0 , y (0) = 1 ,


dx
+ 3x y = 0;
dt
dx dy 3t
3 + = 9te :
dt dt
(Answer: x = te 3t , y = e 3t .)

Problem 3.6.13 x (0) = 0 , y (0) = 1 ,


dx
3x y = t2 e3t ;
dt
dy
2x + 3y = 0:
dt
(Answer: x = te3t , y = e3t t2 e3t .)
CHAPTER 3. THE LAPLACE TRANSFORM 97

Problem 3.6.14 x (0) = 0 , y (0) = 3 ,


dx
3 y = 12e4t sin t;
dt
dy
3x + = 12e4t cos t:
dt
(Answer: x = e4t sin t, y = 3e4t cos t.)

Problem 3.6.15 x (0) = 0 , y (0) = 3 ,


dy
6x + = 0;
dt
dx
3 + x 2y = sin 2t:
dt
(Answer: x = sin 2t, y = 3 cos 2t.)

Problem 3.6.16 x (0) = 1 , y (0) = 4 ,


dx
+ 4x y = 3e2t ;
dt
dy
12x = 2e2t :
dt
2t
(Answer: x = (1 + t) e , y = (4 + 6t) e2t .)

Problem 3.6.17 x (0) = 0 , y (0) = 1 ,


dy
4x = 2e2t 16te2t ;
dt
dx
4x y = 8te2t 5e2t :
dt
2t
(Answer: x = 4te , y = e2t .)

Problem 3.6.18 x (0) = 1 , y (0) = 0 ,


dx
+ y = 0;
dt
dy
x = 0:
dt
(Answer: x = cos t, y = sin t.)
Appendix A

Table of Famous Integrals

R 1
R p R
xr dx = xr+1 (r 6= 1) p dx = ln x + x2 a2 sec2 x dx = tan x
r+1 x2 a2

R 1
R p R
dx = ln (Cx) p dx = ln x + x 2 + a2 csc2 x dx = cot x
x x2 +a2
R R R
ex dx = ex + C sin x dx = cos x + C sec x tan x dx = sec x
R dx 1 1 x
R R
x2 +a2
= a
tan a
+C cos x dx = sin x + C csc x cot x dx = csc x
R dx 1 x a
R R
x2 a2
= 2a
ln x+a
+C sec x dx = ln jsec x + tan xj tan x dx = ln jsec xj
R 1
R R
p dx = sin x
+C csc x dx = ln jcsc x + cot xj cot x dx = ln jsin xj
a2 x2 a

Trigonometric Formulas
sin x 1 2 tan x
tan x = cos x
sec x = cos x
tan 2x = 1 tan2 x

cos x 1 1 cot2 x 1
cot x = sin x
= tan x
csc x = sin x
cot 2x = 2 cot x

d
dx
sin x = cos x sin2 x + cos2 x = 1 sin x cos x = 21 sin 2x

d
dx
tan x = sec2 x sec2 x = 1 + tan2 x sin2 x = 12 (1 cos 2x)

d
dx
sec x = sec x tan x csc2 x = 1 + cot2 x cos2 x = 21 (1 + cos 2x)

cos 2x = cos2 x sin2 x = 1 2 sin2 x = 2 cos2 x 1

98
Appendix B

Assignments

99
APPENDIX B. ASSIGNMENTS 100

B.1 Assignment 1 on Separable and Homo-


geneous Di¤erential Equations
Assignment 1 - Mathematics 3
Separable and Homogeneous Di¤erential Equations

Name: Registration No.:

Specialization: Date: / 20

Assignment Problem B.1.1 Solve the following di¤erential equation:


dy
(y + yx2 ) dx = y 2 + 1.
APPENDIX B. ASSIGNMENTS 101

Name: Registration No.:

Assignment Problem B.1.2 Solve the following di¤erential equation:


dy
2y cos2 2x dx = e4y ; y 8 = 0:
APPENDIX B. ASSIGNMENTS 102

Name: Registration No.:

Assignment Problem B.1.3 Solve the following di¤erential equation:


dy
(2y 3 x 6x3 y) dx = 3y 4 12x2 y 2 + 9x4 :
APPENDIX B. ASSIGNMENTS 103

Name: Registration No.:

Assignment Problem B.1.4 Solve the following di¤erential equation:


dy
3x2 dx = 9x2 + 3xy + y 2 :
APPENDIX B. ASSIGNMENTS 104

Final Answer of Assignment Problem B.1.1:


Separable di¤erential equation
1
y 2 + 1 = Ce2 tan x :
Final Answer of Assignment Problem B.1.2:
Separable di¤erential equation
(4y + 1) e 4y = 5 4 tan 2x:
Final Answer of Assignment Problem B.1.3:
Homogeneous di¤erential equation
y 4 6x2 y 2 + 9x4 = C 2 x6 :
Final Answer of Assignment Problem B.1.4:
Homogeneous di¤erential equation
y = 3x tan (ln (Cx)) :
APPENDIX B. ASSIGNMENTS 105

B.2 Assignment 2 on Exact Di¤erential Equa-


tions
Assignment 2 - Mathematics 3
Exact Di¤erential Equations

Name: Registration No.:

Specialization: Date: / 20

Assignment Problem B.2.1 Solve the following di¤erential equation:


(y 3 y 2 sin x 6e3x ) dx + (3xy 2 + 2y cos x + tan y) dy = 0:
APPENDIX B. ASSIGNMENTS 106

Name: Registration No.:

Assignment Problem B.2.2 Solve the following di¤erential equation:


dy
( sin x csc2 y + 3y 2 ) dx + x3 + cos x cot y = 0:
APPENDIX B. ASSIGNMENTS 107

Name: Registration No.:

Assignment Problem B.2.3 Solve the following …rst-order initial-value prob-


lem:
dy
cos x ln y + sec2 x + y1 sin x + p 1 dx
= 0 ; y 4 = 1:
1 y2
APPENDIX B. ASSIGNMENTS 108

Name: Registration No.:

Assignment Problem B.2.4 Solve the following …rst-order initial-value prob-


lem:
2x 3x
y + 1+x 2 + 3e cos y dx + (x + sec2 y e3x sin y) dy = 0:
y (0) = 0:
APPENDIX B. ASSIGNMENTS 109

Final Answer of Assignment Problem B.2.1:


Exact di¤erential equation,
(x; y (x)) = xy 3 + y 2 cos x 2e3x + ln (sec y) = C:
Final Answer of Assignment Problem B.2.2:
Exact di¤erential equation,
(x; y (x)) = sin x cot y + y 3 + 3 ln x = C:
Final Answer of Assignment Problem B.2.3:
Exact di¤erential equation,
(x; y (x)) = sin x ln y + tan x + sin 1 y = 1 + 2 :
Final Answer of Assignment Problem B.2.4:
Exact di¤erential equation,
(x; y (x)) = xy + tan y + e3x cos y + ln (x2 + 1) = 1:
APPENDIX B. ASSIGNMENTS 110

B.3 Assignment 3 on Linear Di¤erential Equa-


tions
Assignment 3 - Mathematics 3
Linear Di¤erential Equations

Name: Registration No.:

Specialization: Date: / 20

Assignment Problem B.3.1 Solve the following di¤erential equation:


dy
x dx + 4y = x3 x:
APPENDIX B. ASSIGNMENTS 111

Name: Registration No.:

dy
Assignment Problem B.3.2 dx
+ 2 (tan 2x) y = 3e3x cos 2x
APPENDIX B. ASSIGNMENTS 112

Name: Registration No.:

Assignment Problem B.3.3 Solve the following …rst-order initial-value prob-


lem:
dy
dx
+ (cos x) y = 4x3 e sin x ; y (0) = 8:
APPENDIX B. ASSIGNMENTS 113

Name: Registration No.:

Assignment Problem B.3.4 Solve the following …rst-order initial-value prob-


lem:
dy
dx
+ 2y = 8xe 2x ; y (0) = 1:
APPENDIX B. ASSIGNMENTS 114

Final Answer of Assignment Problem B.3.1:


Linear di¤erential equation,
y = Cx 4 + 17 x3 51 x:
Final Answer of Assignment Problem B.3.2:
Linear di¤erential equation,
y = cos 2x (C + e3x ) :
Final Answer of Assignment Problem B.3.3:
Linear di¤erential equation,
y = cos x (6 + 2e5x ) :
Final Answer of Assignment Problem B.3.4:
Linear di¤erential equation,
y = e 2x (1 + 4x2 ) :
APPENDIX B. ASSIGNMENTS 115

B.4 Assignment 4 on Bernoulli Di¤erential


Equations
Assignment 4 - Mathematics 3
Bernoulli Di¤erential Equations

Name: Registration No.:

Specialization: Date: / 20

dy
Assignment Problem B.4.1 dx
+ x2 y = (x4 sec2 x) y 3 :
APPENDIX B. ASSIGNMENTS 116

Name: Registration No.:

dy 1
Assignment Problem B.4.2 dx 4
(tan x) y = (sin x) y 3 :
APPENDIX B. ASSIGNMENTS 117

Name: Registration No.:

dy 5
Assignment Problem B.4.3 6 dx + y cos x = y cos x:
APPENDIX B. ASSIGNMENTS 118

Name: Registration No.:

dy
Assignment Problem B.4.4 dx
= y (3xy 3 1) :
APPENDIX B. ASSIGNMENTS 119

Final Answer of Assignment Problem B.4.1:


Bernoulli di¤erential equation,
1
y2
= x4 (C 2 tan x) ) y = x2 pC 12 tan x :
Final Answer of Assignment Problem B.4.2:
Bernoulli di¤erential equation,
y 4 = sec x C + 2 sin2 x :
Final Answer of Assignment Problem B.4.3:
Bernoulli di¤erential
p equation,
6 sin x:
y= 1 + Ce
Final Answer of Assignment Problem B.4.4:
Bernoulli di¤erential equation,
1
y= p 3
Ce3x +3x+1
:
APPENDIX B. ASSIGNMENTS 120

B.5 Assignment 5 on Method of Undetermined


Coe¢ cients
Assignment 5 - Mathematics 3
Method of Undetermined Coe¢ cients

Name: Registration No.:

Specialization: Date: / 20

Assignment Problem B.5.1 Find the general solution of the following second-
order linear di¤erential equation, using the Method of Undetermined
Coe¢ cients:
y 00 + 4y 0 + 5y = 4ex 10xex :
APPENDIX B. ASSIGNMENTS 121

Name: Registration No.:

Assignment Problem B.5.2 Find the general solution of the following second-
order linear di¤erential equation, using the Method of Undetermined
Coe¢ cients:
y 00 2y 0 8y = 6e 2x :
APPENDIX B. ASSIGNMENTS 122

Name: Registration No.:

Assignment Problem B.5.3 Find the general solution of the following second-
order linear di¤erential equation, using the Method of Undetermined
Coe¢ cients:
y 00 + 6y 0 + 45y = (37x 35) e 2x :
APPENDIX B. ASSIGNMENTS 123

Name: Registration No.:

Assignment Problem B.5.4 Find the general solution of the following second-
order linear di¤erential equation, using the Method of Undetermined
Coe¢ cients:
y 00 3y 0 18y = 36 sin 3x 108 cos 3x:
APPENDIX B. ASSIGNMENTS 124

Name: Registration No.:

Assignment Problem B.5.5 Find the general solution of the following second-
order linear di¤erential equation, using the Method of Undetermined
Coe¢ cients:
y 00 2y 0 + 5y = 15x2 12x 29:
APPENDIX B. ASSIGNMENTS 125

Final Answer of Assignment Problem B.5.1:


yG:S: = yp + yh = ex xex + e 2x (C1 sin x + C2 cos x) :
Final Answer of Assignment Problem B.5.2:
yG:S: = yp + yh = xe 2x + C1 e 2x + C2 e4x :
Final Answer of Assignment Problem B.5.3:
yG:S: = yp + yh = (x 1) e 2x + C1 e 3x sin 6x + C2 e 3x
cos 6x:
Final Answer of Assignment Problem B.5.4:
yG:S: = yp + yh = 4 cos 3x + C1 e 3x + C2 e6x :
Final Answer of Assignment Problem B.5.5:
yG:S: = yp + yh = 3x2 7 + C1 ex cos 2x + C2 ex sin 2x:
APPENDIX B. ASSIGNMENTS 126

B.6 Assignment 6 on Method of Variation of


Parameters
Assignment 6 - Mathematics 3
Method of Variation of Parameters

Name: Registration No.:

Specialization: Date: / 20

Assignment Problem B.6.1 Verify that the function yh (x) = C1 x + C2 x2


is the homogeneous solution of the following linear di¤erential equation:
x2 y 00 2xy 0 + 2y = 5x2 .
Then use this yh (x) to …nd the its general solution, by means of the
Method of Variation of Parameters.
APPENDIX B. ASSIGNMENTS 127

Name: Registration No.:

Assignment Problem B.6.2 Find the general solution of the following


di¤erential equation, using the Method of Variation of Parameters:
y 00 5y 0 + 4y = 18xex :
APPENDIX B. ASSIGNMENTS 128

Name: Registration No.:

Assignment Problem B.6.3 Find the general solution of the following


di¤erential equation, using the Method of Variation of Parameters:
y 00 + 4y 0 + 4y = e 2x sec2 x.
APPENDIX B. ASSIGNMENTS 129

Name: Registration No.:

Assignment Problem B.6.4 Find the general solution of the following


di¤erential equation, using the Method of Variation of Parameters:
y 00 2y 0 + y = 6ex 24x2 ex .
APPENDIX B. ASSIGNMENTS 130

Final Answer of Assignment Problem B.6.1


yG:S: (x) = (C2 + 5 ln x) x2 + C1 x:
Final Answer of Assignment Problem B.6.2
yG:S: = y = (3x2 + 2x) ex + C1 ex + C2 e4x :
Final Answer of Assignment Problem B.6.3
2x
yG:S: = (ln (sec x) + C1 + C2 x) e :
Final Answer of Assignment Problem B.6.4
yG:S: = (3x2 2x4 + C1 + C2 x) ex :
APPENDIX B. ASSIGNMENTS 131

B.7 Assignment 7 on Cauchy-Euler Di¤eren-


tial Equations
Assignment 7 - Mathematics 3
Cauchy-Euler Di¤erential Equations

Name: Registration No.:

Specialization: Date: / 20

Assignment Problem B.7.1 Find the general solution of the following


di¤erential equation:
x2 y 00 2xy 0 + 2y = 5x2 .
APPENDIX B. ASSIGNMENTS 132

Name: Registration No.:

Assignment Problem B.7.2 Find the general solution of the following


di¤erential equation:
x2 y 00 + 6xy 0 14y = 27x2 .
APPENDIX B. ASSIGNMENTS 133

Name: Registration No.:

Assignment Problem B.7.3 Find the general solution of the following


di¤erential equation:
3x2 y" + 11xy 0 + 5y = 352x 9
APPENDIX B. ASSIGNMENTS 134

Final Answer of Assignment Problem B.7.1


This is a Cauchy-Euler di¤erential equation,
yG:S: (x) = 5x2 ln x + C1 x + C2 x2 :
Final Answer of Assignment Problem B.7.2
This is a Cauchy-Euler di¤erential equation,
yG:S: (x) = 3x2 ln x + C1 x2 + C2 x 7 :
Final Answer of Assignment Problem B.7.3
This is a Cauchy-Euler di¤erential equation,
9 1 5=3
yG:S: (x) = 2x + C1 x + C2 x :
APPENDIX B. ASSIGNMENTS 135

B.8 Assignment 8 on Direct Laplace Trans-


form
Assignment 8 - Mathematics 3
Direct Laplace Transform

Name: Registration No.:

Specialization: Date: / 20

Assignment Problem B.8.1 Find the Laplace Transforms of the following


functions:

1. q (t) = e6t (t cos 3t)

2. g (t) = t (e6t cos 3t)

3. w (t) = t e6t cos 3t

4. k (t) = te6t cos 3t

5. p (t) = (t 2) e6(t 2)
cos 3 (t 2) U (t 2)

d
6. h (t) = dt
(te6t cos 3t)
Rt
7. m (t) = 0
z e6z cos 3z dz

8. v (t) = cosh 4t cos 3t + cosh 4t cos 3t:


APPENDIX B. ASSIGNMENTS 136

Final Answer of Assignment Problem B.8.1:


s2 9
1. L (q) = (s 6)(s2 +9)2
:
s 6
2. L (g) = s2 ((s 6)2 +9)
:

1
3. L (w) = s(s 6)(s2 +9)
:
s2 12s+27
4. L (k) = (s2 12s+45)2
:

s2 12s+27
5. L (p) = (s2 12s+45)2
e 2s :

s(s2 12s+27)
6. L (h) = (s2 12s+45)2
:

s2 12s+27
7. L (m) = s(s2 12s+45)2
:

s 4 s+4 s2
8. L (v) = 2((s 4)2 +9)
+ 2((s+4)2 +9)
+ (s2 16)(s2 +9)
:
APPENDIX B. ASSIGNMENTS 137

B.9 Assignment 9 on Inverse Laplace Trans-


form
Assignment 9 - Mathematics 3
Inverse Laplace Transform

Name: Registration No.:

Specialization: Date: / 20

Assignment Problem B.9.1 Find the Inverse Laplace Transforms of the


functions F; G; K:
5s 44
1. F (s) = (s 6)2 (s+1)
e 3s :
s+7
2. G (s) = ln s 1
:
s+3
3. K (s) = (s+3)2 +4
e 6s :
APPENDIX B. ASSIGNMENTS 138

Name: Registration No.:

Assignment Problem B.9.2 Find the Inverse Laplace Transforms of the


functions F; G; K:

1. F (s) = 7e 3s
+ 3s e 4s
+ 10
s2 25
e 6s 12
(s+6)4
e 8s :
1 s
2. G (s) = cot 4
:
s 6
3. K (s) = s2 12s+40
e 7s :
APPENDIX B. ASSIGNMENTS 139

Name: Registration No.:

Assignment Problem B.9.3 Find the Inverse Laplace Transforms of the


functions F; G; K:
2s2 13s 18
1. F (s) = (s 2)(s2 +2s+10)
e 5s :
s 4
2. G (s) = s2 +4s+13
e 7s :

3. K (s) = 3e 2s
+ 4s e 5s
+ s
s2 4
e 3s :
APPENDIX B. ASSIGNMENTS 140

Final Answer of Assignment Problem B.9.1:


1
1. L (F ) = e6(t 3)
2 (t 3) e6(t 3)
e (t 3)
U (t 3) :
1 et e 7t
2. L (G) = t
:
1 3(t 6)
3. L (K) = e cos 2 (t 6) U (t 6) :

Final Answer of Assignment Problem B.9.2:


1
1. L (F ) =
7 (t 3)+3U (t 4)+2 sinh 5 (t 6) U (t 6) 2 (t 8)3 e 6(t 8)
U (t 8) :
1 sin 4t
2. L (G) = t
:
1
3. L (K) = e6(t 7)
cos 2 (t 7) U (t 7) :

Final Answer of Assignment Problem B.9.3:


1 (t 5)
1. L (F ) = e (sin 3 (t 5) + 4 cos 3 (t 5)) 2e2(t 5)
U (t 5) :
1 2(t 7)
2. L (G) = e [cos 3 (t 7) 2 sin 3 (t 7)] U (t 7) :
1
3. L (K) = 3 (t 2) + 4U (t 5) + cosh 2 (t 3) U (t 3) :
APPENDIX B. ASSIGNMENTS 141

B.10 Assignment 10 on Solution of Fredholm


Integral Equations
Assignment 10 - Mathematics 3
Solution of Fredholm Integral Equations by Laplace Transform

Name: Registration No.:

Specialization: Date: / 20

Assignment Problem B.10.1 Use the Convolution Theorem to solve the


following integral equation:
Z t
f (t) = cos 3t sin 3t + 6 f (z) cos 3 (t z) dz:
0
APPENDIX B. ASSIGNMENTS 142

Name: Registration No.:

Assignment Problem B.10.2 Use the Convolution Theorem to solve the


following integral equation:
Z t
2t
f (t) = 2 sin 2t + e 4 f (z) cos 2 (t z) dz:
0
APPENDIX B. ASSIGNMENTS 143

Name: Registration No.:

Assignment Problem B.10.3 Use the Convolution Theorem to solve the


following integral equation:
Zt
f (t) = 3 sin 5t 8 f (z) cos 5 (t z) dz:
0
APPENDIX B. ASSIGNMENTS 144

Name: Registration No.:

Assignment Problem B.10.4 Use the Convolution Theorem to solve the


following integral equation:
Zt
f (t) = cosh 3t 3 f (z) e3(t z)
dz:
0
APPENDIX B. ASSIGNMENTS 145

Final Answer of Assignment Problem B.10.1:


f (t) = e3t :
Final Answer of Assignment Problem B.10.2:
f (t) = e 2t (1 + 4t2 ) :
Final Answer of Assignment Problem B.10.3:
f (t) = 5e 4t sin 3t:
Final Answer of Assignment Problem B.10.4:
f (t) = e 3t :
APPENDIX B. ASSIGNMENTS 146

B.11 Assignment 11 on Laplace Transform


and Initial-value Problems (1)
Assignment 11 - Mathematics 3
Laplace Transform & Initial-value Problems (1)

Name: Registration No.:

Specialization: Date: / 20

Assignment Problem B.11.1 Use the Laplace Transform to solve the


following initial-value problem:
y 00 + 4y 0 + 5y = 10et ; y (0) = 0; y 0 (0) = 5:
APPENDIX B. ASSIGNMENTS 147

Name: Registration No.:

Assignment Problem B.11.2 Use the Laplace Transform to solve the


following initial-value problem:
y 00 2y 0 8y = 6e 2t ; y (0) = 0; y 0 (0) = 1:
APPENDIX B. ASSIGNMENTS 148

Name: Registration No.:

Assignment Problem B.11.3 Use the Laplace Transform to solve the


following initial-value problem:
y 00 3y 0 + 2y = e2t , y (0) = 5 , y 0 (0) = 10
APPENDIX B. ASSIGNMENTS 149

Name: Registration No.:

Assignment Problem B.11.4 Use the Laplace Transform to solve the


following initial-value problem:
y 00 8y 0 + 16y = 6te4t ; y (0) = 1; y 0 (0) = 4:
APPENDIX B. ASSIGNMENTS 150

Final Answer of Assignment Problem B.11.1:


y (t) = et + e 2t (2 sin t cos t)
Final Answer of Assignment Problem B.11.2:
y (t) = te 2t :
Final Answer of Assignment Problem B.11.3:
y = et + 4e2t + te2t
Final Answer of Assignment Problem B.11.4:
y = e3t + te4t
APPENDIX B. ASSIGNMENTS 151

B.12 Assignment 12 on Laplace Transform


and Initial-value Problems (2)
Assignment 12 - Mathematics 3
Laplace Transform & Initial-value Problems (2)

Name: Registration No.:

Specialization: Date: / 20

Assignment Problem B.12.1 Use the Laplace Transform to solve the


following initial-value problem (system of di¤erential equations):

dx
3x + 2y = 0;
dt
dx dy
3 13x + 2 = 0;
dt dt
x (0) = 1 , y (0) = 0:
APPENDIX B. ASSIGNMENTS 152

Name: Registration No.:

Assignment Problem B.12.2 Use the Laplace Transform to solve the


following initial-value problem (system of di¤erential equations):

dx
4x + 2y = 2te4t ;
dt
dy
x+3 12y = 0:
dt
x (0) = 3 , y (0) = 0 .
APPENDIX B. ASSIGNMENTS 153

Name: Registration No.:

Assignment Problem B.12.3 Use the Laplace Transform to solve the


following initial-value problem (system of di¤erential equations):

dy 2t
8x = 8te ;
dt
dx 2t
2x y = 2te ;
dt
x (0) = 0 , y (0) = 0:
APPENDIX B. ASSIGNMENTS 154

Name: Registration No.:

Assignment Problem B.12.4 Use the Laplace Transform to solve the


following initial-value problem (system of di¤erential equations):

dx dy
+ 4 + 4y = 3e t ;
dt dt
dy
x = te t ;
dt
x (0) = 1 , y (0) = 0 .
APPENDIX B. ASSIGNMENTS 155

Final Answer of Assignment Problem B.12.1:


x (t) = e3t cos 2t y (t) = e3t sin 2t .
Final Answer of Assignment Problem B.12.2:
x (t) = 3e4t , y (t) = te4t .
Final Answer of Assignment Problem B.12.3:
x (t) = t2 e 2t , y (t) = 4t2 e 2t .
Final Answer of Assignment Problem B.12.4:
x=e t, y = te t .

You might also like